Problemas Resolvidos Cálculo III

349
Problemas Resolvidos de Integrais M´ ultiplas Uma apostila de aux´ ılio ` a compreens˜ ao ao alculo Diferencial Integral III Prof. Dr. Beto Rober Saavedra Universidade Federal do Vale de S˜ ao Francisco Colegiado de Engenharia de Produ¸ ao http:www.univasf.edu.br/producao/ 1

Transcript of Problemas Resolvidos Cálculo III

Problemas Resolvidosde

Integrais Multiplas

Uma apostila de auxılio a compreensao ao

Calculo Diferencial Integral III

Prof. Dr. Beto Rober Saavedra

Universidade Federal do Vale de Sao Francisco

Colegiado de Engenharia de Producao

http:www.univasf.edu.br/producao/

1

COLABORADORES:

Adalto Liberato de Moura Neto

Anderson Matias da Silva

Andre Soares de Siqueira

Barbara Oliveira Lima

Bruna Parente Granja

Carla Daniela Pereira da Silva

Catiane Queite Simas de Santana

Cyntia de castro Araujo Pereira

Daniel dos Santos Costa

Denisson Augusto Bastos Leal

Diego Galvao Campos Oliveira

Edmilson Jonatas Santos de Brito

Edmo Henrique Martins

Edson Silva Lopes

Eldon de Aquino Costa

Elton Barbosa Santos

Emanuela oliveira dos Santos Paiva

Erick galvao Santana

Eugenio dos Santos de Castro Campos

Francisco Caio Silva Ladislau

Francisco Elde Oliveira Junior

Geilson Ribeiro da Silva

Gilmara Pires Granja

Giovane Alves Bonfim Dias

Glaucia Suerdia Gomes do Nascimento

Gustavo Alves Raphael

Henrique Martins de Miranda

Ilenia Evangelista Rodrigues

Jackson Yanno Araujo de Carvalho

Jadson Patrick Santana de Moraes

Jamile Costa do Nascimento

Jose Antunes da Silva Neto

Jose Augusto Barreira Fonseca Filho

Juman Fernandes Santos Sousa2

Leila Oliveira Santos

Lucas Matheus de Oliveira Barbosa

Luiz Henrique Coimbra Coelho Gonzaga

Marcelo Henryque Costa de Souza

Maria Augusta Ferreira da Costa Andrade

Matheus Moreira Santiago

Natasha Camilo Dias

Osvaldo Campelo de Mello Vasconcelos

Paula Lima Alves

Paulo Henrique Rocha Pereira

Paulo Vitor Torres barbosa

Pedro de Brito Cavalcanti Neto

Pedro Henrique Araujo Sobral

Raquel Rafael de Freitas Silva

Renan Franca da Silva

Ricardo Barbosa de Siqueira

Ricardo Euller Dantas e Silva

Roberta Daniela da Silva Santos

Simone do Nascimento Luz

Tayron Juliano Souza

Thiago Bruo Rodrigues de Rezende Oliveira

Ulderico Rios Oliveira

Vanderleia Dias da Silva

Victor Marcilio de Araujo Souza Peixoto

3

.

′′A diferena entre sonho e realidade e a quantidade certa de tempo e trabalho.

′′

William Douglas

4

1. Determine o volume da regiao solida E limitada pela superfıcie z =

sen(x2 + y2) e a regiao plana circular de centro (0, 0, 0) e raio√

2π.

Solucao . Observar que parte da regiao E fica acima da regiao plana circular de

centro (0, 0, 0) e raio√

π e a outra parte embaixo da regiao plana limitada pelas

circunferencias concentricas de centro (0, 0, 0) e raios√

π e√

2π respetivamente. Logo,

usando coordenadas polares, o volume da regiao E e

V =

∫ 2π

0

∫ √π

0

rsen(r2)drdθ −∫ 2π

0

∫ √2π

√π

rsen(r2)drdθ

=

∫ 2π

0

−1

2cos(r2)

√π

0 dθ +

∫ 2π

0

1

2cos(r2)

√2π√π

= 2π + 2π = 4π

5

2. Calcular∫

R

∫cos(

y+xy−x

)

, onde R e a regiao trapezoidal com vertices

(2, 0), (4, 0), (0, 4) e (0, 2).

Solucao . Vamos fazer a mudanca de variaveis:

u = x + y v = x − y

Essas equacoes definem a transformacao inversa T−1 do plano xy para o plano uv. E,

a transformacao T, do plano uv para o plano xy, e dada pelas equacoes

x =1

2(u + v) y =

1

2(u − v)

O jacobiano de T e ∂(x,y)∂(u,v) = −1

2 . A transformacao T transforma uma regiao S no

plano uv na regiao R como mostra a figura abaixo:

Logo:

R

cos

(y + x

y − x

)

dA =1

2

S

cos(u

v)dudv

=1

2

∫ 4

2

∫ v

−v

cos(u

v)dudv

=1

2

∫ 4

2

v2sen2(u

v)|v−vdv

= 0

6

3. Determine o volume da regiao solida E limitada pela superfıcie z =

(x2 + y2)sen(x2 + y2) e a regiao plana circular de centro (0, 0, 0) e raio√

2π.

Solucao . Observar que parte da regiao E fica acima da regiao plana circular de

centro (0, 0, 0) e raio√

π e a outra parte embaixo da regiao plana limitada pelas

circunferencias concentricas de centro (0, 0, 0) e raios√

π e√

2π respetivamente. Logo,

usando coordenadas polares, o volume da regiao E e

V =

∫ 2π

0

∫ √π

0

r3sen(r2)drdθ −∫ 2π

0

∫ √2π

√π

r3sen(r2)drdθ

Agora, pela substituicao u = r2 e 12du = dr e os respetivos limites de integracao r =

0 → u = 0, u =√

π → u = π e r =√

2π → u = 2π, obtermos:

V =1

2

∫ 2π

0

∫ π

0

usen(u)drdθ − 1

2

∫ 2π

0

∫ 2π

π

usen(u)drdθ

=1

2[

∫ 2π

0

−ucos(u) + sen(u)π0dθ −

∫ 2π

0

−ucos(u) + sen(u)2ππ dθ]

= π + 3π = 4π

7

4. Determine o volume da regiao solida limitada pelas esferas

(x − 1)2 + (y − 1)2 + (z − 1)2 = 9 e (x − 4)2 + (y − 4)2 + (z − 4)2 = 16.

Solucao A distancia entre os centros das esferas dadas e 3√

3. Calcular o volume da

regiao solida dada e mesma coisa que calcular o volume da regiao solida limitada pelas

esferas

x2 + y2 + z2 = 9 e x2 + y2 + (z − 3√

3)2 = 16.

Observar a figura seguinte:

Para encontrar o plano paralelo ao plano XY sobre o que descansa a intersecao das

esferas, precisamos resolver a equacao

16 − z2 = 9 − (z − 3√

3)2

Isto e, o plano procurado e z = 17√

39 . Logo, a regiao E fica acima da regiao plana

8

x2 + y2 ≤ 14327 . O volume de E e

V =

∫ 2π

0

∫√

14327

0

r√

16 − r2 − [3√

3r − r√

9 − r2]drdθ

=

=

∫ 2π

0

−1

3(16 − r2)

32 |√

14327

0 − 3√

3r2

2|√

14327

0 − 1

3(9 − r2)

32 |√

14327

0 dθ = 2, 524....

9

5. Determine a area da parte do cone z2 = 4(x2 + y2) entre os planos z =

1 e z = 2.

Solucao . Observar que a parte do cone entre os planos z = 1 e z = 2. fica acima da

regiao R no plano XY limitada pelas circunferencias x2 + y2 = 14 e x2 + y2 = 1.

Logo, a area procurada e

A =

R

∫√

1 + (∂z

∂x)2 + (

∂z

∂y)2dxdy

=

R

∫√

1 +x2

4(x2 + y2)+

y2

4(x2 + y2)dxdy

=

R

∫ √

5

4dxdy

=

5

4

∫ 2π

0

∫ 1

12

rdrdθ =3

5

4.

10

6. Seja B a bola fechada x2 + y2 + z2 ≤ 4.

(a) Provar por meio de Mudanca de Variaveis que

∫ ∫

B

e3x2+xdV =

∫ ∫

B

e3z2+zdV

(b) Calcular∫ ∫

B

∫ex2

+ ex2+3z3

dV∫ ∫

B

∫ey2 + ez2+3y3dV

Solucao . As coordenadas no espaco R3 podem ser dadas pelas variaveis (x, y, z) ou

pelas variaveis (u, v, w).

(a) Consideramos a mudanca de variaveis dada por

u = z v = y w = x

Observamos que a mudanca de variaveis dada transforma a a bola fechada x2 +

y2 + z2 ≤ 4 na bola fechada u2 + v2 + w2 ≤ 4. Alem disso, | ∂(x,y,z)∂(u,v,w) | = 1.

Logo,

∫ ∫

B

e3x2+xdV =

∫ ∫

B

e3w2+wdudvdw =

∫ ∫

B

e3w2+wdV =

∫ ∫

B

e3z2+zdV

(b) Sejam as transformacoes inversas T−11 dada por

u = y v = x w = z

e T−12 dada por

u = y v = z w = x

Observamos que as duas transformacoes T1 e T2 levam a bola fechada B na bola

fechada B. Alem disso, ambos os modulos dos Jacobianos de T1 e T2 sao igual a

1. Logo, como acima, temos

∫ ∫

B

ex2

dV =

∫ ∫

B

ey2

dV e

∫ ∫

B

ex2+3z3

dV =

∫ ∫

B

ez2+3y3

dV

Logo,∫ ∫

B

∫ex2

+ ex2+3z3

dV∫ ∫

B

∫ey2 + ez2+3y3dV

= 1

11

7. (a) Encontrar todos os pontos (x, y) do plano tais que |x| + |y| = 1.

(b) Calcular a integral dupla∫

B

∫ex+ydA, onde B = {(x, y) ∈ R

2 : |x| + |y| ≤3}.

Solucao .

(a) Denotamos o conjunto E = {(x, y) ∈ R2 : |x| + |y| = 1}. Podemos escrever esse

conjunto como segue

E = E1

E2

E3

E4

onde

E1 = {(x, y) ∈ R2 : x+y = 1, x ≥ 0, y ≥ 0}, E2 = {(x, y) ∈ R

2 : x+y = 1, x ≥ 0, y ≤ 0}

E3 = {(x, y) ∈ R2 : x+y = 1, x ≤ 0, y ≤ 0}, E4 = {(x, y) ∈ R

2 : x+y = 1, x ≥ 0, y ≤ 0}

Observamos que E1 e um segmento de reta que liga os pontos (1, 0) e (0, 1), E2 e

um segmento de reta que liga os pontos (0, 1) e (−1, 0), E3 e um segmento de reta

que liga os pontos (−1, 0) e (0,−1), e E4 e um segmento de reta que liga os pontos

(0,−1) e (1, 0) (ver figura 1).

Figura 1: Conjunto E.

(b) Como no item anterior, prova-se que o conjunto de pontos (x, y) tais que |x|+|y| =

3 e um losango com vertices (3, 0), (0, 3), (−3, 0) e (0,−3). Logo, B e a regiao

limitada por esse losango(ver figura 2). Seja a transformacao inversa T−1 dada

12

Figura 2: O conjunto B transforma-se num Quadrado S de lado de comprimento igual a 3.

por u = x + y v = x − y. Para determinar a regiao S do plano

uv correspondente a B, notamos que os lados de B estao sobre as retas

x + y = 3, x − y = 3, x + y = −3, x − y = −3

e as retas correspondentes do plano uv sao

u = 3, v = 3, u = −3, v = 3

Entao , a regiao S e o quadrado com vertices (3, 3), (−3, 3), (−3,−3) e (3,−3) como

mostra a figura 2. Por outro lado, o valor absoluto do Jacobiano da transformacao T e

igual a∣∣∣∂(x,y)∂(u,v)

∣∣∣ = 1

2 . Logo,

B

ex+ydA =

B

eu

∣∣∣∣

∂(x, y)

∂(u, v)

∣∣∣∣dA

=1

2

S

eudA

=1

2

∫ 3

−3

∫ 3

−3

eududv

= 3(e3 − e−3).

13

8. Determine o volume do solido com vertices (0,0,0), (0,0,1), (0,2,0) e (2,2,0).

Solucao . Para seguir o raciocınio, observar a Figura 1. A base do Tetraedro e um

retangulo, que denotarmos por R, determinado pelas retas y = x, y = 2, x = 0. A

regiao R e a projecao ortogonal do plano, determinado pelos pontos (0,0,1),(0,2,0),(2,2,0),cuja

equacao Cartesiana e z = 2−y2 . Logo, o volume requerido e

Figura 3: Tetraedro

∫ ∫

R

2 − y

2dA =

∫ 2

0

∫ y

0

2 − y

2dxdy

=

∫ 2

0

2y − y2

2dy

=y2

2− y3

6|20 =

2

3.

14

9. Calcular∫ ∫

R

tang(18x2 + 8y2)dA,

onde R e a regiao do primeiro quadrante limitada pela elipse 9x2 + 4y2 = 14 .

Solucao . Pela mudanca de coordenadas:

x =1

3rcos(θ), y =

1

2rsen(θ); 0 ≤ r ≤ 1

2, 0 ≤ θ ≤ π

2.

temos:

∫ ∫

R

tan(18x2 + 8y2)dA =1

6

∫ π2

0

∫ 12

0

tang(2r2)rdrdθ

=1

24

∫ π2

0

−ln(cos(2r2))|120 dθ

=1

24

∫ π2

0

−ln(cos(1

2))dθ

=−π

48ln(cos(

1

2))

15

10. Calcular∫

R

∫(y − x)sen

(y+xy−x

)

, onde R e a regiao trapezoidal com vertices

(2, 0), (4, 0), (0, 4) e (0, 2).

Solucao . Vamos fazer a mudanca de variaveis:

u = x + y v = x − y

Essas equacoes definem a transformacao inversa T−1 do plano xy para o plano uv. E,

a transformacao T, do plano uv para o plano xy, e dada pelas equacoes

x =1

2(u + v) y =

1

2(u − v)

O jacobiano de T e ∂(x,y)∂(u,v) = −1

2 . A transformacao T transforma uma regiao S no

plano uv na regiao R como mostra a figura abaixo:

Logo:

R

(y − x)sen

(y + x

y − x

)

dA =1

2

S

vsen(u

v)dudv

=1

2

∫ 4

2

∫ v

−v

vsen(u

v)dudv

= −1

2

∫ 4

2

v2cos(u

v)|v−vdv

= 0

16

11. Calcule a integral

a)∫ 1

0

∫ 2

1x cos xy dxdy

Solucao.

Fazendo a substituicao simples, temos:

xy = u

du = x dy

Assim,

∫ 2

1

[∫ 1

0cosudu

]

dx =∫ 2

1[senxy]

10 dx =

∫ 2

1senxdx = [− cos x]

21 =

= − cos 2 + cos 1

17

12. Calcule∫ ∫

R

ex2+y2

dydx,

onde R a regiao semicircular limitada pelo eixo X e pela curva , y =√

1 − x2.

Solucao . Em coordenadas cartesianas, a integral em questao e uma intefral nao el-

ementar e nao existe nenhuma maneira direta de integrar ex2+y2

em relacao a x ou y.

Ainda assim essa integral e outras integrais como essa sao importantes em matematica

— em estatıstica por exemplo — e queremos encontrar uma maneira de calcula-la. As

coordenadas polares servem para isso. A substituicao de x = r cosθ , y = r sen θ e a troca

de dydx por rdrd θ nos permitem calcular a integral como :∫∫

Rex2+y2

dydx =∫ π

0

∫ 1

0er2

rdrdθ =∫ π

0

[12er2

]1

0dθ =

∫ π

012 (e − 1)dθ = π

2 (e − 1)

O r em r dr dθ era justamente o que precisavamos para integrar. Sem isso, estariamos

impedidos de prosseguir, como no comeco .

18

13. Encontre o momento polar da inercia em relacao a origem de uma placa fina

de densidade θ(x, y) = 1 limitada pelo quarto de circunferencia x2 + y2 = 1 no

primeiro quadrante .

Solucao . Em coordenadas cartesianas, o momento polar e o valor da integral

∫ 1

0

∫ √1−x2

0

dydx

Integrando em relacao a y, temos :

∫ 1

0

(

x2√

1 − x2 +(1 − x2)

32

3

)

dx

Uma integral difıcil de calcular sem tabelas.

As coisas melhoram se mudamos a interal original para coodenadas polares. Substituindo

x = r cos θ , y = r sen θ e trocando dxdy por r dr θ, obtemos :

∫ 1

0

∫ √1−x2

0

(x2 + y2)dydx =

∫ π2

0

∫ 1

0

(r2)rdrdθ

=

∫ π2

0

[r4

4

]r=1

r=0

dθ =

∫ π2

0

1

4dθ =

π

8

Por que a transformacao em coordenadas polares e tao eficaz aqui ? Um motivo e que

x2 + y2 e simplificada para r2 . Outro motivo e que os limites de integracao tornam-se

constantes.

19

14. Identificando a regiao de integracao . Esbocar e calcular∫ 0

−1

∫ 1−x

−2x

dydx +

∫ 2

0

∫ 1−x

− x2

dydx

Solucao . Com o auxilio da figura abaixo e possıvel verificar a regiao na qual se deseja

calcular.

Figura 4: Grafico.

∫ 0

−1

∫ 1−x

−2x

dydx +

∫ 2

0

∫ 1−x

− x2

dydx

=

∫ 0

−1

1 + x dx +

∫ 2

0

1 − x

2dx

= x +x2

2

∣∣∣∣

0

−1

+ x − x2

4

∣∣∣∣

2

0

= −(

−1 +1

2

)

+ (2 − 1) =3

2

20

15. Encontre a area dentro da lemniscata r2 = 4cos2θ .

Solucao . Tracamos o grafico da lemniscata para determinar os limites de integracao e

vemos que a area total e quatro vezes a area da porcao no primeiro quadrante.

a = 4

∫ π4

0

∫√

4cos2θ

0

rdrdθ = 4

∫ π4

0

[r2

2

]r=√

4cos2θ

r=0

dθ = 4

∫ π4

0

2cos2θdθ = 4sen2θ

]π4

0

= 4.

21

16. Calcule

∫ ∫

B

ex2+y2

dxdy B : (x, y) 1 ≤ x2 + y2 ≤ 4 ,−x ≤ y ≤ x, x ≥ 0.

Solucao .

X = rcosθ

Y = rsenθ

∫ π

π2

∫ 4

1

er2 · rdrdθ =1

2

π2

π =

∫ π

π2

∫ 4

1

eududθ =1

2

∫ π

π2

eu

∣∣∣∣

4

1

=1

2

∫ π

fπ2

e4 − e′ =1

2(θe4 − θe′)π

π2

=1

2

[

(πe4 − πe) − (π

2e4 − π

2e)]

=1

2

(

πe4 − πe − π

2e4 +

π

2e)

=1

2

(2πe4 − 2πe − πe4 + πe

2

)

=1

4(πe4 − π)

22

17. Calcule∫ ∫

R

f(x, y)dxdy

onde R e a regiao triangular com vertices (0,0), ( 1, 0) e (0 , 1)

Solucao .

x − x2

x2 − x1=

y − y2

y2 − y1=

x − 0

0 − 1

=y − 1

1 − 0→ x

−1=

y − 1

1= x = −y + 1

−y = x − 1 → y = 1 − x

∫ 1

0

∫ 1−x

0

x2 + y2dydx =

∫ 1

0

x2y +y3

3

∣∣∣∣

1−x

0

dx =

∫ 1

0

x2(1 − x) +(1 − x)3

3dx

∫ 1

0

x2dx −∫ 1

0

x3dx +

∫ 1

0

(1 − x)3

3dx =

x3

3

∣∣∣∣

1

0

− x4

4

∣∣∣∣

1

0

− 1

3

(1 − x)4

4

∣∣∣∣

1

0

=1

3− 1

4− 1

3

(

0 − 1

4

)

=1

3− 1

4+

1

12=

1

6

23

18. Encontre os limites da integracao para integrar f(r, θ) sobre a regiao R que

esta dentro da cardioide r = 1 + cosθ e fora da circunferencia r = 1 .

Solucao

Passo 1 : Um esboco. Esbocamos a regiao e identificamos as curvas limitantes.

Passo 2 : Os limites de integracao de r. Um raio tıpico a partir da origem entra em

R onde r = 1 e sai onde r = 1 + cosθ.

Passo 3 : Os limites de integracao de θ . Os raios a partir da origem que apresentam

interseccao com R variam de θ = −π/2 a θ = π/2. A integral e :

∫ π2

−π2

∫ 1+cosθ

1

f(r, θ)rdrdθ

Se f(r, θ) e a funcao constante cujo valor e 1 , entao a integral de f sobre r e a area de R .

A area de uma regiao R fechada e limitada no plano de coordenadas polares e

a =

∫ ∫

R

rdrdθ.

Como seria de esperar, essa formula para a area e condizente com todas as formulas

anteriores, embora nao provemos esse fato .

24

19. Identificando a regiao de integracao . Esbocar e calcular

∫ 2

0

∫ 0

x2−4

dydx +

∫ 4

0

∫ √x

0

dydx

Solucao .

Figura 5: Grafico.

∫ 2

0

4 − x2 dx +

∫ 4

0

x1/2 dx

= 4x − x3

3

∣∣∣∣

2

0

+2

3x3/2

∣∣∣∣

4

0

=

(

8 − 8

3

)

+16

3=

32

3

25

20. Encontre o volume da regiao D limitada pelas superfıcies z = x2 + 3y2 z =

8 − x2 − y2.

Solucao . O volume e

v =

∫ ∫

R

dzdydx,

a integral de f(x, y, z) = 1 sobre D . Para encontrarmos os limites de integracao para

calcular a integral, seguimos estes passos :

Passo 1 : Um esboco. As superf’icies apresentam interseccao no cilindro elıptico

x2 + 3y2 = 8 − x2 − y2 ou x2 + 2y2 = 4. A fronteira da regiao R ( a projecao de D spbre

o plano xy ) e uma elipse com a mesma equacao : x2 + 2y2 = 4 . A fronteira superior de

R e a curva y =√

(4 − x2)/2 . A fronteira inferior e a curva y = −√

(4 − x2)/2 .

Passo 2 : Os limites de integracao de Z . A reta M que passa por um ponto tıpico (x, y)

em R que e paralela ao eixo Z entra em D em z = x2 + 3y2 e sai em z = 8 − x2 − y2 .

Passo 3 : Os limites de integracao de y. A reta L que passa por (x, y) que e paralela ao

eixo y entra em R em y = −√

(4 − x2)/2 e sai em y =√

(4 − x2)/2 .

Passo 4 : Os limites de integracao de x . Quando L varre R, o valor de X varia de x = −2

em (−2, 0) a x = 2 em (2, 0, 0) . O volume e

v =∫ ∫

R

∫dzdydx

=∫ 2

−2

∫√

(4−x2)/2

−√

(4−x2)/2

∫ 8−x2−y2

x2+3y2 dzdydx

=∫ 2

−2

∫√

(4−x2)/2

−√

(4−x2)/2(8 − 2x2 − 4y2)dydx

=∫ 2

−2

[(8 − 2x2)y − 4

3y3]√

(4−x2)/2

−√

(4−x2)/2dx

=∫ 2

−2

(

2(8 − 2x2)√

4−x2

2 − 83

(4−x2

2

) 32

)

dx

∫ 2

−2

[

8(

4−x2

2

) 32 − 8

3

(4−x2

2

) 32

]

dx = 4√

23

∫ 2

−2(4 − x2)

32 dx

= 8π√

2 Unidades cubicas.

26

21. Determine os limites de integracao para calcular a integral tripla de uma

funcao f(x, y, z) sobre o tetraedro D com vertices (0, 0, 0) (1, 1, 0) (0, 1, 0) e (0, 1, 1).

Solucao .

Passo 1 : Um esboo. Esbocamos D junto com sua projecao R no plano xz. A superfıcie

limitante superior a direita de D esta no plano y = 1 . A superf’icie limitante inferiror

a esquerda esta no plano y = x + z . A fronteira superior de R e a reta z = 1 − x . A

fronteira inferior e a reta z = 0

Passo 2 : Os limites de integracao de y. A reta que passa por um ponto tıpico (x, z)

em R que e paralela ao eixo y entra em D em y = x + z e sai em y = 1 .

Passo 3 : Os limites de integracao de Z . A reta L que passa por (x, z) que e paralela

ao eixo z entra em R e em z = 0 sai em z = 1 − x

Passo 4 : Os limites de integracao de x. A medida que L varre R , o valor de x varia

de x = 0 a x = 1 . A integral e

∫ 1

0

∫ 1−x

0

∫ 1

x+z

F (x, y, z)dydzdx.

27

22. Esboce a regiao , inverta a ordem e calcule a integral∫ ∫

R

(y − 2x2)dA,

onde R e a regiao limitada pelo quadrado |x| + |y| = 1

Solucao .

Figura 6: Grafico.

=

∫ 0

−1

∫ x+1

−x−1

(y − 2x2)dydx +

∫ 1

0

∫ 1−x

x−1

(y − 2x2)dydx

=

∫ 0

−1

1

2y2 − 2x2 − 2x2y

∣∣∣∣

x+1

−x−1

dx +

∫ 1

0

1

2y2 − 2x2y

∣∣∣∣

1−x

x−1

dx

=

∫ 0

−1

1

2(x + 1)2 − 2x2(x + 1) − 1

2(−x − 1)2 + 2x2(−x − 1)dx

+

∫ 1

0

1

2(1 − x)2 − 2x2(1 − x) − 1

2(x − 1)2 + 2x2(x − 1)dx

= −4

∫ 0

−1

(x3 + x2)dx + 4

∫ 1

0

(x3 − x2)dx

= −4

[x4

4+

x3

3

]0

−1

+ 4

[x4

4− x3

3

]1

0

= 4

[(−1)4

4+

(−1)3

3

]

+ 4

(1

4− 1

3

)

= 8

(3

12− 4

12

)

= − 8

12= −2

3

28

23. Encontre o valor medio de f(x, y, z) = xyz sobre o cubo limitado pelos planos

coordenados e pelos planos x = 2, Y = 2, z = 2 no primeiro octante

Solucao . Esbocamos o cubo com detalhes suficientes para mostrar os limites de inte-

gracao . Depois usamos a equacao (4) para calcular o valor medio de F sobre o cubo.

O volume do cubo e (2)(2)(2) = 8 . O valor da integral de F sobre o cubo e:

∫ 2

0

∫ 2

0

∫ 2

0xyzdxdydz =

∫ 2

0

∫ 2

0

[x2

2 yz]x=2

x=0dydz =

∫ 2

0

∫ 2

02yzdydz

∫ 2

0

[y2z]y=2

y=0dz =

∫ 2

04zdz =

[2z2]2

0= 8

Com esses valores, a equacao (4) da

valor medio de xyz sobre o cubo

=1

volume

∫ ∫

cubo

xyzdV =1

8(8) = 8

Ao calcularmos a integral, escolhemos a ordem dxdydz, mas qualquer um das outras cinco

ordens tambem funciona

29

24. Esboce a regiao , inverta a ordem e calcule a integral∫ ∫

R

xydA,

onde R e a regiao limitada pelas retas y = x y = 2x x + y = 2

Solucao .

Figura 7: Grafico.

∫ 23

0

∫ 2x

0

xydydx +

∫ 1

23

∫ 2−x

x

xydydx

=

∫ 23

0

1

2xy2

∣∣∣∣

2x

x

dx +

∫ 1

23

1

2xy2

∣∣∣∣

2−x

x

dx

=

∫ 23

0

2x3 − 1

2x3dx +

∫ 1

23

1

2x(2 − x)2 − 1

2x3dx

=

∫ 23

0

3

2x3dx +

∫ 1

23

2x − x2dx

=3

8x4

∣∣∣∣

23

0

+ x2 − 2

3x3

∣∣∣∣

1

23

=3

8· 16

81+ 1 − 2

3−[4

9−(

2

3· 8

27

)]

=6

81+

27

81−(

36

81− 16

81

)

=13

81

30

25. Encontre um centroide (δ = 1) do solido limitado pelo cilindro x2 + y2 = 4 e

limitado acima pelo paraboloide z = x2 + y2 e abaixo pelo plano xy

Solucao .

Passo 1 : Um esboco. Esbocamos o solido, limitado acima pelo paraboloide z = r2 e

abaixo pelo plano z = 0 . Sua base R e o disco |r| ≤ 2 no plano xy .

O centroide do s’olido (x, y, z) est’a sobre seu eixo de simetria, neste caso o eixo z. Isso

faz x = y = 0 . Para encontrarmos z , dividimos o primeiro momento Mxy pela massa M.

Passo 2 : Os limites de z . Uma reta M que passa por um ponto tıpico (r, θ) na base

paralela ao eixo z entra no solido en z = 0 e sai em z = r2 .

Passo 3 : Os limites de r . Um raio L que passa por (r, θ) a partir da origem entra em r

em r = 0 e sai em r = 2 .

Passo 4 : Os limites de θ . A medida que L varre a base no sentido anti -horario, o angulo

θ que ele faz com o eixo x positivo varia de θ = 0 a θ = 2π. O valor de Mxy e

Mxy =

∫ 2π

0

∫ 2

0

∫ r2

0

zdzrdrdθ =

∫ 2π

0

∫ 2

0

[z2

2

]r2

0

rdrdθ =

∫ 2π

0

∫ 2

0

r5

2drdθ

∫ 2π

0

[r6

12

]2

0

dθ = =

∫ 2π

0

16

3dθ = =

32π

3.

O valor de M e

M =

∫ 2π

0

∫ 2

0

∫ r2

0

dzrdrdθ =

∫ 2π

0

∫ 2

0

[z]r2

0 rdrdθ =

∫ 2π

0

∫ 2

0

r3drdθ

∫ 2π

0

[r4

4

]2

0

dθ =

∫ 2π

0

4dθ = 8π.

Portanto ,

z =Mxy

M=

32π

3

1

8π=

4

3

e o centroide e (0, 0, 43 ) . Observe que o centroide esta fora do solido.

31

26. Encontre uma solucao em coodenadas esfericas para a esfera x2+y2+(z−1)2 = 1

Solucao . Usamos as equacoes (3) para substituir x, yez :

x2 + y2 + (z − 1)2 = 1.

p2 sen2 φ cos2 θ + p2 sen2 φ sen2θ + (p cos φ − 1)2 = 1

φ2 sen2 φ (cos2 θ + sen2 θ ) + φ2 cos2 φ − 2ρ cos φ + 1 = 1

ρ2 (sen2 φ + cos2 φ) = 2ρ cos φ

ρ2 = 2ρ cos φ

ρ = 2 cos φ

32

27. Esboce a regiao , expresse a area com integral dupla iterada e a parabola

x = y − y2 e a reta y = x + 2

Solucao .

Figura 8: Grafico.

∫ 1

−2

∫ −y2

y−2

dxdy =

∫ 1

−2

−y2 − y + 2dy

= −y3

3− y2

2+ 2y

∣∣∣∣

1

−2

= −1

3− 1

2+ 2 −

(8

3− 2 − 4

)

=9

2

33

28. Esboce a regiao , expresse a area com integral dupla iterada e a parabola

x = y − y2 e a reta y = −x

Solucao .

Figura 9: Grafico.

∫ 2

0

∫ y−y2

−y

dxdy =

∫ 2

0

2y − y2dy

= y2 − y3

3

∣∣∣∣

2

0

= 4 − 8

3=

4

3

34

29. Esboce a regiao , expresse a area com integral dupla iterada e a curva y = ex

e as retas y = 0 x = 0 x = lnz

Solucao .

Figura 10: Grafico.

∫ lnz

0

∫ ex

0

dydx

=

∫ lnz

0

exdx

= ex

∣∣∣∣

lnz

0

= 2 − 1 = 1

35

30. Esboce a regiao , expresse a area com integral dupla iterada e as curvas

y = lnx e y = 2lnx e a reta x = e , no primeiro quadrante

Solucao .

Figura 11: Grafico.

∫ e

1

∫ 2lnx

lnx

dydx

=

1

lnx dx

= x lnx − x

∣∣∣∣

e

1

= (e − 2) − (0 − 1) = 1

36

31. Esboce a regiao de integracao e calcule a integral∫ π

0

∫ x

0xsenydydx.

Solucao . Seja B o triangulo 0 ≤ x ≤ π, 0 ≤ y ≤ x para cada X fixo em [0, π]

Assim,∫ π

0

xsenydydx →∫ π

0

[(−xcosy)]x0 dx

Segue que∫ π

0

−xcosx − (−x)dx →∫ π

0

−xcosx + xdx

∫ π

0

x − cosXdx

Fazendo integral por partes

u = x dv = cosx

du = dx v = senx

Dessa forma,

xsen −∫

senxdx → xsenx − (−cosx) → xsenx + cosx

Voltando para a integral

x2

2− (xsenx + cosx)

∣∣∣∣

π

0

toπ2

2− (πsenπ + cosπ) − (0 + 1)

=π2

2− [(0 − 1) − 1] → π2

2+ 2

∫ π

0

∫ x

0

xsenydydx =π2

2+ 2

37

32. Esboce a regiao de integracao e calcule a integral

∫ 3

0

∫ 2

0

(4 − y2)dydx

Solucao . Pelo teorema de Fubini :∫

R

∫F (x, y)dxdy onde f(x, y) = 4 − y2 e R o

retangulo 0 ≤ x ≤ 3 , 0 ≤ y ≤ 2 onde F (x, y) = 4 − y2 e R o retangulo 0 ≤ x ≤ 3 ,

0 ≤ y ≤ 2 .

Seja f(x, y) definida em R e dada por :

Assim,∫ 3

0

∫ 2

0

(4 − y2)dydx →∫ 3

0

(

4y − y3

3

)2

0

dx

Segue que∫ 3

0

8 − 8

3dx →

∫ 3

0

16

3dx

=16

3x

∣∣∣∣

3

0

→ 48

3= 16

Ou seja∫ 3

0

∫ 2

0

(4 − y2) dydx = 16

38

33. Calcule a integral da funcao f(x, y) = x2 + y2 sobre a regiao triangular com

vertices (0, 0), (1, 0), e (0, 1)

Solucao . Seja B o triangulo 0 ≤ x ≤ 1, 0 ≤ y ≤ 1 − x

B

x2 + y2dydx →∫ 1

0

∫ 1−x

0

x2 + y2 dydx

∫ 1

0

[(

yx2 +y3

3

)]1−x

0

dx →∫ 1

0

(

(1 − x)x2 +(1 − x3)3

3

)

dx

Resulta em:∫ 1

0

x2 − x3 +(1 − x)3

3dx

Faz integral separada

Primeira integral∫ 1

0

x2 dx =x3

3

∣∣∣∣

1

0

=1

3

Segunda integral∫ 1

0

−x3 dx =−x4

4

∣∣∣∣

1

0

=−1

4

Terceira integral∫ 1

0

(1 − x)3

3dx

Resolver por substituicao , chama (1 − x) de u e deriva em relacao a u

Temos :

u = 1 − x

du = −dx

Assim,∫ 1

0

−u3

3du → −1

3

∫ 1

0

u3 du = − 11

3

(u4

4

)

Volta para variavel X

= −1

3

((1 − x)4

4

)

→ (1 − x)4

12

∣∣∣∣

1

0

=1

12

Segue que∫ 1

0

x2 − x3 +(1 − x)3

3dx =

1

3− 1

4+

1

12=

1

6

Portanto∫ 1

0

∫ 1−x

0

x2 + y2 dydx =1

6

39

34. Calcule∫

D

(x + y) da

, onde D e limitada por

y =√

x , y = x2

Solucao . Podemos escrever que a regiao D e :

D = {(x, y)/ 0 ≤ x ≤ 1 , x2 ≤ y ≤√

x }

Como a fronteira de baixo e y = x2 e a de cima e y =√

x , escreve-se a integral como:

D

(x + y) dydx =

∫ 1

0

∫ √x

x2

(x + y) dydx

Segue que

=

∫ 1

0

[

xy +y2

2

]√

x

x2

dx

=

∫ 1

0

(

x3/2 +1

2x − x3 − x4

2

)

dx

=

[2

5x5/2 +

1

4x2 − 1

4x4 − 1

10x5

]1

0

=3

10

Portanto∫ 1

0

∫ √x

x2

(x + y) dydx =3

10

40

35. Calcule∫

D

y3dA,

D e a regiao triangular com vertices (0, 2); (1, 1) e (3, 2)

Solucao . A regiao D e escrita como :

D = {(x, y)/ 1 ≤ y ≤ , 2 − y ≤ x ≤ 2y − 1}

Logo,∫

D

y3 dxdy =

∫ 2

1

∫ 2y−1

2−y

y3 dxdy

=

∫ 2

1

[xy3]x=2y−1

x=2−ydy

Resulta em

=

∫ 2

1

[(2y − 1) − (2 − y)] y3 dy

=

∫ 2

1

(3y4 − 3y3) dy

=

[3

5y5 − 3

4y4

]2

1

=96

5− 12 − 3

5+

3

5→ 147

20

Portanto∫ 2

1

∫ 2y−1

2−y

y3 dxdy =147

20

41

36. Calcule∫

D

(2y − y) DA,

onde D e limitada pelo cırculo de centro na origem e raio 2

Solucao . A regiao D e :

D = {(x, y) / − 2 ≤ x ≤ 2, −√

4 − x2 ≤ y ≤√

4 − x2}

logo ,∫

D

(2x − y) dA =

∫ 2

−2

∫ √4−x2

−√

4−x2

(2x − y) dydx

=

∫ 2

−2

[

2xy − 1

2y2

]√

4−x2

√4−x2

dx

Resulta em

∫ 2

−2

[

2x√

4 − x2 − 1

2(4 − x2) + 2x

4 − x2 +1

2(4 − x2)

]

dx

=

∫ 2

−2

4x√

4 − x2dx

= −4

3(4 − x2)3/2

∣∣∣∣

2

−2

= 0

Portanto∫ 2

−2

∫ √4−x2

−√

4−x2

(2x − y) dydx = 0

42

37. A parte da esfera x2 + y2 + z2 = a2 que esta dentro do cilindro x2 + y2 = ax e

acima do plano XY

Solucao . Sendo

Z =√

a2 − x2 − y2, Zx = −x(a2 + x2 + y2)−1/2, Zy = −y(a2 − x2 − y2)−1/2

Usando

A(s) =

D

∫√

[fx(x, y)]2 + [fy(x, y)]2 + 1 dA z = f(x, y), (x, y) ∈ D

Dessa forma,

A(s) =

D

∫√

x2 + y2

a2 − x2 − y2+ 1 dA

Passa para coordenadas polares:

=

∫ π/2

−π2

∫ acosθ

a

r2

a2 − r2+ 1 rdrdθ

=

∫ π2

−π2

∫ a cosθ

0

ar√a2 − r2

drdθ

Logo,

∫ π2

−π2

[

−a√

a2 − a2cos2θ − a

]

dθ = 2a2

∫ π2

0

(

1 −√

1 − cos2 θ)

2a2

∫ π2

0

dθ − 2a2

∫ π2

0

√sen2θdθ

= −2a2

∫ π2

0

senθdθ

= a2(π − 2)

43

38. Calcule a integral tripla∫ ∫ ∫

EXZ DV , onde E e o solido do tetraedro com

vertices (0, 0, 0) (0, 1, 0) (1, 1, 0) e (0, 1, 1)

Solucao . A regiao de integracao e

0 ≤ x ≤ y − z

0 ≤ y ≤ 1

0 ≤ z ≤ y

Entao temos:∫ 1

0

∫ y

0

∫ y−z

0

XZ dxdzdy

=

∫ 1

0

∫ y

0

1

2(y − z)2 Z dzdy → 1

2

∫ 1

0

[1

2y2z2 − 2

3yz3 +

1

4z4

]y

0

dy

Resulta em :1

24

∫ 1

0

y4 dy =1

24

[1

5y5

]1

0

=1

120

44

39. Calcule a integral tripla∫ ∫ ∫

E

(x + 2y) dV,

onde E e limitado pelo cilindro parabolico y = x2 e pelo planos x = z , x =

y e z = 0

Solucao . O intevalo de integracao e :

0 ≤ x ≤ 1

X ≤ y ≤ x2

0 ≤ z ≤ x

Entao temos:

∫ ∫ ∫

E

(x + 2y)dV →∫ 1

0

∫ x

x2

∫ x

0

(x + 2y) dzdydx

=

∫ 1

0

∫ x

x2

(x2 + 2yx) dydx →∫ 1

0

[x2y + xy2

]y=x

y=x2 dX

Resulta em :

∫ 1

0

(2x3 − x4 − x5)dX =

[1

2x4 − 1

5x5 − 1

6x6

]1

0

=2

15

45

40. Faca o esboco do solido cujo volume e dada pela integral e calcule-a

∫ 2π

0

∫ 2

0

∫ 4−r2

0

rdzdrdθ

Solucao . Essa integral iterada e uma integral tripla sobre a regiao solida

E = {(r, θ, z) 0 ≤ θ ≤ 2π, 0 ≤ r ≤ 2, , 0 ≤ z ≤ 4 − r2}

e a projecao de E acima do plano XY e a paraboloide z = 4 − r2 = 4 − x2 − y2

Utilizando coordenadas cilindricas

∫ 2π

0

∫ 2

0

∫ 4−r2

0

rdzdrdθ =

∫ 2π

0

∫ 2

0

(4r − r3) drdθ

=

∫ 2π

0

[

2r2 − 1

4r4

]r=2

r=0

=

∫ 2π

0

(8 − 4)dθ = 4θ

∣∣∣∣

0

= 8π

Logo∫ 2π

0

∫ 2

0

∫ 4−r2

0

rdzdrdθ = 8π

46

41. Faca o esboco do solido cujo volume e dado pela integral e calcule-a

∫ π2

0

∫ π2

0

∫ 1

0

ρ2sinφdρdφ

Solucao . A regiao de integracao usando coordenadas esfericas

E = {(ρ, θ, φ) / 0 ≤ ρ ≤ 1, 0 ≤ θ ≤ π

2, 0 ≤ φ ≤ π

2}

Sabemos que ρ = x2 + y2 + z2 = 1

Entao temos:

∫ π2

0

∫ π2

0

∫ 1

0

ρ2sen φ dρdθdφ =

∫ π2

0

∫ π2

0

[1

3ρ3sen φ

]p=1

p=0

dθdφ

=

∫ π2

0

∫ π2

0

1

3sen φ dθdφ =

∫ π2

0

1

3sen φ [φ]

θ= π2

θ=0 dφ

Logo,

=1

3

∫ π2

0

π

2sin φdθ =

π

6[−cos]

π20 =

π

6

47

42. Faca o esboco do solido cujo colume e dado pela integral e calcule-a

∫ π3

0

∫ 2π

0

∫ secφ

0

ρ2sen φ dθdφ

Solucao . A regiao de integracao usando coordenadas esfericas

E = {(ρ, θ, φ) / 0leθ ≤ 2π , 0 ≤ φ ≤ π

3, 0 ≤ ρ ≤ sen φ}

Sendo ρ = sec φ equivalente a ρ cosφ = z = 1 A regiao solida e esta limitada entre cone

φ = π3 e o plano z = 1 Portanto temos:

∫ π3

0

∫ 2π

0

∫ secφ

0

ρ2sen φ dρdθdφ =

∫ π3

0

∫ 2π

0

[1

3ρ3senφ

]ρ=secφ

ρ=0

dθdφ

Resulta :

=1

3

∫ π3

0

∫ 2π

0

senφ

cos3φdθdφ =

3

∫ π3

0

(tan φ sec2 φ) dφ

=2π

3

[tan2φ

2

]π3

0

= π

48

43. Calcule∫

R

∫ √

x2 + y2 dxdy sendo R a regiao limitada por x2 + y2 = 2x ,

x2 + y2 = 4x , y = x e y =√

33 x

Solucao . A regiao R:

1) x2 + y2 = 2x

x2 − 2x + y2 = 0

(x − 1)2 + y2 = 1

2)x2 + y2 = 4x

x2 − 4x + y2 = 0

(x − 2)2 + y2 = 4

3) y = x

4)y =√

33 x

De acordo com x2 + y2 = 2x e x2 + y2 = 4x usa coodenadas cilindricas

r2 = 2rcosθ → r(r − 2 cosθ) = 0 → r = 2cosθ

r2 = 4rcosθ → r(r − 4cosθ) = 0 → r = 4cosθ Assim,

π6 ≤ θ ≤ π

4

2cosθ ≤ r ≤ 4cosθ Logo,

R

∫√

x2 + y2 dxdy →∫ π

4

π6

(

r3

3

∣∣∣∣

4 cos θ

2cosθ

)

θ

1

3

∫ π4

π6

64cos3θ − 8cos3 θ dθ

Portanto temos

56

3

∫ π4

π6

cos3θ dθ to56

3

∫ π4

π6

(1 − sen2θ) cosθ dθ

Fazendo a substituicao : chama senθ de U e deriva em relacao a U, temos :

u = senθ

du = cosθdθ

56

3

∫ π4

π6

(1 − u2) du

49

Assim,

56

3

(

u − u3

3

) ∣∣∣∣

π4

π6

=56

3

(

sen θ − sen3θ

3

) ∣∣∣∣

π4

π6

=

√2

2− ( 2

2 )3

3− 0

=

√2

2−

2√

28

3→

√2

2−

√2

4· 1

3

=

√2

2−

√2

12→ 6

√2 −

√2

12=

5√

2

12

50

44. Calcule: (x2 + y2) dxdy Onde B = {(x, y) ∈ IR2 / 1 ≤ x2 + y2 ≤ 4}Solucao .

1 ≤ r ≤ 2 Sendo

x = rcosθ

y = rsenθdxdy = rdrdθ

Considerando

x2 + y2 = r2

Assim,∫

B

r2r drdθ →∫ 2π

0

∫ 2

1

r3 drdθ

=

∫ 2π

0

[

r4

4

∣∣∣∣

2

1

]

dθ →∫ 2π

0

(

4 − 1

4

)

15

4

∫ 2π

0

dθ → θ

∣∣∣∣

0

→ 15

4· 2π =

15

51

45. Calcule a integral abaixo :

∫ 1

0

∫ 4

4x

e−y2

dydx

Solucao . Nesse caso nao e possıvel calcular a integral, pois f(y) = e−y2

nao possui

primitiva, entao esboca-se a area A regiao passa a ser

R : 0 ≤ x ≤ y

4

0 ≤ y ≤ 4

Assim,

∫ 4

0

∫ y4

0

e−y2

dxdy →∫ y

4

0

e−y2

dx → e−y2

x

∣∣∣∣

y4

0

= e−y2 · y

4

=

∫ 4

0

(e−y2 y

4)dy → 1

4

∫ 4

0

(e−y2 · y) dy

Fazendo por substituicao

u = −y2

du = −2ydy → −fdu2 = ydy

Entao temos1

4

∫ 4

0

eu

(

−du

2

)

= −1

8

∫ 4

0

eudu

=1

8

∫ 0

4

eu du → 1

8(eu)

∣∣∣∣

0

4

=1

8

(

e−y2)∣∣∣∣

0

4

=1

8

(eo − e−16

)→ 1

8(1 − e−16)

52

46. Calcular∫ ∫ ∫

T

(x2 + y2)dV,

onde T e a regiao inferior ao cilindro x2 + y2 = 1 e a esfera x2 + y2 + z2 = 4

Solucao .

*Em baixo z = −√

4 − x2 − y2

*Em cima z =√

4 − x2 − y2

Dessa forma

-√

4 − x2 − y2 ≤ z ≤√

4 − x2 − y2

x2 + y2 = 1

Usando coordenadas cilindrica

x = rcosθ

y = rsenθ

z = z

achar o Jacobiano:∣∣∣∣

σ(x, y, z)

σ(r, θ, z)

∣∣∣∣= r

A regiao de integracao e :

R′

-√

4 − r2cos2θ − r2sen2θ ≤ z ≤√

4 − r2cos2θ − r2sen2θ

0 ≤ θ ≤ 2π

0 ≤ r ≤ 1

Logo∫ ∫ ∫

R′

(r2cos2θ + r2sen2θ)r dzdrdθ

∫ 2π

0

∫ 1

0

∫ √4−r2

−√

4−r2

r3 dzdrdθ =

∫ 2π

0

∫ 1

0

r3(√

4 − r2 +√

4 − r2)

drdθ

=

∫ 2π

0

∫ 1

0

r3(2√

4 − r2) drdθ = 2

∫ 2π

0

∫ 1

0

r · r2(√

4 − r2) drdθ

Fazendo

u = 4 − r2 → r2 = 4 − u

−2rdr = du → rdr =−du

2

Portanto

Para r = 0 → u = 4

53

Para r = 1 → u = 3

Entao temos

2

∫ 2π

0

∫ −3

4

(4 − u)√

u − du

2dθ →

∫ 2π

0

∫ 4

3

(4 − u)√

u dudθ

=

∫ 2π

0

∫ 4

3

(4 − u) · u1/2 dudθ

=

∫ 2π

0

∫ 4

3

4u1/2 − u3/2dudθ →∫ 2π

0

4u3/2

3/2− u5/2

5/2

∣∣∣∣

4

3

=

∫ 2π

0

8

3u3/2 − 2

5u5/2

∣∣∣∣

4

3

=

∫ 2π

0

(8

3(8) − 2

5(32) − 8

3(3√

3) +2

5(9√

2)

)

=

∫ 2π

0

(64

3− 64

5− 8

√3 +

18

5

√3

)

=

(64

3− 64

5− 8

√3 +

18

5

√3

)

=

(

256

15− 44

√3

5

)

π

54

47. Calcule∫ ∫ ∫

BZdxdydz onde B e o conjunto x2 + y2 + z2 ≤ 1 e z ≥

x2 + y2

Solucao . Usando coordenadas esfericas.

X = rcosθsenσ

Y = rsenθsemσ

Z = rcosσ

Z =√

x2 + y2

rcosσ =√

r2cos2sen2σ + r2sen2θsenσ

rcosσ =√

r2sen2σ(cos2θ + sen2θ)

rcosσ = rsenσ

cosσ = senσ → senσ

cosσ= 1 → tgσ = 1

σ =π

4(Cone que passa na origem )

Entao temos∫ 2π

0

∫ π4

0

∫ 1

0

rcosσ (r2 senσ) drdσdθ

=

∫ 2π

0

∫ π4

0

∫ 1

0

r3 cosσ sen σ drdydθ

=

∫ 2π

0

∫ π4

0

r4

4

∣∣∣∣

1

0

cosσ senσ dσ dtheta

=

∫ 2π

0

∫ π4

0

1

4cosσsenσ dσdθ

Sabemos que cos σsenσ = sen 2σ2

=

∫ 2π

0

∫ π4

0

1

4

sen2σ

2dσdθ → 1

4

∫ 2π

0

(−cos 2σ)

4

∣∣∣∣

π4

0

=1

16(0 − (−1))

∫ 2π

0

dθ =1

6(2π) =

π

8

55

48. Calcule o volume do solido compreendido entre o cone e a superfıcie delimi-

tados pelas equacoes a seguir:

x2 + y2 + z2 = 2z

x2 + y2 + z2 − 2z = 0

x2 + y2 + z2 − 2z + 1 = 1

(x2 + y2) + (z − 1)2 = 1

Solucao . Achando a variacao de raio R varia de 0 ate a esfera x2 + y2 + z2 = 2z

r2sen2σcos2θ + r2sen2σsen2θ + r2cos2σ = 2rcosσ

r = 2cosσ

A regiao de integracao e :

0 ≤ r ≤ 2cosσ

0 ≤ σ ≤ π4

0 ≤ θ ≤ 2π

Volume e

∫ ∫ ∫

dxdydz =

∫ ∫ ∫ ′

R

r2senσ drdσdθ =

∫ 2π

0

∫ π4

0

∫ 2cosσ

0

r2senσ drdσdθ

∫ 2π

0

∫ π4

0

r3

3

∣∣∣∣

2cosσ

0

senσ dσdθ =

∫ 2π

0

∫ π4

0

8

3cos3σsenσ dσdθ

∫ 2π

0

∫√

22

1

8

3u3 dudθ =

8

3

∫ 2π

0

∫ 1

√2

2

u3dudθ

=8

3

∫ 2π

0

u4

4

∣∣∣∣

1

√2

2

dθ =8

3

∣∣∣∣

0

(1

4−

416

4

)

8

3

∫ 2π

0

(1

4− 1

16

)

dθ → 8

3

∫ 2π

0

(3

16

)

dθ =1

2

∫ 2π

0

=2π

2= π

56

49. Encontre o volume do elipsoide x2

a2 + y2

b2 + z2

c2 ≤ 1

Solucao .

V =

∫ ∫ ∫

R

dxdydz

Transformar elipsoide em uma esfera por mudanca de variavel

Logo,∫ 2π

0

∫ π

0

∫ 1

0

abcr2senσ drdσdθ

∫ 2π

0

∫ π

0

[

r3

3abc senσ

∣∣∣∣

1

0

]

dσdθ

= abc

∫ 2π

0

∫ π

0

[1

3senσ

]

dσdθ

=abc

3

∫ 2π

0

[

−cosσ

∣∣∣∣

π

0

]

=abc

3

∫ 2π

0

(1 − (−1))dθ → abc

3

[

∣∣∣∣

0

]

=abc

3[4π]

=4πabc

3

57

50. Calcule∫

R

∫ √y−x

1+y+x dxdy onde R e o triangulo de vertices (0, 0) (1, 0) (0, 1)

Solucao .

u = y − x

v = 1 + y + x

u + v = 1 + 2y

2y = u + v − 1

y =u + v − 1

2

u = y − x

u =u + v − 1

2− x

2u =u + v − 1 − 2x

2

x =−u + v − 1

2

Encontrar o Jacobiano

∂(x, y)

∂(u, v)= A =

∂x∂u

∂x∂V

∂y∂u

∂y∂V

= A =

−12

12

12

12

=1

2

Substituindo x e y em u = y − x e v = 1 + y + x, encontra-se novos pontos .

(0, 0) → (0, 0)(1, 0) → (−1, 2)(0, 1) → (1, 2)

Logo,∫ ∫ √

u

v

1

2dvdu →

∫ 1

−1

∫ 2

1

u1/3

v

1

2dvdu

=1

2

∫ 1

−1

∫ 2

1

u1/3

vdvdu → 1

2

∫ 1

−1

[

lnv · u1/3

∣∣∣∣

2

1

]

du

=1

2

∫ 1

−1

[

u1/3ln2 − ln1]

du → 1

2

[

u4/3

4/3ln2

∣∣∣∣

1

−1

]

=3

8

[

u4/3 · ln2]1

−1→ 3

8

[

14/3 · ln2 − (−14/3)ln2]

=3

8[ln2 − ln2] → 3

8(0) = 0

58

51. Calcular a massa e o centro de massa da regiao D : {(x, y) : −1 6 x 6 1, 0 6 y 6 1}e a densidade ρ(x, y) = x2

Solucao . Primeiramente vamos calcular a massa M;

M =

∫ ∫

x2dA =

∫ 1

0

∫ 1

−1

x2dxdy =

∫ 1

0

x3

3|10dy =

∫ 1

0

2

3dy =

2

3y|10

Logo; M = 23

Calculemos agora o centro de massa.

X =1

M

D

xx2dxdy =2

3

∫ 1

0

∫ 1

−1

x3dxdy =

∫ 1

0

x4

4|1−1dy = 0

Y =1

M

D

yx2dxdy =2

3

∫ 1

0

∫ 1

−1

yx3dxdy =

∫ 1

0

yx3

3|1−1dy =

∫ 1

0

ydy =y2

2=

1

2

Portanto o centro de massa de D :

(0,1

2)

E sua massa:

M =2

3

59

52. Calcular∫

R

∫xy e

xy dydx onde R = [0,1] X [1,2]

Solucao . Inicialmente temos;

R

∫x

ye

xy =

∫ 1

0

∫ 2

1

x

ye

xy dydx =

∫ 1

0

exy |21dx =

∫ 1

0

−ex2 + exdx = −2e

x2 + ex|10

= (−2e12 + e) − (−2 + 1) = −2e

12 + e + 1

Logo:∫

R

∫x

ye

xy dydx = −2e

12 + e + 1

60

53. Calcular a area do grafico entre a funcao seno e cosseno. sin[0, 2Π] e cos[0, 2Π]

Solucao . A area que procuramos esta compreendida entre os ponto em que o seno igual

ao coseno. Vamos proccurar esses pontos. sinx = cosx se,somente se, x = x4 ou x = xΠ

4

A =

∫ 5Π4

x4

∫ sin

cos

dydx =

∫ 5Π4

x4

(sinx−cosx)dx = [−cosx−sinx]|5Π4

x4

= (

√2

2+

√2

2)−(−

√2

2−√

2

2) =

2√

2

2

Portanto a area procurada e

A =2√

2

2

61

54. Calcular a area da regiao entre y =√

x e o eixo x. [0,4]

Solucao . Sabemos que: y =√

x −→ y2 = x, logo temos duas opcoes para encontrar

essa area.

1 - A =∫ 4

0

∫√x

0dydx

ou

2 - A =∫ 2

0

∫ 4

y2 dydx

Resolveremos pela primeira opcao.

A =

∫ 4

0

∫ √x

0

dydx =

∫ 4

0

y|√

x0 dx =

∫ 4

0

√xdx =

∫ 4

0

x12 dx =

x32

32

|40 =16

4

Portanto a area da procurada eh

A =16

4

62

55. Calcular o volume do solido entre o plano x+2y+z = 2 e os eixos coordenados.

Solucao .

Para z = 0, temos que y = 2−x2

Para y = 0, temos que z = 2 − x

A funcao que precisamos para calcular esse volume, encontramos a partir da area do

triangulo retangulo formado pelo grafico. Logo, A(x) = 12

(2−x)2 (2 − x) = (2−x)2

4 . Pronto

agora podemos calcular o volume.

V =

∫ 2

0

A(x)dx =

∫ 2

0

(2 − x)2

4dx =

−(2 − x)3

12|20 =

8

12=

2

3

Portanto o volume do solido eh:

V =2

3

63

56. Calcular o volume de f(x, y) = x2+y, sobre a regiao R = {(x,y), 1 6 x2+y2 6 5}.

Solucao .

Temos f(x, y) = x2 + y, sobre a regiao R = {(x,y), 1 6 x2 + y2 6 5}, vamos usar uma

mudanca de coordenada para resolver esse problema. Utilizaremos coordenadas polares.

A regiao R em polares fica assim: S = {(r,Θ) , 1 6 r 6√

5 e 0 6 Θ 6 2Π}

f(x, y) −→ f(rcosΘ, rsinΘ) = r2cos2Θ + rsinΘ

Diante disso temos:

V =

∫ 2Π

0

∫ √5

1

(r2cosΘ + rsinΘ)rdrdΘ =

∫ 2Π

0

∫ √5

1

(r3cosΘ + r2sinΘ)rdrdΘ

V =

∫ 2Π

0

[r4

4cos2Θ+

r3

3sinΘ]|

√5

1 dΘ =

∫ 2Π

0

(25

4cos2Θ+

5√

5

3cosΘ− 1

4cos2Θ− 1

3sinΘ)dΘ

V =

∫ 2Π

0

(6cos2Θ +5√

5

3− 1sinΘ)dΘ = [(6

Θ

2+

sin2Θ

4) − 5

√5

3− 1cosΘ]|2Π0 = 6Π

Logo, o volume procurado e:

V = 6Π

64

57. Calcular o volume limitado acima pelo hemisferio z =√

16 − x2 − y2 sobre o

disco x2 + y2 = 4.

Solucao .

Novamente devemos usar coordenadas polares.

R : {(r,Θ)} ; 0 6 r 6 2 e 0 6 Θ 6 2Π

f(rcosΘ, rsinΘ) =√

16 − r2

V =∫ 2Π

0

∫ 2

0

√16 − r2drdΘ

Primeiramente vamos calcular o valor da integral∫ 2

0

√16 − r2dr

Essa integral e resolvida por subistituicao:

u = 16 − r2 e du = −2rdr

Assim;

∫ 2

0

16 − r2dr = −1

2

∫ 12

16

√udu = −1

2

u32

32

|1216 =

√123

3+

43

3=

64

3− 24

√3

3

Pronto agora podemos calcular a segunda integral.

∫ 2Π

0

64

3− 24

√3

3dΘ =

64 − 24√

3

32Π

Portanto encontramos o volume procurado:

V =64 − 24

√3

32Π

65

58. Calcular a massa da lamina que e 14 do cırculo unitario, sabendo que a densi-

dade ρ(x, y) e diretamente proporcional a distancia d(ρ, (0, 0)).

Solucao .

Como sabemos que a densidade e proporcional a distancia, temos:

ρ = kd(ρ, (0, 0)) = k√

x2 + y2 −→ ρ = kr

Isso, pois de acordo com as coordenadas polares x2 + y2 = r2, assim√

r2 = r

Fazendo a mudanca de coordenadas: R: { (x,y) ; x2 + y2 6 1 e x > 0 e y > 0} −→ R: {(r,Θ) ; 0 6 r 6 1 e 0 6 Θ 6

Π2 }

M =

∫ Π2

0

∫ 1

0

krdrdΘ =

∫ Π2

0

kr2|10dΘ =

∫ Π2

0

k

3dΘ =

k

3Θ|

Π20 =

6

Logo, a massa e igual a

M =kΠ

6

66

59. Calcular area da regiao contida no plano z = 2 − x − y, que cobre 14 do cırulo

unitario.

Solucao .

Inicialmente vamos calcular as derivadas parciais da funcao:

f(x, y) = 2 − x − y ; fx = −1 ; fy = −1

Vale salientar que no cırculo unitario a variacao e de 0 −→ 1

S =

R

∫ √

1 + f2x + f2

y dA =

∫ 1

0

∫ √1−x2

0

√3dydx =

√3

R

dydx =

√3Π

4

Podemos tambm usar coordenadas polares para resolucao deste problema. Em polares

fica assim:

S =√

3

∫ Π2

0

∫ 1

0

rdrdΘ =√

3

∫ Π2

0

1

2dΘ =

√3Π

4

Logo, a area da regiaao e

S =

√3Π

4

67

60. Calcular a area do paraboloide z = 1 + x2 + y2, sobre a regiao delimitada pelo

cırculo x2 + y2 = 4

Solucao .

Calculando as derivadas parciais da funcao temos:

f(x, y) = 1 + x2 + y2 ; fx = 2x ; fy = 2y

Usando a formula para calcularmos a area:

S =

R

∫ √

1 + f2x + f2

y dA =

∫ 2Π

0

∫ 2

0

1 + 4r2rdrdΘ

Aplicando uma substituicao temos:

u = 1 + 42

du = 8rdr

Logo;∫ 2Π

0

1

8

∫ 17

1

u12 dudΘ =

1

8

∫ 2Π

0

2

3u

32 |171 dΘ =

1

6(17

√17 − 1)Π

Portanto, a area e

S =1

6(17

√17 − 1)Π

68

61. Calcular a massa do solido Q, que esta entre o elipsoide 4x2 + 4y2 + z2 = 16 e o

plano xy, sabendo que a densidade no ponto (x,y,z) e proporcional a distacia

a xy.

Solucao .

Para z = 0 temos: 4x2 + 4y2 = 16 —– x2 + y2 = 4

Temos, ρ(x, y, z) = kz, pois a distancia proporcional.

z2 = 16 − 4x2 − 4y2 Em polares 0 6 z 6√

16 − 4r2

M =

∫ ∫

Q

ρ(x, y, z)dv =

∫ 2Π

0

∫ 2

0

∫ √16−4r2

0

kzrdzdrdΘ =

∫ 2Π

0

∫ 2

0

krz2

2|√

16−4r2

0 drdΘ

∫ 2Π

0

∫ 2

0

k

2(16 − 4r3)drdΘ =

k

2

∫ 2Π

0

8r2 − r4|20dΘ =k

22Π(32 − 16) = 16kΠ

Logo, a massa do solido Q e

M = 16kΠ

69

62. Calcular o volume do solido limitado acima e abaixo pela esfera x2 +y2 +z2 = 4

e pelo cilindro x2 + y2 − 2y = 0.

Solucao .

De coordenadas polares temos: x2 + y2 = r2

r2 = 2y = 2rsinΘ −→ r = 2sinΘ −→ 0 6 r 6 2sinΘ

z2 = 4 − (x2 + y2) −→ z2 = 4 − r2 −→ −√

4 − r2 6 z 6√

4 − r2

V =

∫ ∫

Q

1dv =

∫ Π

0

∫ 2sinΘ

0

∫ √4−r2

−√

4−r2

1rdzdrdΘ =

∫ Π

0

∫ 2sinΘ

0

rz|√

4−r2

−√

4−r2drdΘ

V =

∫ Π

0

∫ 2sinΘ

0

2√

4 − r2rdrdΘ

Usaremos uma substituicao para resolver a segunda integral.

u = 2 − r2 −→ du = −2dr

Logo;∫ 2sinΘ

0

2√

4 − r2rdr =

∫ 2sinΘ

0

−√

udu = −3

2

∫ 2sinΘ

0

(4 − r32 )

Agora que ja temos como resolver a segunda integral, voltamos a integracao original;

−3

2

∫ Π

0

∫ 2sinΘ

0

(4−r32 )drdΘ = −3

2

∫ Π

0

(4−r32 )|2sinΘ

0 dΘ = −3

2

∫ Π

0

[(4−4sin2Θ)32 −4

32 ]dΘ

= −3

2

∫ Π

0

[(8cos2Θ)32 − 8]dΘ =

16

3

∫ Π

0

(1 − cos3Θ)dΘ =16

3

∫ Π

0

[(1 − cos2Θ)cosΘ]dΘ =

16

3

∫ Π

0

[Θ −∫ Π

0

(1 − sin2Θ)cosΘ]dΘ =16

3[Θ −

∫ Π

0

cos2ΘdΘ +

∫ Π

0

sin2ΘcosΘdΘ] =

=16

3[Θ − sinΘ +

sin3Θ

3]|Π0 =

16

3[Π − sinΠ +

sin3Π

3] =

16Π

3

Portanto, o volume almeijado e:

V =16Π

3

70

63. Calcular o volume do solido limitado acima esfera x2+y2+z2 = 9 e lateralmente

pelo cone z =√

x2 + y2.

Solucao .

Temos que a equacao da esfera e: x2 + y2 + z2 = 9 = ρ2 −→ ρ2 = 9 −→ ρ = 3

Calculamos a intersecao do cone com a esfera: x2 + y2 + z2 = 9 e z =√

x2 + y2 logo;

z = x2 + y2 −→ 2z2 = 9 −→ z = 3√2

Como z = ρcosφ e z = 3√2

, temos 3√2

= 3cosφ

Assim podemos conluir que cosφ =√

22 −→ φ = Π

4

V =

∫ ∫

Q

1dv =

∫ 2Π

0

∫ Π4

0

∫ 3

0

ρ2sinφdρdφdΘ =

∫ 2Π

0

∫ Π4

0

9sinφdφdΘ =

= −9

∫ 2Π

0

cosΘ|Π40 − 9(

√2

2− 1)2Π = 9Π(2 −

√2)

Portanto, o do solido e:

V = 9Π(2 −√

2)

71

64. Calcular o centro de massa do solido Q que esta compreendido entre o parabololide

z = x2 + y2 e o plano z = 4, sabendo que a densidade e uniforme.

Solucao .

Temos o seguinte: (x, y, z) = (0, 0, z) e ρ(x, y, z) = k

M =

∫ ∫

Q

ρ(x, y, z)dv =

∫ 2Π

0

∫ Π2

0

∫ √20

0

kρ2sinφdρdφdΘ =

=

∫ 2Π

0

∫ Π2

0

ksinφ(ρ3

3)|√

200 dφdΘ =

∫ 2Π

0

∫ Π2

0

k20√

20

3sinφdφdΘ =

∫ 2Π

0

−k20√

20

3(cosφ)|

Π20 dΘ =

=

∫ 2Π

0

k20√

20

3dΘ = k

20√

20

3(Θ)|2Π0 = [

40Π√

20

3]k

Pronto encontramos a massa

M = [40Π

√20

3]k

Vamos agora calcular o centro de massa.

Mxy =

∫ 2Π

0

∫ Π2

0

∫ √20

0

(ρcosφ)

zkρ2sinφdρdφdΘ =

(20)2

4

sin2φ

22Π = 100

1

22Π = 100kΠ

Logo, o centro de massa do solido Q e

(x, y, z) = (0, 0,300

40√

20)

72

65. Calcular∫

S

∫yda, da regiao S delimitada a baixo pelas parabolas y2 = 4 − 4x e

y2 = 4 + 4x e acima pelo eixo x.

Solucao .

Vale salientar que S e a imagem inversa do retangulo R, assim usamos uma transformacao

T(u,v) = (x,y), tal que x = u2 − v2 e y = 2uv

∂x∂u = 2u e ∂x

∂v = −2v

∂y∂u = 2v e ∂y

∂v = −2u

Calculando o jacobiano temos,

∣∣∣∣∣∣

2u 2v

−2v 2u

∣∣∣∣∣∣

= 4u2 + 4v2

∫ 1

0

∫ 1

0

2uv(4u2+4v2)dudv =

∫ 1

0

∫ 1

0

(8u3v+8uv3)dudv = int10[2u4v+4u2v3]|10dv = v2+v2|10 = 2

Logo;

∫ 1

0

∫ 1

0

2uv(4u2 + 4v2)dudv = 2

73

66. Calcular o volume do conjunto de todos os pontos (x,y,z) tais que 0 6 x 6 1 e

0 6 y 6 1 e 0 6 z 6 x2 + y2.

Solucao .

Primeiramente sabemos que a regiao B e um retangulo, vejamos:

B

∫(x2 + y2)dxdy onde, B e retangulo 0 6 x 6 1 e 0 6 y 6 1

∫ 1

0

∫ 1

0

(x2 + y2dxdy) =

∫ 1

0

[x3

3+ y2]|10dy =

∫ 1

0

1

3+ y2 = [

1

3y +

y3

3]|10 =

1

3+

1

3=

2

3

Portanto, o volume em questao e:

V =2

3

74

67. Calcular o volume do conjunto de todos os pontos (x,y,z) tais que x2 + y2 6

z 6 2 − x2 − y2.

Solucao .

Primeiramente vamos determinar a intersecao dos graficos z = x2 + y2 e z = 2 − x2 − y2

z = x2 + y2 = 2 − x2 − y2 ⇐⇒ x2 + y2 = 1, logo a intesecao e a circunferencia de centro

(0, 0, 1) e raio 1.

Desta forma temos:

V =

B

(1 − x2 − y2)dxdy

Passando para coordenadas polares: 1 − x2 − y2 = 1 − r2

V =

B

(1 − x2 − y2)dxdy =

∫ 2Π

0

∫ 1

0

(1 − r2)rdrdΘ =

∫ 2Π

0

∫ 1

0

r − r3drdΘ =

=

∫ 2Π

0

[

∫ 1

0

rdr −∫ 1

0

r3dr]dΘ =

∫ 2Π

0

[r2

2|10 −

r4

4|10]dΘ =

1

4Θ|2Π0 =

Π

2

Assim o volume e igual a:

V =Π

2

75

68. Inverta a ordem de integracao e calcule∫ 1

0

∫ 1√ysinx3dxdy.

Solucao .

Sabemos que x: [√

y, 1] e y: [0, 1], assim B = {(x, y)εR2/0 6 y 6 1e√

y 6 x 6 1}

Se temos√

y = x −→ y = x2

∫ 1

0

∫ 1

√y

sinx3dydx =

∫ 1

0

∫ x2

0

sinx3dydx =

∫ 1

0

sinx3

∫ x2

0

dydx =

∫ 1

0

sinx3[y]x2

0 dx =

=

∫ 1

0

x2sinx3dx = −x3

3cosx3|10 =

1

3(1 − cos1)

Logo;

∫ 1

0

∫ x2

0

sinx3dydx =1

3(1 − cos1)

76

69. Calcule∫

B

∫ cos(x−y)sin(x+y)dxdy onde B: { 1 6 x + y 6 2, x > 0 e y > 0 }.

Solucao .

Fazendo uma mudanca de variavel temos: u = x − y e v = x + y

x = u2 + v

2 e y = v2 − u

2

Calculando as derivadas parciais;

∂x∂u = 1

2 e ∂x∂v = 1

2

∂y∂u = − 1

2 e ∂y∂v = 1

2

Calculando o jacobiano temos,

∣∣∣∣∣∣

12

12

− 12

12

∣∣∣∣∣∣

=1

2

B

∫cos(x − y)

sin(x + y)dxdy =

S

∫cosu

sinv

1

2dudv =

1

2

∫ 2

1

∫ v

−v

cosu

sinvdudv =

1

2

∫ 2

1

sinu

sinv|v−vdv =

∫ 2

1

dv = 1

Portanto;

B

∫cos(x − y)

sin(x + y)dxdy = 1

77

70. Calcule∫ 1

0

∫ x

0x√

x2 + 3y2.

Solucao .

Temos que;

x : [0, 1] e y : [0, x] ; 0 6 x 6 1 ; 0 6 y 6 x

Fazendo uma mudanca de variavel temos: u = x − y e v = x + y

x = ρcosΘ e√

3y = ρsinΘ

ρ = secΘ e Θ = Π3

Assim;

x = ρcosΘ e y =√

33 ρsinΘ

Calculando as derivadas parciais;

∂x∂u = −ρsinΘ e ∂x

∂v = cosΘ

∂y∂u =

√3

3 cosΘ e ∂y∂v =

√3

3 sinΘ

Calculando o jacobiano temos,

∣∣∣∣∣∣

−ρsinΘ cosΘ√

33 cosΘ

√3

3 sinΘ

∣∣∣∣∣∣

=

√3

∫ 1

0

∫ x

0

x√

x2 + 3y2dydx =

√3

3

∫ Π3

0

∫ secΘ

0

ρ3cosΘdρdΘ =

√3

3

∫ Π3

0

ρ4

4cosΘ|secΘ

0 dΘ =

=

√3

12

∫ Π3

0

sec3ΘdΘ =

√3

12[secΘtgΘ + ln(secΘ + tgΘ)]

Π30 =

√3

12[2√

3 + ln(2 +√

3)]

Logo;

∫ 1

0

∫ x

0

x√

x2 + 3y2 =

√3

12[2√

3 + ln(2 +√

3)]

78

71. Calcule∫ Π

0

∫ x

0xsinydydx.

Solucao . Inicialmente vamos calcular a primitiva de xsiny em relacao a y.

∫ Π

0

∫ x

0

xsinydydx =

∫ Π

0

[−xcosy]x0dx =

∫ Π

0

(x − xcosx)dx =

Agora vamos calcular a primitiva (x − xcosx) em relacao a x.

= [x2

2− (cosx + sinx)]Π0 =

Π2

2+ 2

Logo;

∫ Π

0

∫ x

0

xsinydydx =Π2

2+ 2

79

72. Calcule∫ ln 8

1

∫ ln y

0ex+ydxdy.

Solucao . Como os limites de integracao e a funcao ja foram definidos na questao, vamos

apenas calcular as primitivas da funcao, primeiro em relacao a x e depois a y e aplicarmos

os limites de integracao.

Comecemos, primitiva em relacao a x:

∫ ln 8

1

∫ ln y

0

ex+ydxdy =

∫ ln 8

1

[ex+y]ln y0 dy =

=

∫ ln 8

1

yey − ey = [(y − 1)ey − ey]ln 81 = 8(ln 8 − 1) − 8 + e = 8 ln 8 − 16 + e

Portanto,

∫ ln 8

1

∫ ln y

0

ex+ydxdy = 8 ln 8 − 16 + e

80

73. Calcule o volume da regiao limitada pelo paraboloide z = x2+y2 e inferiormente

triangulo delimitado pelas retas y = x, x = 0 e x + y = 2 no plano xy.

Solucao .Calculando so limites de integracao, para y temos:

x + y = 2ey = x −→ y = 2 − x, x 6 y 6 2 − x

E para x temos que:

0 6 x 6 1

Agora vamos encontrar o volume:

V =

∫ 1

0

∫ 2−x

x

(x2 + y2)dydx =

∫ 1

0

[x2y +y3

3]2−xx dx =

∫ 1

0

[2x2 − 7x3

3+

(2 − x)3

3]dx

V = [2x3

3− 7x4

12− (2 − x)4

12]10 = (

2

3− 7

12− 1

12) − (0 − 0 − 16

12) =

4

3

Logo, o volume procurado e:

V =4

3

81

74. Calcule∫ 1

0

∫ 1

0

∫ 1

0(x2 + y2 + z2)dzdydx.

Solucao . Primeiro calculamos integral em relacao a z, depois em relacao y e por ultimo

a x, vejamos:

∫ 1

0

∫ 1

0

(x2+y2+z3

3|10)dydx =

∫ 1

0

∫ 1

0

(x2+y2+1

3)dydx =

∫ 1

0

(x2+y3

3|10+

1

3)dx =

∫ 1

0

(x2+1

3+

1

3)dx =

∫ 1

0

(x2 +2

3)dx = (

x3

3|10 +

2

3) =

1

3+

2

3= 1

Logo;

∫ 1

0

∫ 1

0

∫ 1

0

(x2 + y2 + z2)dzdydx = 1

82

75. Calcule∫√

2

0

∫ 3y

0

∫ 8−x2−y2

x2+3y2 dzdxdy.

Solucao . Inicialmente calculamos integral em relacao a z, depois em relacao x e por

ultimo a y, vejamos:

∫ √2

0

∫ 3y

0

∫ 8−x2−y2

x2+3y2

dzdxdy =

∫ √2

0

∫ 3y

0

(z)8−x2−y2

x2+3y2 dxdy =

∫ √2

0

∫ 3y

0

(8−2x2 +4y2)dxdy =

∫ √2

0

(8x− 2

3x3 +4xy2)3y

0 dy =

∫ √2

0

(24y−18y3 −12y3) =

= (12y2 − 15

2y4)

√2

0 = 24 − 30 = 6

Portanto;

∫ √2

0

∫ 3y

0

∫ 8−x2−y2

x2+3y2

dzdxdy = 6

83

76. Calcule∫ e

1

∫ e

1

∫ e

1dxdydz.

Solucao . Inicialmente calculamos integral em relacao a x, depois em relacao a y e por

ultimo a z, vejamos:

∫ e

1

∫ e

1

∫ e

1

dxdydz =

∫ e

1

∫ e

1

[lnx

yz]e1dydz =

∫ e

1

∫ e

1

1

yzdydz =

∫ e

1

[lny

z]e1dz =

∫ e

1

∫ e

1

1

zdz = [lnz]e1 = 1

Assim;

∫ e

1

∫ e

1

∫ e

1

dxdydz = 1

84

77. Calcule∫ 1

0

∫ 3−3x

0

∫ 3−3x−y

0dzdydx.

Solucao . Inicialmente calculamos integral em relacao a z, depois em relacao a y e por

ultimo a x, vejamos:

∫ 1

0

∫ 3−3x

0

∫ 3−3x−y

0

dzdydx =

∫ 1

0

∫ 3−3x

0

z|3−3x−y0 dydx =

∫ 1

0

∫ 3−3x

0

(3 − 3x − y)dydx =

=

∫ 1

0

(3y − 3xy − y2

2)|3−3x

0 dx =

∫ 1

0

[(3 − 3x)2 − 1

2(3 − 3x)2]dx

Colocandos alguns temos em evidencia temos;

∫ 1

0

[(3 − 3x)2 − 1

2(3 − 3x)2]dx =

9

2

∫ 1

0

(1 − x)2dx = −3

2[(1 − x)3]10(1 − x) =

3

2

Desta forma;

∫ 1

0

∫ 3−3x

0

∫ 3−3x−y

0

dzdydx =3

2

85

78. Calcule∫ 1

0

∫ Π

0

∫ Π

0ysinzdxdydz.

Solucao . Inicialmente calculamos integral em relacao a x, depois em relacao a y e por

ultimo a z, vejamos:

∫ 1

0

∫ Π

0

∫ Π

0

ysinzdxdydz =

∫ 1

0

∫ Π

0

ysinz(x)|Π0 dydz =

∫ 1

0

∫ Π

0

Πysinz =

∫ 1

0

Π(y2

2)|Π0 sinzdz =

∫ 1

0

Π3

2sinzdz =

Π3

2

∫ 1

0

sinzdz =Π3

2(−cosz)|10 =

Π3

2(1 − cos1)

Logo;

∫ 1

0

∫ Π

0

∫ Π

0

ysinzdxdydz =Π3

2(1 − cos1)

86

79. Calcule∫ 1

−1

∫ 1

−1

∫ 1

−1(x + y + z)dydxdz.

Solucao . Inicialmente calculamos integral em relacao a y, depois em relacao a x e por

ultimo a z, vejamos:

∫ 1

−1

∫ 1

−1

∫ 1

−1

(x + y + z)dydxdz =

∫ 1

−1

∫ 1

−1

[xy +y2

2+ zy]11dxdz =

∫ 1

−1

∫ 1

−1

(2x + 2z)dxdz =

=

∫ 1

−1

(x2 + 2zx)|1−1dz =

∫ 1

−1

4zdz = 0

Portanto;

∫ 1

−1

∫ 1

−1

∫ 1

−1

(x + y + z)dydxdz = 0

87

80. Calcule∫ 3

0

∫√9−x2

0

∫√9−x2

0dzdydx.

Solucao . Inicialmente calculamos integral em relacao a z, depois em relacao a y e por

ultimo a x, vejamos:

∫ 3

0

∫ √9−x2

0

∫ √9−x2

0

dzdydx =

∫ 3

0

∫ √9−x2

0

z|√

9−x2

0 dydx =

∫ 3

0

∫ √9−x2

0

9 − x2dydx =

=

∫ 3

0

y|√

9−x2

0

9 − x2dx =

∫ 3

0

(9 − x2)dx = [9x − x3

3]30 = 18

Assim;

∫ 3

0

∫ √9−x2

0

∫ √9−x2

0

dzdydx = 18

88

81. Calcule∫ 2

0

∫√

4−y2

−√

4−y2

∫ 2x+y

0dzdxdy.

Solucao . Inicialmente calculamos integral em relacao a z, depois em relacao a x e por

ultimo a y,aplicando os limites de integracao em cada uma dessas etapas, vejamos:

∫ 2

0

∫√

4−y2

−√

4−y2

∫ 2x+y

0

dzdxdy =

∫ 2

0

∫√

4−y2

−√

4−y2

z|2x+y0 dxdy =

∫ 2

0

∫√

4−y2

−√

4−y2

2x + ydxdy =

=

∫ 2

0

[x2 + xy]

√4−y2

−√

4−y2dy =

∫ 2

0

(4 − y2)12 (2y)dy = [−2

3(4 − y2)

23 ]20 =

2

3(4)

23 =

16

3

Portanto;

∫ 2

0

∫√

4−y2

−√

4−y2

∫ 2x+y

0

dzdxdy =16

3

89

82. Calcule∫ 1

0

∫ 2−x

0

∫ 2−x−y

0dzdydx.

Solucao . Inicialmente calculamos integral em relacao a z, depois em relacao a y e por

ultimo a x,aplicando os limites de integracao em cada uma dessas etapas, vejamos:

∫ 1

0

∫ 2−2x

0

∫ 2−2x−y

0

dzdydx =

∫ 1

0

∫ 2−2x

0

[z]2−2x−y0 dydx =

∫ 1

0

∫ 2−2x

0

(2 − 2x − y)dydx =

=

∫ 1

0

(2y − 2xy − y2

2)|2−2x

0 dx =

∫ 1

0

[(2 − x)2 − 1

2(2 − 2x)2]dx

Colocandos alguns temos em evidencia temos;

∫ 1

0

[(2 − 2x)2 − 1

2(2 − 2x)2]dx =

1

2

∫ 1

0

(2 − x)2dx = [−1

6(2 − x)3]10 = −1

6+

8

6=

7

6

Desta forma;

∫ 1

0

∫ 2−2x

0

∫ 2−2x−y

0

dzdydx =7

6

90

83. Calcule∫ e

1

∫ e

1

∫ e

1ln r ln s ln tdtdrds.

Solucao . Inicialmente calculamos integral em relacao a t, depois em relacao a r e por

ultimo a s,aplicando os limites de integracao em cada uma dessas etapas, vejamos:

∫ e

1

∫ e

1

∫ e

1

ln r ln s ln tdtdrds =

∫ e

1

∫ e

1

(ln r ln s)[t ln t − t]e1drds =

∫ e

1

∫ e

1

(ln r ln s)drds =

=

∫ e

1

(ln s)[r ln r − r]e1ds = [s ln s − s]e1 = 1

Assim;

∫ e

1

∫ e

1

∫ e

1

ln r ln s ln tdtdrds = 1

91

84. Calcule o volume da regiao no primeiro octante limitada pelos planos coorde-

nados, pelo plano y = 1 − x e pela superfıcie z = cos(Πx2 ), 0 6 x 6 1.

Solucao . Analizando o enuciado da questao podemos encontrar facilmente os limites de

integracao.

Em y: 0 6 y 6 1 − x

Em x: 0 6 x 6 1

Em z: 0 6 z 6 cos(Πx2 )

Assim, temos:

V =

∫ 1

0

∫ 1−x

0

∫ cos(Πx2 )

0

dzdydx =

∫ 1

0

∫ 1−x

0

[z]cos(Πx

2 )0 dydx =

∫ 1

0

∫ 1−x

0

cos(Πx

2) =

V =

∫ 1

0

cos(Πx

2)[y]1−x

0 dx =

∫ 1

0

cos(Πx

2)(1 − x) =

∫ 1

0

cos(Πx

2)dx −

∫ 1

0

xcos(Πx

2)dx

Aplicando uma subustituicao na segunda parte da integral temos:

u = x

du = dx

Logo;

V = [2

Πsin

Πx

2]01 − 4

Π2

∫ Π2

0

u cos udu =2

Π− 4

Π2[cosu + u sin u]

Π20 =

2

Π− 4

Π2(Π

2) =

4

Π2

Assim;

V =4

Π2

92

85. Calcule∫ 2Π

0

∫ 1

0

∫√2−r2

rdzrdrdΘ.

Solucao . Analizando o enuciado da questao observamos que devemos usar coordenadas

cilındricas, integrando primeiro em relacao a z, logo depois a r e por fim a Θ.

∫ 2Π

0

∫ 1

0

∫ √2−r2

r

dzrdrdΘ =

∫ 2Π

0

∫ 1

0

[z]√

2−r2

r rdrdΘ =

∫ 2Π

0

∫ 1

0

[r(2 − r2)12 − r2]drdΘ =

=

∫ 2Π

0

[−1

3(2 − r2)

32 − r3

3]10dΘ =

∫ 2Π

0

(2

23

3− 2

3)dΘ = (

223

3− 2

3)[Θ]2Π0 =

4Π(√

2 − 1)

3

Assim;

∫ 2Π

0

∫ 1

0

∫ √2−r2

r

dzdrdΘ =4Π(

√2 − 1)

3

93

86. Calcule∫ 2Π

0

∫ 3

0

∫√18−r2

r2

3

dzrdrdΘ.

Solucao . Analizando o enuciado da questao observamos que devemos usar coordenadas

cilındricas, integrando primeiro em relacao a z, logo depois a r e por fim a Θ.

∫ 2Π

0

∫ 3

0

∫ √18−r2

r2

3

dzrdrdΘ =

∫ 2Π

0

∫ 3

0

[z]√

18−r2

r2

3

rdrdΘ =

∫ 2Π

0

∫ 3

0

[r(18−r2)12−r3

3]drdΘ =

[−1

3(18 − r2)

32 − r4

12]30[Θ]2Π0 =

9Π(8√

2 − 7)

2

Portanto;

∫ 2Π

0

∫ 3

0

∫ √18−r2

r2

3

dzrdrdΘ =9Π(8

√2 − 7)

2

94

87. Calcule∫ 2Π

0

∫ Θ2Π

0

∫ 3+24r2

0dzrdrdΘ.

Solucao . Analizando o enuciado da questao observamos que devemos usar coordenadas

cilındricas, integrando primeiro em relacao a z, logo depois a r e por fim a Θ.

∫ 2Π

0

∫ Θ2Π

0

∫ 3+24r2

0

dzrdrdΘ =

∫ 2Π

0

∫ Θ2Π

0

[z]3+24r2

0 rdrdΘ =

∫ 2Π

0

∫ Θ2Π

0

(3r − 24r3)drdΘ =

=

∫ 2Π

0

[3

2r2 + 6r4]

Θ2Π0 dΘ =

3

2

∫ 2Π

0

(Θ2

4Π2+

4Θ4

16Π2)dΘ =

3

2[

Θ3

12Π2+

Θ5

5Π4]2Π0 =

17Π

5

Logo;

∫ 2Π

0

∫ Θ2Π

0

∫ 3+24r2

0

dzrdrdΘ =17Π

5

95

88. Encontre os limites da integracao para integrar f(r, θ) sobre a regiao R que

esta dentro da cardioide r = 1 + cosθ e fora da circunferencia r = 1 .

Solucao . Passo 1 : Um esboco. Esbocamos a regiao e identificamos as curvas

limitantes.

Passo 2 : Os limites de integracao de r. Um raio tıpico a partir da origem entra em

R onde r = 1 e sai onde r = 1 + cosθ.

Passo 3 : Os limites de integracao de θ . Os raios a partir da origem que apresentam

interseccao com R variam de θ = −π/2 a θ = π/2. A integral e :

∫ π2

−π2

∫ 1+cosθ

1

f(r, θ)rdrdθ

Se f(r, θ)e a funcao constante cujo valor e 1 , entao a integral de f sobre r e a area de

R .

A area de uma regiao R fechada e limitada no plano de coorde-

nadas polares e

a =

∫ ∫

R

rdrdθ.

Como seria de esperar, essa formula para a area e condizente com todas as formulas

anteriores, embora nao provemos esse fato .

96

89. Calcule∫ ∫

R

ex2+y2

dydx,

onde R a regiao semicircular limitada pelo eixo X e pela curva , y =√

1 − x2.

Solucao . Em coordenadas cartesianas, a integral em questao e uma intefral nao elemen-

tar e nao existe nenhuma maneira direta de integrar ex2+y2

em relacao a X ou Y.Ainda

assim essa integral e outras integrais como essa sao importantes em matematica — em

estatıstica por exemplo — e queremos encontrar uma maneira de calcula-la.As coorde-

nadas polares servem para isso. A substituicao de X = R cosθ , y = r sen θ e a troca de

dy dx por r dr d θ nos permitem calcular a integral como :

∫∫

R

ex2+y2

dydx =

∫ π

0

∫ 1

0

er2

rdrdθ =

∫ π

0

[1

2er2

]1

0

∫∫

R

ex2+y2

dydx =

∫ π

0

1

2(e − 1)dθ =

π

2(e − 1)

O r em r dr dθ era justamente o que precisavamos para integrar. Sem isso, estariamos

impedidos de prosseguir, como no comeco .

97

90. Encontre o momento polar da inercia em relacao a origem de uma placa fina

de densidade δ(x, y) = 1 limitada pelo quarto de circunferencia x2 + y2 = 1 no

primeiro quadrante .

Solucao . Em coordenadas cartesianas, o momento polar e o valor da integral

∫ 1

0

∫ √1−x2

0

dydx

Integrando em relacao a y, temos :

∫ 1

0

(

x2√

1 − x2 +(1 − x2)

32

3

)

dx

Uma integral difıcil de calcular sem tabelas.

As coisas melhoram se mudamos a interal original para coodenadas polares. Substituindo

x = r cos θ , y = r sen θ e trocando dxdy por r dr θ, obtemos :

∫ 1

0

∫ √1−x2

0

(x2 + y2)dydx =

∫ π2

0

∫ 1

0

(r2)rdrdθ

=

∫ π2

0

[r4

4

]r=1

r=0

dθ =

∫ π2

0

1

4dθ =

π

8

Por que a transformacao em coordenadas polares e tao eficaz aqui ? Um motivo e que

x2 + y2 e simplificada para r2 . Outro motivo e que os limites de integracao tornam-se

constantes.

98

91. Calcule∫ Π

0

∫ ΘΠ

0

∫ 3√

4−r2

−√

4−r2 zdzrdrdΘ.

Solucao . Analizando o enuciado da questao observamos que devemos usar coordenadas

cilındricas, integrando primeiro em relacao a z, logo depois a r e por fim a Θ.

∫ Π

0

∫ ΘΠ

0

∫ 3√

4−r2

−√

4−r2

zdzrdrdΘ =

∫ Π

0

∫ ΘΠ

0

z2

2rdrdΘ =

∫ Π

0

∫ ΘΠ

0

1

2[9(4−r2)−(4−r2)]rdrdΘ =

= 4

∫ Π

0

∫ ΘΠ

0

(4r−r3)drdΘ = 4

∫ Π

0

[2r2−r4

4]ΘΠ0 = 4

∫ Π

0

(2Θ2

Π2− Θ4

4Π2)dΘ = [

2Θ3

3Π2− Θ5

20Π2]Π0 =

37Π

15

Logo;

∫ Π

0

∫ ΘΠ

0

∫ 3√

4−r2

−√

4−r2

zdzrdrdΘ =37Π

15

99

92. Calcule∫ 2Π

0

∫ 1

0

∫ 1√2−r2

r3zdzrdrdΘ.

Solucao . Analizando o enuciado da questao observamos que devemos usar coordenadas

cilındricas, integrando primeiro em relacao a z, logo depois a r e por fim a Θ.

∫ 2Π

0

∫ 1

0

∫ 1√2−r2

r

3zdzrdrdΘ = 3

∫ 2Π

0

∫ 1

0

[z]1√

2−r2

r rdrdΘ = 3

∫ 2Π

0

∫ 1

0

[r(2−r2)−12−r2]drdΘ =

= 3

∫ 2Π

0

[−(2 − r2)12 − r3

3]dΘ = 3(

√2 − 4

3)[Θ]2Π0 = Π(6

√2 − 8)

Portanto;

∫ 2Π

0

∫ 1

0

∫ 1√2−r2

r

3zdzrdrdΘ = Π(6√

2 − 8)

100

93. Calcule∫ 2Π

0

∫ 1

0

∫ 12

− 12

(r2 sin2 Θ + z2)dzrdrdΘ.

Solucao . Analizando o enuciado da questao observamos que devemos usar coordenadas

cilındricas, integrando primeiro em relacao a z, logo depois a r e por fim a Θ.

∫ 2Π

0

∫ 1

0

∫ 12

− 12

(r2 sin2 Θ + z2)dzrdrdΘ =

∫ 2Π

0

∫ 1

0

(r2 sin2 Θ + [z3

3]12

− 12

)rdrdΘ =

=

∫ 2Π

0

∫ 1

0

(r3 sin2 Θ +r

12)drdΘ =

∫ 2Π

0

[r4

4sin2 Θ +

r2

24]10 =

∫ 2Π

0

(sin2 Θ

4+

1

24) =

Π

3

Desta forma;

∫ 2Π

0

∫ 1

0

∫ 12

− 12

(r2 sin2 Θ + z2)dzrdrdΘ =Π

3

101

94. Calcule∫ Π

0

∫ Π

0

∫ 2 sin φ

0ρ2 sinφdρdφdθ.

Solucao . Analizando o enuciado da questao observamos que devemos usar coordenadas

esfericas, integrando primeiro em relacao a ρ, logo depois a φ e por fim a Θ.

∫ Π

0

∫ Π

0

∫ 2 sin φ

0

ρ2 sinφdρdφdθ =

∫ Π

0

∫ Π

0

[ρ3

3]2 sin φ0 sinφdφdθ =

8

3

∫ Π

0

∫ Π

0

sin4 φdφdθ =

8

3

∫ Π

0

= ([− sin3 φ cos φ

4]Π0 +

3

4

∫ π

0

sin2 φdφ)dθ = 2

∫ π

0

∫ π

0

sin2 φdφdθ =

∫ π

0

[θ− sin 2θ

2]dθ =

∫ π

0

Πdθ = π2

Assim;

∫ Π

0

∫ Π

0

∫ 2 sin φ

0

ρ2 sinφdρdφdθ = π2

102

95. Calcule∫ 2Π

0

∫ Π4

0

∫ 2

0(ρ cos φ)ρ2 sinφdρdφdθ.

Solucao . Analizando o enuciado da questao observamos que devemos usar coordenadas

esfericas, integrando primeiro em relacao a ρ, logo depois a φ e por fim a Θ.

∫ 2Π

0

∫ Π4

0

∫ 2

0

(ρ cos φ)ρ2 sin φdρdφdθ =

∫ 2Π

0

∫ Π4

0

[ρ4

4]20 cosφ sin φdφdθ =

∫ 2Π

0

∫ Π4

0

4 cos φ sin φdφdθ =

∫ 2π

0

[2 sin φ]Π40 dθ =

∫ 2π

0

dθ = [θ]2π0 = 2π

Portanto;

∫ 2Π

0

∫ Π4

0

∫ 2

0

(ρ cos φ)ρ2 sinφdρdφdθ = 2π

103

96. Calcule∫ 2Π

0

∫ π

0

∫ 1−cos φ2

0ρ2 sin φdρdφdθ.

Solucao . Analizando o enuciado da questao observamos que devemos usar coordenadas

esfericas, integrando primeiro em relacao a ρ, logo depois a φ e por fim a Θ.

∫ 2Π

0

∫ π

0

∫ 1−cos φ2

0

ρ2 sin φdρdφdθ =

∫ 2Π

0

∫ π

0

[ρ3

3]1−cos φ

20 sinφdφdθ =

1

24

∫ 2Π

0

∫ π

0

(1−cos φ)3 sin φdφdθ =

1

96

∫ 2π

0

[(1 − cos φ)4]π0dθ =1

96

∫ 2π

0

(24 − 0)dθ =16

96

∫ 2π

0

dθ =16

96[θ]2Π0 =

1

6(2π) =

π

3

Assim;

∫ 2Π

0

∫ π

0

∫ 1−cos φ2

0

ρ2 sinφdρdφdθ =π

3

104

97. Calcule∫ 3π

2

0

∫ π

0

∫ 1

05ρ3 sin3 φdρdφdθ.

Solucao . Analizando o enuciado da questao observamos que devemos usar coordenadas

esfericas, integrando primeiro em relacao a ρ, logo depois a φ e por fim a Θ.

∫ 3π2

0

∫ π

0

∫ 1

0

5ρ3 sin3 φdρdφdθ =

∫ 3π2

0

∫ π

0

[5ρ4

4]10 sin3 φdφdθ =

5

4

∫ 3π2

0

∫ π

0

sin3 dφdθ =

5

4

∫ 3π2

0

([− sin2 φ cos φ

3]π0+

2

3

∫ π

0

sin φdφ)dθ =5

6

∫ 3π2

0

[− cos φ]π0dθ =5

3

∫ 3π2

0

dθ = [θ]3π2

0 =5π

2

Desta forma;

∫ 3π2

0

∫ π

0

∫ 1

0

5ρ3 sin3 φdρdφdθ =5π

2

105

98. Calcule∫ 2π

0

∫ π4

0

∫ sec φ

0(ρ cos φ)ρ2 sin φdρdφdθ.

Solucao . Analizando o enuciado da questao observamos que devemos usar coordenadas

esfericas, integrando primeiro em relacao a ρ, logo depois a φ e por fim a Θ.

∫ 2π

0

∫ π4

0

∫ sec φ

0

(ρ cos φ)ρ2 sin φdρdφdθ =

∫ 2π

0

∫ π4

0

∫ sec φ

0

ρ3 sin φ cos φdρdφdθ =

∫ 2π

0

∫ π4

0

[ρ4

3]sec φ0 sin φ cosφdφdθ =

1

4

∫ 2π

0

∫ π4

0

tan φ sec2 φdφdθ =1

4

∫ 2π

0

[1

2tan2 φ]

π40 dθ =

1

8

∫ 2π

0

dθ =1

8[θ]2π

0 =π

4

Logo;

∫ 2π

0

∫ π4

0

∫ sec φ

0

(ρ cos φ)ρ2 sin φdρdφdθ =π

4

106

99. Calcule∫ 2π

0

∫ π3

0

∫ 2

sec φ3ρ2 sinφdρdφdθ.

Solucao . Analizando o enuciado da questao observamos que devemos usar coordenadas

esfericas, integrando primeiro em relacao a ρ, logo depois a φ e por fim a Θ.

∫ 2π

0

∫ π3

0

∫ 2

sec φ

3ρ2 sinφdρdφdθ =

∫ 2π

0

∫ π3

0

[3ρ3

3]2sec φ sin φdφdθ =

∫ 2π

0

∫ π3

0

(8−sec3 φ) sin φdφdθ =

=

∫ 2π

0

[−8 cos φ− 1

2sec2 φ]

π30 dθ =

∫ 2π

0

[(−4−2)− (−8− 1

2)]dφ =

5

2

∫ 2π

0

dθ =5

2[θ]2π

0 =5π

2

Portanto;

∫ 2π

0

∫ π3

0

∫ 2

sec φ

3ρ2 sin φdρdφdθ =5π

2

107

100. Calcule∫ 2

0

∫ 0

−π

∫ π2

π4

ρ3 sin 2φdφdθdρ.

Solucao . Analizando o enuciado da questao observamos que devemos usar coordenadas

esfericas, integrando primeiro em relacao a φ, logo depois a θ e por fim a ρ.

∫ 2

0

∫ 0

−π

∫ π2

π4

ρ3 sin 2φdφdθdρ =

∫ 2

0

∫ 0

−π

ρ3[−cos 2φ

2]

π2π4dθdρ =

∫ 2

0

∫ 0

−π

ρ3

2dθdρ =

=

∫ 2

0

ρ3π

2dρ = [

ρ4π

8]20 = 2π

Logo;

∫ 2

0

∫ 0

−π

∫ π2

π4

ρ3 sin 2φdφdθdρ = 2π

108

101. Calcule∫ π

3π6

∫ 2 csc φ

csc φ

∫ 2π

0ρ2 sin φdθdρdφ.

Solucao . Analizando o enuciado da questao observamos que devemos usar coordenadas

esfericas, integrando primeiro em relacao a θ, logo depois a ρ e por fim a φ.

∫ π3

π6

∫ 2 csc φ

csc φ

∫ 2π

0

ρ2 sinφdθdρdφ =

∫ π3

π6

∫ 2 csc φ

csc φ

ρ2 sinφ[θ]2π0 dρdφ = 2π

∫ π3

π6

∫ 2 csc φ

csc φ

ρ2 sinφdρdφ =

=2π

3

∫ π3

π6

[ρ3 sinφ]2 csc φcsc φ dφ =

14π

3

∫ π3

π6

csc2 φdφ =14π

3[tan φ]

π3π6

=28π

3√

3

Assim;

∫ π3

π6

∫ 2 csc φ

csc φ

∫ 2π

0

ρ2 sin φdθdρdφ =28π

3√

3

109

102. Calcule∫ 1

0

∫ π

0

∫ π4

012ρ sin3 φdφdθdρ.

Solucao . Analizando o enuciado da questao observamos que devemos usar coordenadas

esfericas, integrando primeiro em relacao a φ, logo depois a θ e por fim a ρ.

∫ 1

0

∫ π

0

∫ π4

0

12ρ sin3 φdφdθdρ =

∫ 1

0

∫ π

0

(12ρ[− sin2 φ cos φ

3]

π40 + 8ρ

∫ π4

0

sin φdφ)dθdρ =

=

∫ 1

0

∫ π

0

(− 2ρ√2− 8ρ[cos φ]

π40 )dθdφ =

∫ 1

0

∫ π

0

(8ρ − 10ρ√2

)dθdφ =

∫ 1

0

(8ρ − 10ρ√2

)[θ]π0dρ =

= π

∫ 1

0

(8ρ − 10ρ√2

)dρ = π[4ρ2 − 5ρ2

√2

]10 =(4√

2 − 5)π√2

Desta forma;

∫ 1

0

∫ π

0

∫ π4

0

12ρ sin3 φdφdθdρ =(4√

2 − 5)π√2

110

103. Calcule∫ π

2π6

∫ π2

−π2

∫ 2

csc φ5ρ4 sin3 φdρdθdφ.

Solucao . Analizando o enuciado da questao observamos que devemos usar coordenadas

esfericas, integrando primeiro em relacao a ρ, logo depois a θ e por fim a φ.

∫ π2

π6

∫ π2

−π2

∫ 2

csc φ

5ρ4 sin3 φdρdθdφ =

∫ π2

π6

∫ π2

−π2

[5ρ5

5]2csc φ sin3 φdθdφ =

∫ π2

π6

∫ π2

−π2

(32−csc5) sin3 φdθdφ =

=

∫ π2

π6

∫ π2

−π2

(32 sin3 φ − csc2)dθdφ =

∫ π2

π6

(32 sin3 φ − csc2)[θ]π2

−π2dφ =

= π

∫ π2

π6

(32 sin3 φ − csc2)dφ = π[−32 sin2 cosφ

3]

π2π6

+64π

3

∫ π2

π6

sin φdφ + π[cot φ]π2π6

=

= π(32√

3

24) − 64π

3[cos φ]

π2π6

+ π(√

3) =

√3

3π + (

64π

3)(

√3

2) =

33π√

3

3= 11π

√3

Portanto;

∫ π2

π6

∫ π2

−π2

∫ 2

csc φ

5ρ4 sin3 φdρdθdφ = 11π√

3

111

104. Calcule∫

B

∫ydxdy onde B(0,0),(1,0) e (1,1)

Solucao . Fazendo o estudo dos pontos achamos o intervalo de integracao e calculamos

∫ 1

0

∫ y

1

ydxdy

=

∫ 1

0

xy

∣∣∣∣∣

y

1

dy

=

∫ 1

0

y − y2dy

=y2

2− y3

3

=1

2− 1

3

Assim, chegamos ao valor da integral do problema

=1

6

112

105. Calcule∫

B

∫ydxdy onde {(x, y) ∈ R2| − 1 ≤ x ≤ 1, 0 ≤ y ≤ x + 2}

Solucao . Com os intervalos de integracao dados, calculamos a integral dupla

∫ 1

−1

∫ x+2

0

ydydx

=

∫ 1

−1

y2

2

∣∣∣∣∣

x+2

0

dx

=1

2

∫ 1

−1

(x + 2)2dx

=1

2

∫ 1

−1

x2 + 4x + 4dx

=1

2(x3

3+ 2x2 + 4x)

∣∣∣∣∣

1

−1

=1

2(1

3+ 2 + 4 +

1

3− 2 + 4)

=1

2(1 + 6 + 12 + 1 − 6 + 12

3)

Assim o valaor da integral do problema e

=13

3

113

106. Calcule∫

B

∫ydxdy onde B: (-1,0), (0,0), (1,1) e (0,1)

Solucao . Com os pontos dados no problema achamos os intervamos de integracao e

diante disso a integral dupla e calculada assim:

∫ 1

0

∫ y

y−1

ydxdy

=

∫ 1

0

∫ y

y−1

xy

∣∣∣∣∣

y−1

y

dy

=

∫ 1

0

yy − (y − 1)ydy

=

∫ 1

0

y2 − y2 + ydy

=y2

2

∣∣∣∣∣

1

0

=1

2

114

107. Calcule∫

B

∫xdxdy onde B: (0,0), (1,1) e (2,0)

Solucao . Com os pontos dados no problema achamos os intervamos de integracao ,

calculamos a integral dupla abaixo:

∫ 1

0

∫ 2−y

y

xdxdy

=1

2

∫ 1

0

x2

∣∣∣∣∣

2−y

y

dy

=1

2

∫ 1

0

(2 − y)2 − y2dy

=1

2

∫ 1

0

4 − 4y + y2 − y2

=1

2(4y − 2y2)

∣∣∣∣∣

1

0

Chegamos ao resuldado da integral do problema

= 1

115

108. Calcule o volume do conjunto dado. x ≥ 0 , x ≤ y ≤ 1 e 0 6 z 6 ey2

Solucao .

∫ 1

0

∫ y

0

ey2

dxdy =

∫ 1

0

(xey2

)

∣∣∣∣∣

y

0

dy

=

∫ 1

0

yey2

dy

Chamando u = y2 e du = 2ydy temos :

=1

2

∫ 1

0

eudu

=1

2(eu)

∣∣∣∣∣

1

0

=1

2(e1 − e0)

=1

2(e1 − 1)

116

109. Calcule o volume do conjunto dado. x2 + y2 ≤ z ≤ 1 − x2

Solucao .

Fazendo: z = 1 − x2 − x2 − y2 temos que z = 1 − 2x2 − y2

Utilizando coordenadas polares temos que:

x = r√2

cos θ 0 ≤ r ≤ 1

y = r sin θ 0 ≤ θ ≤ 2π. Efetuando o

calculo do jacobiano temos:

j =

∣∣∣∣∣

d(x,y)d(r,θ)

∣∣∣∣∣

=

∣∣∣∣∣∣

1√2

cos θ − r√2

sin θ

sin θ r cos θ

∣∣∣∣∣

= r√2

cos2 θ + r√2

sin2 θ = r√2

∫ 2π

0

∫ 1

0

(1 − 2r2

2cos2 θ − r2 sin2 θ)

r√2drdθ =

∫ 2π

0

∫ 1

0

(1 − r2)r√2drdθ

=

∫ 2π

0

∫ 1

0

r√2− r3

√2drdθ

=1√2

∫ 2π

0

r2

2− r4

4

∣∣∣∣∣

1

0

=1√2

∫ 2π

0

1

4dθ

=1

4√

22π

2√

2

117

110. Calcule o volume do conjunto de todos os (x,y,z) tais que 0 ≤ x ≤ 1 , 0 ≤ y ≤ 1

e 0 ≤ z ≤ x2 + y2. O volume de tal conjunto e∫ 1

0

∫ 1

0(x2 + y2)dxdy

Solucao . Como o volume dado a formula foi dado, resolvendo a integral dupla temos:

∫ 1

0

∫ 1

0

x2 + y2dxdy

=

∫ 1

0

(x3

3+ xy2)

∣∣∣∣∣

1

0

dy

=

∫ 1

0

(1

3+ y2)dy

=1

3y +

y3

3

∣∣∣∣∣

1

0

=1

3+

1

3

O volume do conjunto de todos os (x,y,z) e

=2

3

118

111. Calcule∫

B

∫(xy)dxdy onde B e o conjunto de todos os (x,y,z) tais que 0 ≤ x ≤ 1

e 0 ≤ y ≤ x2

Solucao .Tendo os intervalos dados no problema so precisamos substituir na integral

dupla abaixo:

∫ 1

0

∫ x2

0

xydydx

=

∫ 1

0

xy2

2

∣∣∣∣∣

x2

0

dx

=

∫ 1

0

xx4

2dx

=x6

12

∣∣∣∣∣

1

0

Assim, o valor da integral e

=1

12

119

112. Calcule usando integral dupla o conjunto dado onde 1 ≤ x ≤ 2 e 0 ≤ y ≤ 1, de

x cos xy

Solucao . Diante do intervalo do conjunto dado, temos a integral dupla:

∫ 2

1

∫ 1

0

x cos xydydx

Integrando com relacao a y

=

∫ 2

1

x sin xy

x

∣∣∣∣∣

1

0

dx

=

∫ 2

1

x sin x

xdx

=

∫ 2

1

sinxdx

Integrando com relacao a x

= − cos x

∣∣∣∣∣

2

1

Ou seja, a integral dupla no intervalo dado e

= − cos 2 + cos 1

120

113. Calcule usando integral dupla o conjunto dado onde 1 ≤ x ≤ 2 e 0 ≤ y ≤ 1,

x sin πy

Solucao . Diante do intervalo do conjunto dado, temos a integral dupla:

∫ 1

0

∫ 2

1

x sin πydxdy

Integrando com relacao a x

=

∫ 1

0

x2

2

∣∣∣∣∣

1

0

sin πydy

=3

2

∫ 1

0

sin πydy

Integrando com relacao a y

= −3

2fcosπyπ

∣∣∣∣∣

1

0

=3

2π+

3

Ou seja, a integral dupla no intervalo dado e

=3

π

121

114. Calcule∫

A

∫(xyex2−y2

)dxdy, onde A e o retangulo −1 ≤ x ≤ 1 e 0 ≤ y ≤ 3

Solucao . Resolvendo a integral dupla definida pelo retangulo do problema, de imediato

ja possuimos os intervalos de integracao :

∫ 3

0

∫ 1

−1

xyex2−y2

dxdy

Fazendo uma substituicao afim de facilitar nossos calculos, chamamos u = x2 − y2 e

du = 2xdx

=

∫ 3

0

xyeu 1

2x

∣∣∣∣∣

1

−1

dy

=

∫ 3

0

y

2eu

∣∣∣∣∣

1

−1

dy

Assim,

=

∫ 3

0

y

2ex2−y2

∣∣∣∣∣

1

−1

dy

=

∫ 3

0

y

2(e1−y2 − e1−y2

)dy

= 0

122

Nas questoes de 13 a 21 Calcule o volume do conjunto dado

115. Calcule o volume do conjunto dado, x2 + y2 e x + y + 2 6 z 6 4

Solucao . Precisamos de z para resolver problemas de volume, fazendo z = 4− x− y− 2

temos que z = 2 − x − y

Percebemos que o conjunto dado nos permite a utilizacao de coordenadas polares onde

x = r cos θ 0 ≤ r ≤ 1

y = r sin θ 0 ≤ θ ≤ 2π

Assim, substituindo x e y na equacao z pelas coordenadas polares encontradas temos:

∫ 2π

0

∫ 1

0

(2 − r cos θ − r sin θ)rdrdθ

=

∫ 2π

0

∫ 1

0

(2r − r2 cos θ − r2 sin θ)drdθ

=

∫ 2π

0

∫ 1

0

2r − r2(cos θ + sin θ)drdθ

Integrando com relacao a r temos:

=

∫ 2π

0

r2 − r3

3(cos θ + sin θ)

∣∣∣∣∣

1

0

=

∫ 2π

0

1 − 1

3(cos θ + sin θ)dθ

=

∫ 2π

0

dθ − 1

3

∫ 2π

0

cos θdθ

Integrando com relacao a θ temos:

= 2π + (1

3senθ)

∣∣∣∣∣0

2π + (cosθ)

∣∣∣∣∣0

= 2π

Ou seja, o volume do conjunto dado e 2π

123

116. Calcule o volume do conjunto dado, x ≥ 0 , y ≥ 0 , x + y ≤ 1 e 0 ≤ z ≤ x2 + y2

Solucao . Precisamos de z para resolver problemas de volume, fazendo z = x2 + y2 − 2

e que y = 1 − x

Assim:

∫ 1

0

∫ 1−x

0

x2 + y2dydx

=

∫ 1

0

(x2y +y3

3)

∣∣∣∣∣

1−x

0

dx

=

∫ 1

0

(x2(1 − x) +(1 − x)3

3)dx

=1

3

∫ 1

0

(3x2 − 3x2 + x3 + 3x2 − 3x + 1)dx

=1

3

∫ 1

0

(−2x3 + 6x2 − 3x + 1)dx

=1

3(−x4

2+ 2x3 − 3x2

2+ x)

∣∣∣∣∣

1

0

=1

3(−1

2+ 2 − 3

2+ 1)

Entao o volume do problema e

=1

3

124

117. Calcule o volume do conjunto dado, 0 ≤ y ≤ 1 − x2 , 0 ≤ z ≤ 1 − x2

Solucao . Sabemos que o volume e necessita de z para fazer a integracao dupla, fazendo

z = 1 − x2

temos

∫ 1

−1

∫ 1−x2

0

1 − x2dydx

Integrando com relacao a y

=

∫ 1

−1

(y − yx2)

∣∣∣∣∣

1−x2

0

dx

=

∫ 1

−1

1 − x2 − (1 − x2)x2dx

=

∫ 1

−1

(1 − x2 − x2 + x4)dx

=

∫ 1

−1

(x4 − 2x2 + 1)dx

Integrando com relacao a x

= (x5

5− 2x3

3+ x)

∣∣∣∣∣

1

−1

= (1

5− 2

3+ 1) − (−1

5+

2

3− 1)

= (1

5− 2

3+ 1 +

1

5− 2

3+ 1)

Obtemos o volume do conjunto, que e

=16

15

125

118. Calcule o volume do conjunto dado, x2 + y2 + 3 6 z 6 4.

Solucao . Isolando o z no intervalo dado temos z = 4− x2 − y2 − 3 onde z = 1− x2 − y2

Mais uma vez o problema nos permite a utilizacao de coordenadas polares onde:

x = r cos θ 0 ≤ r ≤ 1

y = r sin θ 0 ≤ θ ≤ 2π

Assim, substituindo x e y na equacao z pelas coordenadas polares encontradas temos:

∫ 2π

0

∫ 1

0

(2 − r2 cos2 θ − r2 sin2 θ)rdrdθ

=

∫ 2π

0

∫ 1

0

1 − r2(cos2 θ + sin2 θ)rdrdθ

=

∫ 2π

0

∫ 1

0

2r − r2(cos θ + sin θ)drdθ

=

∫ 2π

0

∫ 1

0

r − r3drdθ

=

∫ 2π

0

(r2

2− r4

4)

∣∣∣∣∣

1

0

=

∫ 2π

0

1

4dθ

Integrando com relacao a θ temos:

2

126

119. Calcule o volume do conjunto dado, x2 + 4y2 6 4 e x + y 6 z 6 x + y + 1

Solucao . Fazendo z = x + y + 1 − x − y temos que z = 1

Utilizando as coordenadas polares ja que nossa funcao e do tipo Ax2 + By2 temos que

x = r cos θ 0 ≤ r ≤ 2

y = r2 sin θ 0 ≤ θ ≤ 2π

. Observe que o jacobiano mudou pois (2y) = rsenθ. Assim,

substituindo x e y na equacao z pelas coordenadas polares encontradas temos:

∫ 2π

0

∫ 2

0

(1)r

2drdθ

=

∫ 2π

0

(r2

4)

∣∣∣∣∣

2

0

= 1

∫ 2π

0

Integrando com relacao a θ temos:

= 2π

127

120. Calcule o volume do conjunto dado, x ≥ 0 , x ≤ y ≤ 1 e 0 6 z 6 ey2

Solucao . Apartir do conjunto dado temos:

∫ 1

0

∫ y

0

ey2

dxdy

=

∫ 1

0

(xey2

)

∣∣∣∣∣

y

0

dy

=

∫ 1

0

yey2

dy

Fazeno uma substituicao simples, chamando u = y2 e du = 2ydy temos:

=1

2

∫ 1

0

eudu

=1

2(eu)

∣∣∣∣∣

1

0

=1

2(e1 − e0)

Assim, o volume e:

=1

2(e1 − 1)

128

121. Calcule o volume do conjunto dado, x2 + y2 ≤ z ≤ 1 − x2.

Solucao . Achando z isolando um dos lados da desigualdade temos z = 1− x2 − x2 − y2

onde z = 1 − 2x2 − y2

Como o conjunto nos permite a utilizacao de coordenadas polares temos que:

x = r√2

cos θ 0 ≤ r ≤ 1

y = r sin θ 0 ≤ θ ≤ 2π

Efetuando o calculo do jacobiano temos:

j =

∣∣∣∣∣

d(x,y)d(r,θ)

∣∣∣∣∣

=

∣∣∣∣∣∣

1√2

cos θ − r√2

sin θ

sin θ r cos θ

∣∣∣∣∣

=r√2

cos2 θ +r√2

sin2 θ =r√2

Assim, substituindo x e y na equacao z pelas coordenadas polares encontradas temos:

∫ 2π

0

∫ 1

0

(1 − 2r2

2cos2 θ − r2 sin2 θ)

r√2drdθ

=

∫ 2π

0

∫ 1

0

(1 − r2)r√2drdθ

=

∫ 2π

0

∫ 1

0

r√2− r3

√2drdθ

=1√2

∫ 2π

0

r2

2− r4

4

∣∣∣∣∣

1

0

=1√2

∫ 2π

0

1

4dθ

=1

4√

22π

Concluimos entao , que o volume e:

2√

2

129

122. Calcule o volume do conjunto dado, x2 + y2 ≤ z ≤ 2x

Solucao . z = 2x − x2 − y2 ⇒ (x − 1)2 + y2 = 1

Fazendo uso das coordenadas polares temos que:

x = 1 + r cos θ 0 ≤ r ≤ 1

y = r sin θ 0 ≤ θ ≤ 2π

Substituindo os valores obtidos na transformacao de variaveis em z temos:

∫ 2π

0

∫ 1

0

2(r cos θ + 1) − (r cos θ + 1)2 − (r sin θ)2rdrdθ

=

∫ 2π

0

∫ 1

0

2r cos θ + 2 − (r2 cos2 θ + 2r cos θ + 1) − (r2 sin2 θ)rdrdθ

=

∫ 2π

0

∫ 1

0

(2r cos θ + 2 − r2 cos2 θ − 2r cos θ − 1 − r2 sin2 θ)rdrdθ

=

∫ 2π

0

∫ 1

0

1 − r2(sin2 θ + cos2 θ)rdrdθ

=

∫ 2π

0

(r2

2− r4

4)

∣∣∣∣∣

1

0

=

∫ 2π

0

(1

2− 1

4)dθ

=

∫ 2π

0

1

4dθ

2

130

123. Utilize da integral dupla para calcular a area do conjunto B e o conjunto de

todos (x,y) tais que ln x ≤ y ≤ 1 + ln x, y ≥ 0 e x ≤ e

Solucao . Utilizando do calculo de area ondee definido como sendo a integral dupla de

dxdy, e sendo dado o intervalo de integracao de y e x temos

∫ e

0

∫ 1+ln x

ln x

dydx

=

∫ e

0

(1 + ln x − ln x)dx

=

∫ e

0

1dx

= x

∣∣∣∣∣

e

0

Logo, a area e:

= e

131

124. Utilize da integral dupla para calcular a area do conjunto B = {(x, y) ∈ R2|x3 ≤y ≤ √

x}

Solucao . Igualando os x temos:

x3 = x12

x3 − x12 = 0

x(x2 − x− 1

2) = 0

x = 0

x − x− 1

2= 0

x2 − 1√x

x2√

x − 1 = 0

x2x12 − 1 = 0

x32 = 1

x = 1

Depois de encontrado os limites de x (onde as curvas se interceptam) temos:

∫ 1

0

∫ √x

x3

dydx

=

∫ 1

0

(√

x − x3)dx

=

∫ 1

0

(x12 − x3)dx

=x

32

32

− x4

4

∣∣∣∣∣

1

0

=2

3(1)

32 − 1

4

=8 − 3

12

Assim, a area do problema dado e:

=5

12

132

125. Calcule∫

A

∫sin2 x1+4y2 dxdy, onde A e o retangulo 0 ≤ x ≤ π

2 , 0 ≤ y ≤ 12

Solucao .∫ 1

2

0

∫ π2

0

sin2 x

1 + 4y2dxdy

Temos que sin2 x = 1−cos 2x2 , assim a integral dupla fica:

=

∫ 12

0

∫ π2

0

(1

2 + 8y2)1 − cos 2xdxdy

=

∫ 12

0

1

2 + 8y2(x − sin 2x

2)

∣∣∣∣∣

π2

0

dy

=

∫ 12

0

1

2 + 8y2(π

2)dy

=

∫ 12

0

π

4 + 16y2dy

=

∫ 12

0

π16

416 + y2

dy

Sabemos que∫

1a2+y2 = 1

a arctan ya

16

∫ 12

0

1

( 12 )2 + y2

dy

16(112

arctany12

)

∣∣∣∣∣

12

0

16(2 arctan 2y)

∣∣∣∣∣

12

0

16(2

π

4)

Logo, a integral dupla do problema tem como solucao

=π2

32

133

126. Calcule a area da regiao limitada pela elipse x2

a2 + y2

b2 = 1, onde (a > 0 e b > 0)

Solucao . Utilizando de coordenadas polares temos que:

x = ar cos θ 0 ≤ r ≤ 1

y = br sin θ 0 ≤ θ ≤ 2π

Fazendo o calculo do jacobiano temos:

j =

∣∣∣∣∣∣

a cos θ −ar sin θ

b sin θ br cos θ

∣∣∣∣∣

= abr cos2 θ + abr sin2 θ = abr

Aplicando a integral dupla para o calculo de area temos:

∫ 2π

0

∫ 1

0

abrdrdθ

=ab

2

∫ 2π

0

(r2

2)

∣∣∣∣∣

1

0

=ab

2

∫ 2π

0

=ab

2(2π)

O resultado foi o esperado para o problema

= abπ

134

127. Calcule usando coordenadas polares∫ ∫

Bxydxdy onde B e o circulo

x2 + y2 − 2y ≤ 0 , x ≥ 0 e assumindo 0 ≤ θ ≤ π2 e 0 ≤ r ≤ 2 sin θ.

Solucao . Completando quadrado na desigualdade dada, temos que

x2 + y2 − 2y = 0

x2 + (y − 1)2 − 1 = 0

x2 + (y − 1)2 = 1

Assim,

V x = r cos θ

V y = r sin θ

0 ≤ θ ≤ π

2

0 ≤ r ≤ 2 sin θ

Resolvendo a integral, nao esquecendo do jacobiano pois mudamos para coordenadas

polares temos

⇒∫ π

2

0

∫ 2 sin θ

0

r2 cos θ sin θrdrdθ =

∫ π2

0

[r4 cos θ sin θ

4|2 sin θ0 ]dθ

= 4

∫ π2

0

(cos θ sin5 θ)dθ

Fazendo uma substituicao simples

u = sin θ

du = cos θdθ

= 4

∫ π2

0

u5du

= 4(u6

6|

π20 )

= 4(sin4 θ

6|

π20 )

Entao a integral dupla do problema tem como solucao

=2

3

135

128. Encontre o volume da regiao D limitada pelas superfıcies z = x2 + 3y2 z =

8 − x2 − y2.

Solucao . O volume e

v =

∫ ∫

R

dzdydx,

a integral de f(x, y, z) = 1 sobre D . Para encontrarmos os limites de integracao para

calcular a integral, seguimos estes passos :

Passo 1 : Um esboco. As superf’icies apresentam interseccao no cilindro elıptico

x2 + 3y2 = 8 − x2 − y2 ou x2 + 2y2 = 4. A fronteira da regiao R ( a projecao de D spbre

o plano xy ) e uma elipse com a mesma equacao : x2 + 2y2 = 4 . A fronteira superior de

R e a curva y =√

(4 − x2)/2 . A fronteira inferior e a curva y = −√

(4 − x2)/2 .

Passo 2 : Os limites de integracao de Z . A reta M que passa por um ponto tıpico (x, y)

em R que e paralela ao eixo Z entra em D em z = x2 + 3y2 e sai em z = 8 − x2 − y2 .

Passo 3 : Os limites de integracao de y. A reta L que passa por (x, y) que e paralela ao

eixo y entra em R em y = −√

(4 − x2)/2 e sai em y =√

(4 − x2)/2 .

Passo 4 : Os limites de integracao de x . Quando L varre R, o valor de X varia de x = −2

em (−2, 0) a x = 2 em (2, 0, 0) . O volume e

v =∫ ∫

R

∫dzdydx

=∫ 2

−2

∫√

(4−x2)/2

−√

(4−x2)/2

∫ 8−x2−y2

x2+3y2 dzdydx

=∫ 2

−2

∫√

(4−x2)/2

−√

(4−x2)/2(8 − 2x2 − 4y2)dydx

=∫ 2

−2

[(8 − 2x2)y − 4

3y3]√

(4−x2)/2

−√

(4−x2)/2dx

=∫ 2

−2

(

2(8 − 2x2)√

4−x2

2 − 83

(4−x2

2

) 32

)

dx

∫ 2

−2

[

8(

4−x2

2

) 32 − 8

3

(4−x2

2

) 32

]

dx = 4√

23

∫ 2

−2(4 − x2)

32 dx

= 8π√

2 Unidades cubicas.

136

129. Encontre um centroide (δ = 1) do solido limitado pelo cilindro x2 + y2 = 4 e

limitado acima pelo paraboloide z = x2 + y2 e abaixo pelo plano xy.

Solucao .

Passo 1 : Um esboco. Esbocamos o solido, limitado acima pelo paraboloide z = r2 e

abaixo pelo plano z = 0 . Sua base R e o disco |r| ≤ 2 no plano xy . O centroide do

s’olido (x, y, z) est’a sobre seu eixo de simetria, neste caso o eixo z. Isso faz x = y = 0 .

Para encontrarmos z , dividimos o primeiro momento Mxy pela massa M .

Passo 2 : Os limites de z . Uma reta M que passa por um ponto tıpico (r, θ) na base

paralela ao eixo z entra no solido en z = 0 e sai em z = r2 .

Passo 3 : Os limites de r . Um raio L que passa por (r, θ) a partir da origem entra em r

em r = 0 e sai em r = 2 .

Passo 4 : Os limites de θ . A medida que L varre a base no sentido anti -horario, o angulo

θ que ele faz com o eixo x positivo varia de θ = 0 a θ = 2π. O valor de Mxy e

Mxy =

∫ 2π

0

∫ 2

0

∫ r2

0

zdzrdrdθ =

∫ 2π

0

∫ 2

0

[z2

2

]r2

0

rdrdθ =

∫ 2π

0

∫ 2

0

r5

2drdθ

∫ 2π

0

[r6

12

]2

0

dθ = =

∫ 2π

0

16

3dθ = =

32π

3.

O valor de M e

M =

∫ 2π

0

∫ 2

0

∫ r2

0

dzrdrdθ =

∫ 2π

0

∫ 2

0

[z]r2

0 rdrdθ =

∫ 2π

0

∫ 2

0

r3drdθ

∫ 2π

0

[r4

4

]2

0

dθ =

∫ 2π

0

4dθ = 8π.

Portanto ,

z =Mxy

M=

32π

3

1

8π=

4

3.

e o centroide e (0, 0, 43 ) . Observe que o centroide esta fora do solido

137

130. Encontre uma equac ao em coordenadas esfericas para o cone z =√

x2 + y2.

Solucao 1. Use geometria. O cone e simetrico em relacao ao eixo z e corta o primeiro

quadrante do plano yz ao longo da reta z = y. O angulo entr o cone e o eixo z positivo

e portanto, π/4 radianos. O cone consiste nos pontos cujas coodenadas esfericas tem φ

igaul a π/4 , assim sua equacao e φ = π/4 .

Solucao 2. Use algebra. Se usarmos as equacoes (3) para substituir x , y e z , obteremos

o mesmo resultado :

z =√

x2 + y2

ρ cos φ =√

ρ2 sen2 φ

ρ cos φ = ρ sen φ

cos φ = sen φ

φ =π

4

138

131. Calcule a integral

a)∫ 1

0

∫ 2

1x cos xy dxdy

Solucao:

Fazendo a substituicao simples, temos:

xy = u

du = x dy

Assim,

∫ 2

1

[∫ 1

0cosudu

]

dx =∫ 2

1[senxy]

10 dx =

∫ 2

1senxdx = [− cos x]

21 =

= − cos 2 + cos 1

139

132. Encontre os limites da integracao para integrar f(r, θ) sobre a regiao R que

esta dentro da cardioide r = 1 + cosθ e fora da circunferencia r = 1 .

Solucao

Passo 1 : Um esboco. Esbocamos a regiao e identificamos as curvas limitantes.

Passo 2 : Os limites de integracao de r. Um raio tıpico a partir da origem entra em

R onde r = 1 e sai onde r = 1 + cosθ.

Passo 3 : Os limites de integracao de θ . Os raios a partir da origem que apresentam

interseccao com R variam de θ = −π/2 a θ = π/2. A integral e :

∫ π2

−π2

∫ 1+cosθ

1

f(r, θ)rdrdθ

Se f(r, θ)e a funcao constante cujo valor e 1 , entao a integral de f sobre r e a area de

R .

A area de uma regiao R fechada e limitada no plano de coorde-

nadas polares e

a =

∫ ∫

R

rdrdθ.

Como seria de esperar, essa formula para a area e condizente com todas as formulas

anteriores, embora nao provemos esse fato .

140

133. Calcule∫ ∫

R

ex2+y2

dydx,

onde R a regiao semicircular limitada pelo eixo x e pela curva , y =√

1 − x2.

Solucao . Em coordenadas cartesianas, a integral em questao e uma integral nao

elementar e nao existe nenhuma maneira direta de integrar ex2+y2

em relacao a x ou

y. Porem, as coordenadas polares servem para isso. A substituicao de x = r cosθ , y = r

sen θ e a troca de dydx por rdrdθ nos permitem calcular a integral como :

∫ ∫

R

ex2+y2

dydx =

∫ π

0

∫ 1

0

er2

rdrdθ =

∫ π

0

[1

2er2

]1

0

dθ =

∫ π

0

1

2(e − 1)dθ =

π

2(e − 1).

141

134. Calcule o volume dado por

∫∫

A

xsen(πy) dA onde A e a regiao compreendida

em 0 ≤ x ≤ 1 e 1 ≤ y ≤ 2:

Solucao . Temos, entao :

V =

∫ 2

1

∫ 1

0

xsen (πy) dydx

=

∫ 2

1

[

−x

πcos(πy)

]1

0dx

=

∫ 2

1

−x

π(cos π − cos 0) dx

=

∫ 2

1

−x

π(−1 − 1) dx

=

∫ 2

1

2x

πdx =

[x2

π

]2

1

=4

π− 1

π=

3

π

142

135. Calcule o volume dado por

∫ 1

0

∫ 2

1

yexy dxdy :

Solucao .

V =

∫ 1

0

∫ 2

1

yexydx dxdy =

∫ 1

0

β(x) dy

Onde

β(x) =

∫ 2

1

yexy dx

Tome u = xy → du = ydx

x = 1; u = y

x = 2; u = 2y

β(u) =

∫ 2y

y

eu du = eu∣∣2yy = e2y − ey

Logo,

V =

∫ 1

0

β(x) dy

=

∫ 1

0

(e2y − ey) dy

=

[1

2e2y − ey

]1

0

=

[1

2e2 − e

]

−[1

2e0 − e0

]

=1

2(e − 1)2

143

136. Calcule

∫ 2

1

∫ 1

0

1

1 + x2 + 2xy + y2dydx

Solucao . Fatorando o divisor, temos

∫ 2

1

∫ 1

0

1

1 + (x + y)2dydx

Tome u = x + y → du = dy

y = 0; u = x

y = 1; u = x + 1

⇒∫ 2

1

∫ x+1

x

1

1 + u2dudx =

∫ 2

1

[arctg u]x+1x dx

=

∫ 2

1

arctg (x + 1)︸ ︷︷ ︸

I

− arctg x︸ ︷︷ ︸

τ

dx

Integrando I por partes, temos

u = arctg (x + 1) → du =1

1 + (x + 1)2

dv = dx → v = x

⇒ I = [xarctg (x + 1)]21 −

∫ 2

1

x

1 + (x + 1)2dx

Tome u = x + 1onde x = u − 1 e du = dx

⇒∫

u − 1

1 + u2du =

∫ (u

1 + u2− 1

1 + u2

)

du

=1

2ln(1 + u2) − arctg u + k =

1

2ln([1 + (x + 1)2) − arctg (x + 1)

Logo,

I =

[

xarctg (x + 1) −(

1

2ln(1 + (x + 1)2) − arctg (x + 1)

)]2

1

⇒ I = 3arctg 3 − 2arctg 2 − 1

2ln 10 +

1

2ln 5

144

De modo ana logo a I, temos

τ =

[

xarctg x − 1

2ln(1 + x2)

]2

1

= 2arctg 2 − arctg 1 − 1

2ln 5 +

1

2ln 2

Portanto

∫ 2

1

∫ 1

0

1

1 + x2 + 2xy + y2dydx = 3arctg 3 − 2arctg 2 − 1

2ln 10 +

1

2ln 5 +

π

4− 1

2ln 2

= 3arctg 3 − 4arctg − ln 2 +1

2ln 5 +

π

4

145

137. Calcule o volume dado por

∫ ∫

B

1

ln ydB e B = {(x, y) ∈ R2 | 2 ≤ y ≤ 3, 0 ≤ x ≤ 1

y}

Solucao . Temos,

V =

∫ 3

2

∫ 1y

0

1

ln ydxdy =

∫ 3

2

[x

ln y

] 1y

0

dy =

∫ 3

2

1

y ln ydy

Tome u = ln y → du = 1y dy

y = 2; u = ln 2

y = 3; u = ln 3

Entao ,

V =

∫ ln 3

ln 2

1

udu = [ln u]

ln 3ln 2 = ln (ln 3) − ln (ln 2)

146

138. Calcule o volume do solido limitado por f(x, y) = yxcos(x2) na regiao dada por B =

{(x, y) ∈ R2 | 0 ≤ x ≤ 1, x2 ≤ y ≤ 1}

Solucao . Sabemos que o volume eigual a:

V =

∫ 1

0

∫ 1

x2

yx cosx2 dydx

=

∫ 1

0

[

xy2

2cos x2

]1

x2

dx

=1

2

∫ 1

0

x cos x2(1 − x4) dx

=1

2

∫ 1

0

(x cos x2 − x5 cosx2) dx

=1

2

∫ 1

0

x cos x2 dx − 1

2

∫ 1

0

x5 cosx2︸ ︷︷ ︸

I

dx

=1

4

[sen x2

]1

0− 1

2I

Em I tome u = x2 → du = 2xdx

x = 0; u = 0

x = 1; u = 1

⇒ I =1

2

∫ 1

0

u2 cosu du

Integrando I por partes, temos

r = u2 → dr = 2udu

ds = cosu → s = senu

⇒ I = u2sen u|10 − 2

∫ 1

0

usenu︸ ︷︷ ︸

τ

du

Integrando τ por partes, temos

τ = −u cosu|10 + sen u|10

147

Logo,

I =1

2

[u2sen u − 2(−u cos u + sen u)

]1

0

=1

2[sen 1 + 2 cos 1 − 2sen 1]

= cos 1 − 1

2sen1

Portanto

V =1

4

[sen x2

]1

0− 1

2I

=1

4sen 1 − 1

2cos 1 +

1

4sen 1

=1

2sen 1 − 1

2cos 1

148

139. Integre

∫∫

B

(x2 + 2y) dxdy onde B = x2 + y2 ≤ 4, usando coordenadas polares.

Solucao . Mudando para coodenadas polares, temos

r2 cos2 θ + r2sen2 θ = 4

⇒ r =√

4 = 2

0 ≤ r ≤ 2

0 ≤ θ ≤ 2π

⇒∫ 2π

0

∫ 2

0

[r2 cos2 θ + 2rsen θ

]rdrdθ =

∫ 2π

0

∫ 2

0

[r3 cos2 θ + 2r2sen θ

]drdθ

=

∫ 2π

0

[r4

4cos2 θ +

2

3r3sen θ

]2

0

dθ =

∫ 2π

0

[

4 cos2 θ +16

3sen θ

]

=

[

4

(1

2θ +

1

4sen 2θ

)

− 16

3cos θ

]2π

0

= 4π

149

140. Resolva

∫∫

B

sen (4x2 + y2) dxdy onde B = {∀(x, y) ∈ R2 |x2 + y2 ≤, y ≥ 0}

Solucao . Tome

2x = r cos θ

y = rsen θ

x = 12r cos θ

y = rsen θ

Para calcular o jacobiano, temos

• ∂x

∂r=

1

2cos θ e

∂x

∂θ= −r

2sen θ

• ∂y

∂r= sen θ e

∂y

∂θ= r cos θ

⇒ ∂(x, y)

∂(r, θ)=

12 cos θ − r

2sen θ

sen θ r cos θ

=r

2cos2 θ +

r

2sen2 θ =

r

2

Logo,∫∫

B

sen (4x2 + y2) dxdy =

∫ π

0

∫ 1

0

r

2sen (r2) drdθ

=

∫ π

0

[

−1

4cos r2

]1

0

dθ =

∫ π

0

[

−1

4cos 1 +

1

4

]

dθ =π

4(1 − cos 1)

150

141. Calcule

∫∫

B

ex2+y2

dxdy onde B e a regiao dada por B = {∀(x, y) ∈ R3 | 1 ≤ x2 +

y2 ≤ 4}

Solucao . Tome

x = cos θ

y = rsen θ⇒ {1 ≤ r ≤ 2 e 0 ≤ θ ≤ 2π

Logo, temos

∫ 2π

0

∫ 2

1

1

4er2 · r drdθ

=

∫ 2π

0

[1

8er2

]2

1

=

∫ 2π

0

[1

8e4 − 1

8e

]

=

[1

8e4θ − 1

8eθ

]2π

0

4(e4 − e)

151

142. Encontre o volume do conjunto: {(x, y, z) ∈ R3 | 0 ≤ x ≤ 1, 0 ≤ y ≤ 1 e 0 ≤ z ≤xyex2−y2}

Solucao .

V =

∫ 1

0

∫ 1

0

xyex2−y2

dydx

=

∫ 1

0

[

−x

2ex2−y2

]1

0dx

=1

2

∫ 1

0

x[

−ex2−1 + e2]

dx

=1

4

[

−ex2−1 + e2]1

0

= −1

4+

1

4e−1 +

1

4e − 1

4

=1

4(e − 2 + e−1)

=1

4(e − 1)(1 − e−1)

152

143. Calcule o volume do solido limitado pelo grafico de f(x, y) = xy√

x2 + y2 e B e

a regiao dada pelo retangulo 0 ≤ x ≤ 1, 0 ≤ y ≤ 1

Solucao .

V =

∫ 1

0

∫ 1

0

xy√

x2 + y2 dydx

=1

3

∫ 1

0

[

x√

x2 + y2]1

0dx

=1

3

∫ 1

0

[

x√

(x2 + 1)3 − x3]

dx

=1

3

∫ 1

0

x√

(x2 + 1)3 dx − 1

3

∫ 1

0

x4 dx

=1

15

[√

(x2 + 1)5 − x5]1

0

=1

15(4√

2 − 2)

=2

15(2√

2 − 1)

153

144. Calcule o volume do solido limitado pelo grafico de f(x, y) = y3exy2

e B e o

retangulo 0 ≤ x ≤ 1, 1 ≤ y ≤ 2

Solucao .

V =

∫ 1

0

∫ 2

1

y3exy2

dxdy =

∫ 2

1

[y3

y2exy2

]1

0

dy

=

∫ 2

1

[

yexy2]1

0dy =

∫ 2

1

(yey2 − y) dy

=1

2

[

ey2 − y2]2

1=

1

2(e4 − e − 3)

154

145. Calcule o volume do solido limitado pelo grafico de f(x, y) =y

x + y2e B e a

regiao dada pelo retangulo 1 ≤ x ≤ 4, 0 ≤ y ≤ √x

Solucao .

V =

∫ 4

1

∫ √x

0

y

x + y2dydx

Tome u = y2 → du = 2ydy

y = 0; u = 0

y =√

x; u = xTemos, entao

∫ 4

1

∫ x

0

1

2(x + u)dudx =

1

2

∫ 4

1

ln(x + u)|x0dx

=1

2

∫ 4

1

(ln(2x) − ln x) dx =1

2

∫ 4

1

ln2x

xdx

=1

2

∫ 4

1

ln 2 dx =1

2ln 2 · x|41

=3

2ln 2

155

146. Calcule o volume do conjunto dado: x2 + y2 + 3 ≤ z ≤ 4

Solucao . Fazendo x2 + y2 + 3 = c e tornando

• c = 3

temos x2 + y2 = 0 entao x = 0 e y = 0

• c = 4

temos x2 + y2 = 1

Mudando para coordenadas polares, temos

∫ 2π

0

∫ 1

0

[4 − (r2 cos2 θ + r2sen2 θ + 3)]rdrdθ

=

∫ 2π

0

∫ 1

0

[4 − r2 − 3)]rdrdθ =

∫ 2π

0

∫ 1

0

(r − r3)drdθ

=

∫ 2π

0

[r2

2− r4

4

]1

0

dθ =

∫ 2π

0

(1

2− 1

4

)

=

4

]2π

0

2

156

147. Calcule o volume do solido limitado pelo grafico de f(x, y) = xy√

x2 + y2 e B e

a regiao dada pelo retangulo 0 ≤ x ≤ 1, 0 ≤ y ≤ 1

Solucao .

V =

∫ 1

0

∫ 1

0

xy√

x2 + y2 dydx

=1

3

∫ 1

0

[

x√

x2 + y2]1

0dx

=1

3

∫ 1

0

[

x√

(x2 + 1)3 − x3]

dx

=1

3

∫ 1

0

x√

(x2 + 1)3 dx − 1

3

∫ 1

0

x4 dx

=1

15

[√

(x2 + 1)5 − x5]1

0

=1

15(4√

2 − 2)

=2

15(2√

2 − 1)

157

148. Calcule o volume do solido limitado pelo grafico de f(x, y) = y3exy2

e B e o

retangulo 0 ≤ x ≤ 1, 1 ≤ y ≤ 2

Solucao .

V =

∫ 1

0

∫ 2

1

y3exy2

dxdy =

∫ 2

1

[y3

y2exy2

]1

0

dy

=

∫ 2

1

[

yexy2]1

0dy =

∫ 2

1

(yey2 − y) dy

=1

2

[

ey2 − y2]2

1=

1

2(e4 − e − 3)

158

149. Calcule o volume do solido limitado pelo grafico de f(x, y) =y

x + y2e B e a

regiao dada pelo retangulo 1 ≤ x ≤ 4, 0 ≤ y ≤ √x

Solucao .

V =

∫ 4

1

∫ √x

0

y

x + y2dydx

Tome u = y2 → du = 2ydy

y = 0; u = 0

y =√

x; u = xTemos, entao

∫ 4

1

∫ x

0

1

2(x + u)dudx =

1

2

∫ 4

1

ln(x + u)|x0dx

=1

2

∫ 4

1

(ln(2x) − ln x) dx =1

2

∫ 4

1

ln2x

xdx

=1

2

∫ 4

1

ln 2 dx =1

2ln 2 · x|41

=3

2ln 2

159

150. Calcule o volume V do conjunto dado: x2 + 4y2 ≤ 4 e x + y ≤ z ≤ x + y + 1

Solucao . Mudando para coordenadas polares, temos

V =1

2

∫ 2π

0

∫ 2

0

[r cos θ + rsen θ + 1 − (r cos θ + rsen θ)]rdrdθ

=1

2

∫ 2π

0

∫ 2

0

rdrd =1

4

∫ 2π

0

[r2]2

0dθ =

∫ 2π

0

dθ = 2π

160

151. Calcule o volume V do conjunto dado: x2 + y2 ≤ a2 e y2 + z2 ≤ a2 (a > 0)

Resolucao : Como z =√

a2 − y2

Temos, entao

V = 8

∫ a

0

∫√

a2−y2

0

a2 − y2 dxdy

= 8

∫ a

0

a2 − y2 · [x]

√a2−y2

0 dy

= 8

∫ a

0

(a2 − y2) dy

= 8

[

a2y − y3

3

]a

0

= 8

[

a3 − a3

3

]

=16

3a3

161

152. Calcule o volume V do conjunto dado: x2 + y2 ≤ z ≤ 1 − x2

Solucao . Temos que a intercecao entre os graficos e x2 + y2 = 1− x2 ⇒ 2x2 + y2 = 1 e

uma elipse de raio 1. Mudando para coordenadas polares, temos

V =

√2

2

∫ 2π

0

∫ 1

0

[1 − r2(2 cos2 +sen2 θ)]rdrdθ

=

√2

2

∫ 2π

0

∫ 1

0

[1 − r2(cos2 + 1)]rdrdθ

=

√2

2

∫ 2π

0

∫ 1

0

[r − r3 cos2 −r3 θ)]drdθ

=

√2

2

∫ 2π

0

[r2

2− r4

4cos2 −r4

4

]1

0

=

√2

2

∫ 2π

0

[1

2− 1

4cos2 −1

4

]

=

√2

8

∫ 2π

0

(1 − cos2 θ) dθ

=

√2

8

∫ 2π

0

[

θ −(

θ

2+

1

4sen2 θ

)]2π

0

=

√2π

8

162

153. Calcule o volume V do conjunto dado: x + y + z ≤ 1, x ≥ 0, y ≥ 0 e z ≥ 0

Solucao .

V =

∫ 1

0

∫ 1−x

0

(1 − x − y) dydx

=

∫ 1

0

[

y − xy − y2

2

]1−x

0

dx

=

∫ 1

0

[

1 − 2x + x2 − 1 − 2x + x2

2

]

dx

=

[

x − x2 +x3

3− 1

2

(

x − x2 +x3

3

)]1

0

=1

3− 1

6

=1

6

163

154. Calcule a area limitada pela equacaox2

a2+

y2

b2= 1

Solucao . Tome

x = ar cos θ

y = brsen θ

Calculando o jacobiano, temos

∂(x, y)

∂(r, θ)=

a cos θ −arsen θ

bsen θ br cos θ

= abr

agora substituindo na equacao da elipse, temos:

a2r2 cos2 θ

a2+

b2r2sen2 θ

b2= 1

Entao 0 ≤ r ≤ 1 e 0 ≤ θ ≤ 2π. Assim,

AreaB =

∫ 2π

0

∫ 1

0

abr drdθ =

∫ 2π

0

[

abr2

2

]1

0

=1

2

∫ 2π

0

ab dθ =1

2[ab]

2π0 = abπ

164

155. Calcule o centro massa do solido cuja densidade e dada pela funcao δ(x, y) = y

e B e a regiao dada pelo quadrado 0 ≤ x ≤ 1, 0 ≤ y ≤ 1.

Solucao . Onde

massaB =

∫∫

B

y dxdy

=

∫ 1

0

∫ 1

0

y dydx =

∫ 1

0

[y2

2

]1

0

dx =[x

2

]1

0=

1

2

O centro de massa e o ponto (xc, yc) onde

xc =

∫∫

Bx dm

massaB=

∫∫

Bxy dxdy

massaB

yc =

∫∫

By dm

massaB=

∫∫

By2 dxdy

massaB

Temos, entao∫∫

B

xy dxdy =

∫ 1

0

∫ 1

0

xy dydx =

∫ 1

0

[

xy2

2

]1

0

dx

=1

4

[x2]1

0=

1

4

Assim

xc =1412

=1

2

De modo analogo, temos

yc =1312

=2

3

Logo, o centro de massa fica no ponto ( 12 , 2

3 ).

165

156. Calcule o centro de massa onde a regiao e dada por B = {(x, y) ∈ R2 |x2 +4y2 ≤1, y ≥ 0} e a densidade e proporcional a do ponto ao eixo x.

Solucao . Temos que δ(x, y) = ky; k ∈ R seja a funcao densidade

Tome

x = r cos θ

y = 12rsen θ

Calculando o jacobiano, temos

∂(x, y)

∂(r, θ)=

cos θ −rsen θ

12sen θ 1

2r cos θ

=r

2

e a regiao de integracao e

r2 cos2 θ +4

4r2sen2 θ ≤ 1

assim, 0 ≤ r ≤ 1, 0 ≤ θ ≤ π,

Entao

massaB =

∫ π

0

∫ 1

0

k

4r2sen θ drdθ =

∫ π

0

[r3

3sen θ

]1

0

=

[

−k

6cos θ

0

=k

3

Tambem,∫ π

0

∫ 1

0

k

4r3sen θ cos θ drdθ =

k

16

∫ π

0

sen θ cos θ dθ

=

[k

32sen2 θ

0

= 0

e temos tambem∫ π

0

∫ 1

0

k

4r3sen2 θ drdθ =

k

16

∫ π

0

sen2 θ drdθ

=k

16

2− 1

4sen2 θ

0

=kπ

32

166

Entao

yc =kπ32k3

=3π

32

Logo, o centro de massa e no ponto (xc, yc) = (0, 3π32 ).

167

157. Calcule

∫∫∫

B

xyzdxdydz;onde a regiao dada e B = {(x, y, z) ∈ R3 |x ≤ 2, 0 ≤ y ≤1 e 1 ≤ z ≤ 2}.

Solucao . Temos, entao

∫ 2

0

∫ 1

0

∫ 2

1

xzy dzdydx

=

∫ 2

0

∫ 2

1

[

xyz2

2

]2

1

dydx

=

∫ 2

0

∫ 1

0

xy

(4 − 1

2

)

dydx

=3

2

∫ 2

0

[

xy2

2

]1

0

dx

=3

4

∫ 2

0

x dx

=

[3

4· x2

2

]2

0

=3

2

168

158. Calcule

∫∫∫

B

xdxdydz; Onde 0 ≤ x ≤ 1, 0 ≤ y ≤ 1 e x + y ≤ z ≤ x + y + 1.

Solucao . Temos, entao

∫ 1

0

∫ 1

0

∫ x+y

x+y+1

x dzdydx =

∫ 1

0

∫ 1

0

[xz]x+y+1x+y dydx

=

∫ 1

0

∫ 1

0

x(x + y + 1 − x − y) dydx =

∫ 1

0

[xy]10 dx =

[x2

2

]1

0

=1

2

169

159. Calcule

∫∫∫

B

1 − z2dxdydz onde 0 ≤ x ≤ 1, 0 ≤ y ≤ z e 0 ≤ z ≤ 1.

Solucao . Temos, entao

∫ 1

0

∫ 1

0

∫ z

0

x dydzdx =

∫ 1

0

∫ 1

0

z√

1 − z2 dzdx

= −1

3

∫ 1

0

[√

(1 − z2)3]1

0dx =

1

3

∫ 1

0

dx =[x

3

]1

0=

1

3

170

160. Calcule

∫∫∫

B

1 − z2dxdydz; Onde 0 ≤ x ≤ 1, 0 ≤ y ≤ 1 e 0 ≤ z ≤ 1.

Solucao . Temos entao∫ 1

0

∫ 1

0

∫ 1

0

1 − z2 dzdydx

Tomemos z = sen θ → dz = cos θ e 0 ≤ θ ≤ π

Entao :

∫ 1

0

∫ 1

0

∫ 1

0

1 − sen2 x dzdydx

=

∫ 1

0

∫ 1

0

∫ 1

0

cos θ

2dθdydx

=1

2

∫ 1

0

∫ 1

0

2+

1

4sen 2θ

0

dydx

=1

2

∫ 1

0

∫ 1

0

π

2dydx =

1

2

∫ 1

0

π

2dx

4

171

161. Calcule

∫∫∫

B

dxdydz onde a regiao dada e x2 + y2 ≤ z ≤ 2x

Solucao . Entao temos:

∫∫

K

∫ 2x

x2+y2

dzdydx =

∫∫

K

[2x − (x2 + y2)] dydx

Mudando para coordenadas polares

x = r cos θ + 1

y = rsen θ

⇒ {0 ≤ r ≤ 1 e 0 ≤ θ ≤ 2π}

Temos, entao

∫ 2π

0

∫ 1

0

[2(r cos θ + 1) − r2] rdrdθ =

∫ 2π

0

∫ 1

0

(2r2 cos θ + r − r3) drdθ

=

∫ 2π

0

[2

3r3 cos θ +

r2

2− r4

4

]1

0

=

∫ 2π

0

(2

3cos θ +

1

4

)

=2

3

[

sen θ +θ

4

]2π

0

2

172

162. Calcule

∫∫∫

B

(x2 + z2) dxdydz ondeB e a regiao x2 + z2 ≤ 1, 0 ≤ z ≤ 1

Solucao . Entao temos

∫∫

K

∫ 1

0

(x2 + z2) dzdydx =

∫∫

K

[

x2z +z3

3

]1

0

dydx

=

∫∫

K

(

x2 +1

3

)

dydx

Mudando para coordenadas polares, temos

∫ 2π

0

∫ 1

0

[

(r cos θ)2 +1

3

]

rdrdθ =

∫ 2π

0

∫ 1

0

[

r3 cos2 θ +r

3

]

drdθ

=

∫ 1

0

[r4

4cos2 θ +

r2

6

]1

0

dθ =

∫ 1

0

[1

4cos2 θ

]

=1

4

2+

1

4sen2 θ +

1

6

]2π

0

dθ =7π

12

173

163. Calcule

∫∫∫

B

dxdydz; Onde B e a regiao x2 + y2 ≤ z ≤ 2x + 2y − 1

Solucao . Calculando a intercecao , temos que x2+y2 ≤ 2x+2y−1 ⇒ (x−1)2+(y−1) ≤ 1

Entao∫∫

K

∫ 2x+2y−1

x2+y2

dzdydx =

∫∫

K

[2x + 2y − 1 − (x2 + y2)] dydx

Mudando para coordenadas polares, temos

x = r cos θ + 1

y = rsen θ + 1⇒ {0 ≤ r ≤ 1, 0 ≤ θ ≤ 2π}

Segue agora

∫ 2π

0

∫ 1

0

[2r cos θ + 2rsen θ + 3 − r2] rdrdθ =

∫ 2π

0

∫ 1

0

[2r2(cos θ + sen θ) + 3r − r3] drdθ

=

∫ 2π

0

[2r3

3(cos θ + sen θ) +

5

4

]

dθ =

[2

3(sen θ − cos θ) +

4

]2π

0

=5π

2

174

164. Calcule

∫∫∫

B

dxdydz onde B e a regiao dada por x2 + y2 ≤ 1, x2 + y2 + z2 ≤ 4

Solucao . Tem-se que z ≤√

4 − (x2 + y2)

Entao∫∫

K

∫√

4−(x2+y2)

0

2z dzdydx =

∫∫

K

2

[4 − (x2 + y2)

2

]

dydx

Mudando para coordenadas polares, temos

∫ 2π

0

∫ 1

0

[4 − r2] rdrdθ =

∫ 2π

0

[

2r2 − r4

4

]1

0

=

[7θ

4

]2π

0

=7π

4

175

165. Calcule

∫∫∫

B

xdxdydz onde B e a regiao dada por x2 ≤ y ≤ x, 0 ≤ z ≤ x + y

Solucao . Temos∫∫

K

∫ x+y

0

x dzdydx =

∫∫

K

x(x + y) dydx

=

∫ 1

0

∫ x

x2

(x2 + xy) dydx =

∫ 1

0

[

x2y + xy2

2

]x

x2

dx

=

∫ 1

0

[

x3 +x3

2− x4 − x5

2

]

dx =

[x4

4+

x4

8− x5

5− x6

12

]1

0

=11

120

176

166. Calcule

∫∫∫

B

2zdxdydz onde B e a regiao dada por 4x2 + 9y2 + z2 ≤ 4 e z ≥ 0

Solucao . Tem-se que z ≤√

4 − (4x2 + 9y2)

Entao ,∫∫

K

∫√

4−(4x2+9y2)

0

2z dzdydx

=

∫∫

K

[z2]√

4−(4x2+9y2)

0dydx =

∫∫

K

[4 − (4x2 + 9y2)] dydx

Onde K = 4x2 + 9y2 ≤ 4.

Tomemos

2x = r cos θ

3y = rsen θ

x = r2 cos θ

y = r3sen θ

Assim,

4

(r2

4cos2 θ

)

+ 9

(r2

9sen2 θ

)

≤ 4

⇒ 0 ≤ r ≤ 2, 0 ≤ θ ≤ 2π

Temos, entao∫ 2π

0

∫ 2

0

[4 − r2]r

6drdθ =

1

6

∫ 2π

0

[

2r2 − r4

4

]2

0

=2

3

∫ 2π

0

dθ =2

3[θ]

2π0 =

3

177

167. Calcule o volume do solido sob o grafico de f(x, y) = x2 + y2, limitado por

B = [0, 1] × [0, 1]

Solucao .

Observe que:

f(x, y) = x2 + y2 > 0 ∀(x, y) ∈ B

Temos que o volume e dado pela expressao :

v =

∫ ∫

B

f(x, y)dxdy

Logo, o volume sera:∫ 1

0

∫ 1

0

(x2 + y2)dxdy

=

∫ 1

0

[x3

3+ xy2

]1

0

dy

=

∫ 1

0

(1

3+ y2

)

dy

=

[1

3y +

y3

3

]1

0

=2

3

178

168. Calcular o volume do solido limitado pela regiao 1 ≤ Z ≤√

4 − x2 − y2

Solucao .

V =

∫ ∫

k

[∫√

4−x2−y2

1

dz

]

dxdy

=

∫ ∫

k

(√

4 − x2 − y2 − 1)

dxdy

Temos que a fronteira k e definida por:

1 =√

4 − x2 − y2

1 = 4 − x2 − y2

x2 + y2 = 3

Isto e,

k : x2 + y2 ≤ 3

Agora, utilizando coordenadas polares, temos:

x = ρ cos θ

y = ρ sin θ

Logo, o volume do solido sera :

V =

∫ 2π

0

∫ √3

0

[√

4 − ρ2 − 1]

ρdρdθ

=

∫ 2π

0

∫ √3

0

[√

4 − ρ2ρ − ρ]

= 2π

[(−1

3

)

(4 − ρ2)32 − ρ2

2

]√

3

0

[(−1

3+

1

3(8)

)

− 3

2

]

5

179

169. Calcule∫ ∫

B(x2 + y2)dxdy onde B =

{(x, y) ∈ <2|1 ≤ x2 + y2 ≤ 4

}.

Solucao .

Utilizando coordenadas polares, tomemos:

x = ρ cos θ

y = ρ sin θ

E observe que os limites de integracao serao

0 ≤ θ ≤ 2π

1 ≤ ρ ≤√

4

Dessa forma temos que:∫ ∫

B

x2 + y2dxdy

=

∫ ∫

B(θρ)

(ρ2 cos2 θ + ρ2 sin2 θ)ρdρdθ

=

∫ √4

1

∫ 2π

0

ρ3dθdρ

∫ √4

1

[ρ3θ]2π

0dρ

=

∫ √4

1

2πρ3dρ

=

[2π

4ρ4

]√

4

1

=2π

416 − 2π

4

=15π

2

180

170. Calcule o volume V do conjunto dado: x2 + y2 ≤ z ≤ 1 − x2

Resolucao : Temos que a intercecao entre os graficos e x2 + y2 = 1−x2 ⇒ 2x2 + y2 = 1

e uma elipse de raio 1. Mudando para coordenadas polares, temos

V =

√2

2

∫ 2π

0

∫ 1

0

[1 − r2(2 cos2 +sen2 θ)]rdrdθ

=

√2

2

∫ 2π

0

∫ 1

0

[1 − r2(cos2 + 1)]rdrdθ

=

√2

2

∫ 2π

0

∫ 1

0

[r − r3 cos2 −r3 θ)]drdθ

=

√2

2

∫ 2π

0

[r2

2− r4

4cos2 −r4

4

]1

0

=

√2

2

∫ 2π

0

[1

2− 1

4cos2 −1

4

]

=

√2

8

∫ 2π

0

(1 − cos2 θ) dθ

=

√2

8

∫ 2π

0

[

θ −(

θ

2+

1

4sen2 θ

)]2π

0

=

√2π

8

181

171. Calcule o volume V do conjunto dado: x + y + z ≤ 1, x ≥ 0, y ≥ 0 e z ≥ 0

Resolucao :

V =

∫ 1

0

∫ 1−x

0

(1 − x − y) dydx

=

∫ 1

0

[

y − xy − y2

2

]1−x

0

dx

=

∫ 1

0

[

1 − 2x + x2 − 1 − 2x + x2

2

]

dx

=

[

x − x2 +x3

3− 1

2

(

x − x2 +x3

3

)]1

0

=1

3− 1

6

=1

6

182

172. Calcule a area limitada pela equacaox2

a2+

y2

b2= 1

Resolucao : Tome

x = ar cos θ

y = brsen θ

Calculando o jacobiano, temos

∂(x, y)

∂(r, θ)=

a cos θ −arsen θ

bsen θ br cos θ

= abr

agora substituindo na equacao da elipse, temos:

a2r2 cos2 θ

a2+

b2r2sen2 θ

b2= 1

Entao 0 ≤ r ≤ 1 e 0 ≤ θ ≤ 2π. Assim,

AreaB =

∫ 2π

0

∫ 1

0

abr drdθ =

∫ 2π

0

[

abr2

2

]1

0

=1

2

∫ 2π

0

ab dθ =1

2[ab]

2π0 = abπ

183

173. Calcule o centro massa do solido cuja densidade e dada pela funcao δ(x, y) = y

e B e a regiao dada pelo quadrado 0 ≤ x ≤ 1, 0 ≤ y ≤ 1.

Solucao . Onde

massaB =

∫∫

B

y dxdy

=

∫ 1

0

∫ 1

0

y dydx =

∫ 1

0

[y2

2

]1

0

dx =[x

2

]1

0=

1

2

O centro de massa e o ponto (xc, yc) onde

xc =

∫∫

Bx dm

massaB=

∫∫

Bxy dxdy

massaB

yc =

∫∫

By dm

massaB=

∫∫

By2 dxdy

massaB

Temos, entao∫∫

B

xy dxdy =

∫ 1

0

∫ 1

0

xy dydx =

∫ 1

0

[

xy2

2

]1

0

dx

=1

4

[x2]1

0=

1

4

Assim

xc =1412

=1

2

De modo analogo, temos

yc =1312

=2

3

Logo, o centro de massa fica no ponto ( 12 , 2

3 ).

184

174. Calcule o centro de massa onde a regiao e dada por B = {(x, y) ∈ R2 |x2 +4y2 ≤1, y ≥ 0} e a densidade e proporcional a do ponto ao eixo x.

Solucao . Temos que δ(x, y) = ky; k ∈ R seja a funcao densidade

Tome

x = r cos θ

y = 12rsen θ

Calculando o jacobiano, temos

∂(x, y)

∂(r, θ)=

cos θ −rsen θ

12sen θ 1

2r cos θ

=r

2

e a regiao de integracao e

r2 cos2 θ +4

4r2sen2 θ ≤ 1

assim, 0 ≤ r ≤ 1, 0 ≤ θ ≤ π,

Entao

massaB =

∫ π

0

∫ 1

0

k

4r2sen θ drdθ =

∫ π

0

[r3

3sen θ

]1

0

=

[

−k

6cos θ

0

=k

3

Tambem,∫ π

0

∫ 1

0

k

4r3sen θ cos θ drdθ =

k

16

∫ π

0

sen θ cos θ dθ

=

[k

32sen2 θ

0

= 0

e temos tambem∫ π

0

∫ 1

0

k

4r3sen2 θ drdθ =

k

16

∫ π

0

sen2 θ drdθ

=k

16

2− 1

4sen2 θ

0

=kπ

32

185

Entao

yc =kπ32k3

=3π

32

Logo, o centro de massa e no ponto (xc, yc) = (0, 3π32 ).

186

175. Calcule

∫∫∫

B

xyzdxdydz;onde a regiao dada e B = {(x, y, z) ∈ R3 |x ≤ 2, 0 ≤ y ≤1 e 1 ≤ z ≤ 2}.

Solucao . Temos, entao

∫ 2

0

∫ 1

0

∫ 2

1

xzy dzdydx

=

∫ 2

0

∫ 2

1

[

xyz2

2

]2

1

dydx

=

∫ 2

0

∫ 1

0

xy

(4 − 1

2

)

dydx

=3

2

∫ 2

0

[

xy2

2

]1

0

dx

=3

4

∫ 2

0

x dx

=

[3

4· x2

2

]2

0

=3

2

187

176. Calcule

∫∫∫

B

xdxdydz; Onde 0 ≤ x ≤ 1, 0 ≤ y ≤ 1 e x + y ≤ z ≤ x + y + 1.

Solucao . Temos, entao

∫ 1

0

∫ 1

0

∫ x+y

x+y+1

x dzdydx =

∫ 1

0

∫ 1

0

[xz]x+y+1x+y dydx

=

∫ 1

0

∫ 1

0

x(x + y + 1 − x − y) dydx =

∫ 1

0

[xy]10 dx =

[x2

2

]1

0

=1

2

188

177. Calcule

∫∫∫

B

1 − z2dxdydz onde 0 ≤ x ≤ 1, 0 ≤ y ≤ z e 0 ≤ z ≤ 1.

Solucao . Temos, entao

∫ 1

0

∫ 1

0

∫ z

0

x dydzdx =

∫ 1

0

∫ 1

0

z√

1 − z2 dzdx

= −1

3

∫ 1

0

[√

(1 − z2)3]1

0dx =

1

3

∫ 1

0

dx =[x

3

]1

0=

1

3

189

178. Calcule

∫∫∫

B

1 − z2dxdydz; Onde 0 ≤ x ≤ 1, 0 ≤ y ≤ 1 e 0 ≤ z ≤ 1.

Solucao . Temos entao∫ 1

0

∫ 1

0

∫ 1

0

1 − z2 dzdydx

Tomemos z = sen θ → dz = cos θ e 0 ≤ θ ≤ π

Entao :

∫ 1

0

∫ 1

0

∫ 1

0

1 − sen2 x dzdydx

=

∫ 1

0

∫ 1

0

∫ 1

0

cos θ

2dθdydx

=1

2

∫ 1

0

∫ 1

0

2+

1

4sen 2θ

0

dydx

=1

2

∫ 1

0

∫ 1

0

π

2dydx =

1

2

∫ 1

0

π

2dx

4

190

179. Calcule

∫∫∫

B

dxdydz onde a regiao dada e x2 + y2 ≤ z ≤ 2x

Solucao . Entao temos:

∫∫

K

∫ 2x

x2+y2

dzdydx =

∫∫

K

[2x − (x2 + y2)] dydx

Mudando para coordenadas polares

x = r cos θ + 1

y = rsen θ

⇒ {0 ≤ r ≤ 1 e 0 ≤ θ ≤ 2π}

Temos, entao

∫ 2π

0

∫ 1

0

[2(r cos θ + 1) − r2] rdrdθ =

∫ 2π

0

∫ 1

0

(2r2 cos θ + r − r3) drdθ

=

∫ 2π

0

[2

3r3 cos θ +

r2

2− r4

4

]1

0

=

∫ 2π

0

(2

3cos θ +

1

4

)

=2

3

[

sen θ +θ

4

]2π

0

2

191

180. Calcule

∫∫∫

B

(x2 + z2) dxdydz ondeB e a regiao x2 + z2 ≤ 1, 0 ≤ z ≤ 1

Solucao . Entao temos

∫∫

K

∫ 1

0

(x2 + z2) dzdydx =

∫∫

K

[

x2z +z3

3

]1

0

dydx

=

∫∫

K

(

x2 +1

3

)

dydx

Mudando para coordenadas polares, temos

∫ 2π

0

∫ 1

0

[

(r cos θ)2 +1

3

]

rdrdθ =

∫ 2π

0

∫ 1

0

[

r3 cos2 θ +r

3

]

drdθ

=

∫ 1

0

[r4

4cos2 θ +

r2

6

]1

0

dθ =

∫ 1

0

[1

4cos2 θ

]

=1

4

2+

1

4sen2 θ +

1

6

]2π

0

dθ =7π

12

192

181. Calcule

∫∫∫

B

dxdydz; Onde B e a regiao x2 + y2 ≤ z ≤ 2x + 2y − 1

Solucao . Calculando a intercecao , temos que x2+y2 ≤ 2x+2y−1 ⇒ (x−1)2+(y−1) ≤ 1

Entao∫∫

K

∫ 2x+2y−1

x2+y2

dzdydx =

∫∫

K

[2x + 2y − 1 − (x2 + y2)] dydx

Mudando para coordenadas polares, temos

x = r cos θ + 1

y = rsen θ + 1⇒ {0 ≤ r ≤ 1, 0 ≤ θ ≤ 2π}

Segue agora

∫ 2π

0

∫ 1

0

[2r cos θ + 2rsen θ + 3 − r2] rdrdθ =

∫ 2π

0

∫ 1

0

[2r2(cos θ + sen θ) + 3r − r3] drdθ

=

∫ 2π

0

[2r3

3(cos θ + sen θ) +

5

4

]

dθ =

[2

3(sen θ − cos θ) +

4

]2π

0

=5π

2

193

182. Calcule

∫∫∫

B

dxdydz onde B e a regiao dada por x2 + y2 ≤ 1, x2 + y2 + z2 ≤ 4

Solucao . Tem-se que z ≤√

4 − (x2 + y2)

Entao∫∫

K

∫√

4−(x2+y2)

0

2z dzdydx =

∫∫

K

2

[4 − (x2 + y2)

2

]

dydx

Mudando para coordenadas polares, temos

∫ 2π

0

∫ 1

0

[4 − r2] rdrdθ =

∫ 2π

0

[

2r2 − r4

4

]1

0

=

[7θ

4

]2π

0

=7π

4

194

183. Calcule

∫∫∫

B

xdxdydz onde B e a regiao dada por x2 ≤ y ≤ x, 0 ≤ z ≤ x + y

Solucao . Temos∫∫

K

∫ x+y

0

x dzdydx =

∫∫

K

x(x + y) dydx

=

∫ 1

0

∫ x

x2

(x2 + xy) dydx =

∫ 1

0

[

x2y + xy2

2

]x

x2

dx

=

∫ 1

0

[

x3 +x3

2− x4 − x5

2

]

dx =

[x4

4+

x4

8− x5

5− x6

12

]1

0

=11

120

195

184. Calcule

∫∫∫

B

2zdxdydz onde B e a regiao dada por 4x2 + 9y2 + z2 ≤ 4 e z ≥ 0

Solucao . Tem-se que z ≤√

4 − (4x2 + 9y2)

Entao ,∫∫

K

∫√

4−(4x2+9y2)

0

2z dzdydx

=

∫∫

K

[z2]√

4−(4x2+9y2)

0dydx =

∫∫

K

[4 − (4x2 + 9y2)] dydx

Onde K = 4x2 + 9y2 ≤ 4.

Tomemos

2x = r cos θ

3y = rsen θ

x = r2 cos θ

y = r3sen θ

Assim,

4

(r2

4cos2 θ

)

+ 9

(r2

9sen2 θ

)

≤ 4

⇒ 0 ≤ r ≤ 2, 0 ≤ θ ≤ 2π

Temos, entao∫ 2π

0

∫ 2

0

[4 − r2]r

6drdθ =

1

6

∫ 2π

0

[

2r2 − r4

4

]2

0

=2

3

∫ 2π

0

dθ =2

3[θ]

2π0 =

3

196

185. Calcule o volume do solido sob o grafico de f(x, y) = x2 + y2, limitado por

B = [0, 1] × [0, 1]

Solucao .

Observe que:

f(x, y) = x2 + y2 > 0 ∀(x, y) ∈ B

Temos que o volume e dado pela expressao :

v =

∫ ∫

B

f(x, y)dxdy

Logo, o volume sera:∫ 1

0

∫ 1

0

(x2 + y2)dxdy

=

∫ 1

0

[x3

3+ xy2

]1

0

dy

=

∫ 1

0

(1

3+ y2

)

dy

=

[1

3y +

y3

3

]1

0

=2

3

197

186. Calcular o volume do solido limitado pela regiao 1 ≤ z ≤√

4 − x2 − y2

Solucao .

V =

∫ ∫

k

[∫√

4−x2−y2

1

dz

]

dxdy

=

∫ ∫

k

(√

4 − x2 − y2 − 1)

dxdy

Temos que a fronteira k e definida por:

1 =√

4 − x2 − y2

1 = 4 − x2 − y2

x2 + y2 = 3

Isto e,

k : x2 + y2 ≤ 3

Agora, utilizando coordenadas polares, temos:

x = ρ cos θ

y = ρ sin θ

Logo, o volume do solido sera :

V =

∫ 2π

0

∫ √3

0

[√

4 − ρ2 − 1]

ρdρdθ

=

∫ 2π

0

∫ √3

0

[√

4 − ρ2ρ − ρ]

= 2π

[(−1

3

)

(4 − ρ2)32 − ρ2

2

]√

3

0

[(−1

3+

1

3(8)

)

− 3

2

]

5

198

187. Calcule∫ ∫

B(x2 + y2)dxdy onde B =

{(x, y) ∈ <2|1 ≤ x2 + y2 ≤ 4

}.

Solucao .

Utilizando coordenadas polares, tomemos:

x = ρ cos θ

y = ρ sin θ

E observe que os limites de integracao serao

0 ≤ θ ≤ 2π

1 ≤ ρ ≤√

4

Dessa forma temos que:∫ ∫

B

x2 + y2dxdy

=

∫ ∫

B(θρ)

(ρ2 cos2 θ + ρ2 sin2 θ)ρdρdθ

=

∫ √4

1

∫ 2π

0

ρ3dθdρ

∫ √4

1

[ρ3θ]2π

0dρ

=

∫ √4

1

2πρ3dρ

=

[2π

4ρ4

]√

4

1

=2π

416 − 2π

4

=15π

2

199

188. Calcule a integral tripla∫ ∫ ∫

Bxyz2dV , onde B = [0, 1] × [−1, 2] × [0, 3].

Solucao .

Observando os limites de integracao temos que:

∫ ∫ ∫

B

xyz2dV

=

∫ 3

0

∫ 2

−1

∫ 1

0

xyz2dxdydz

Integrando em relacao a x temos

∫ 3

0

∫ 2

−1

[x2yz2

2

]1

0

dydz

∫ 3

0

∫ 2

−1

(yz2

2

)

dydz

Em relacao a y temos∫ 3

0

[y2z2

4

]2

−1

dz

∫ 3

0

(

z2 − z2

4

)

dz

Por ultimo, integrando em relacao a z temos que

[

z2 − z2

4

]3

0

(

9 − 27

12

)

27

4

Portanto,∫ 3

0

∫ 2

−1

∫ 1

0

xyz2dxdydz =27

4

200

189. Calcule∫ ∫ ∫

Be√

x2+y2+z2dV , onde B e a bola unitaria B =

{(x, y, z) : x2 + y2 + z2 ≤ 1

}.

Solucao .

Note que√

x2 + y2 + z2 = ρ

e que os limites de integracao serao :

0 ≤ Φ ≤ π

0 ≤ θ ≤ 2π

0 ≤ ρ ≤ 1

Logo temos que

∫ ∫ ∫

B

e√

x2+y2+z2dV =

∫ π

0

∫ 2π

0

∫ 1

0

eρρ2 sin(φ)dρdθdφ

Fazendo uma integrancao por partes de∫

eρρ2dρ temos:

eρρ2 − 2

eρρdρ

= eρρ2 − 2eρρ + 2eρ∣∣∣

1

0= (e − 2)

Logo, temos

∫ π

0

∫ 2π

0

∫ 1

0

eρρ2 sin(φ)dρdθdφ =

∫ π

0

∫ 2π

0

(e − 2) sin φdθdφ

=

∫ π

0

(e − 2) sin φθ∣∣∣

0dφ

=

∫ π

0

(e − 2)(2π) sin φdφ

= −(2 − e)(2π) cos φ∣∣∣

π

0

= 2(e − 2)(2π)

= 4π(e − 2)

201

190. Seja a aplicacao f : <2 7−→ <2, f(r, θ) = (r cos(θ), r sin(θ)). Essa funcao e C1?

Calcular o seu Jacobiano.

Solucao .

Para uma transformacao T ser do tipo C1 temos que provar que a funcao f tenha todas

as derivadas parciais de primeira ordem contınuas.

Fazendo

u1 = r cos θ

u2 = r sin θ

Temos que as derivadas parciais

{∂u1

∂r= cos θ,

∂u1

∂θ= −r sin θ,

∂u2

∂r= sin θ,

∂u2

∂θ= r cos θ

}

Portanto temos que as derivadas de primeira ordem sao contınuas, logo, temos que o

jacobiano da funcao f e:

J =

∂u1

∂r∂u1

∂θ

∂u2

∂r∂u2

∂θ

J = det

cos θ −r sin θ

sin θ r cos θ

= r cos2 θ + r sin2 θ

= r

Portanto o Jacobiano da funcao f e igual ao raio r.

202

191. Calcule a integral iterada∫ 3

1

∫ 1

0(1 + 4xy)dxdy

Solucao .

∫ 3

1

(x + 2x2y)]|10dy

=

∫ 3

1

(1 + 2y)dy

= [y + y2]|31 = (3 + 9) − (1 + 1) = 10

203

192. Calcule a integral iterada∫ 4

2

∫ 1

−1(x2 + y2)dxdy

Solucao .

∫ 4

2

[x2y +1

3y3]|1−1dx

=

∫ 4

2

[(x2 +1

3) − (−x2 − 1

3)]dx

=

∫ 4

2

(2x2 +2

3)dx

= [2

3x3]42

= (128

3+

8

3) − (

16

3+

4

3) =

116

3

204

193. Calcule a integral iterada∫ 2

1

∫ 1

0(x + y)−2dxdy

Solucao .

∫ 2

1

[−(x + y)1]10dy

=

∫ 2

1

[y−1 − (1 + y)−1]dy

= [lny − ln(1 + y)]21

= ln2 − ln3 − 0 + ln2 = ln4

3

205

194. Calcule a integral iterada∫ ln2

0

∫ ln5

0e2x−ydxdy

Solucao .

(

∫ ln5

0

e2xdx)(

∫ ln2

0

e−ydy)

= [1

2e2x]ln5

0 [−e−y]ln20

= (25

2

−1

2)(−1

2+ 1) = 6

206

195. Calcule a integral dupla∫ ∫

R(6x2y3 − 5y4)dA,R = (x, y)|0 ≤ x ≤ 3, 0 ≤ y ≤ 1

Solucao .

∫ 3

0

∫ 1

0

(6x2y3 − 5y4)dydx

=

∫ 3

0

[3

2x2y4 − y5]10dx

=

∫ 3

0

(3

2x2 − 1)dx

= [1

2x3 − x]30

27

2− 3 =

21

2

207

196. Calcule a integral dupla∫ ∫

RxyeydA,R = (x, y)|0 ≤ x ≤ 2, 0 ≤ y ≤ 1

Solucao .

∫ 2

0

∫ 1

0

xyeydydx

=

∫ 2

0

xdx

∫ 1

0

yeydy

= [1

2x2]20[e

y(y − 1)]10

Resolvendo com integral por partes temos,

=1

2(4 − 0)(0 + e0) = 2

208

197. Calcule a integral dupla∫ ∫

Rxy2

x2+1dA,R = (x, y)|0 ≤ x ≤ 1,−3 ≤ y ≤ 3

Solucao .

∫ 1

0

∫ 3

−3

xy2

x2 + 1dydx

=

∫ 1

o

x

x2 + 1dx

−3

3y2dy

= [1

2ln(x2 + 1)]10[

1

3y3]3−3

=1

2(ln2 − ln1)

1

3(27 + 27)

= 9ln2

209

198. Calcule a integral dupla∫ ∫

R1+x2

1+y2 dA,R = (x, y)|0 ≤ x ≤ 1, 0 ≤ y ≤ 1

Solucao .

∫ 1

0

∫ 1

0

1 + x2

1 + y2dydx

=

∫ 1

0

(1 + x2)dx

∫ 1

0

1

1 + y2dy

= [x +1

3x3]10[tg

−1y]10

= (1 +1

3− 0)(

π

4− 0)

3

210

199. Calcule a integral dupla∫ ∫

RxexydA,R = [0, 1]X[0, 1]

Solucao .

∫ 1

0

∫ 1

0

xexydydx

=

∫ 1

0

[exy]10dx

=

∫ 1

0

(ex − 1)dx

= [ex − x]10

= e − 2

211

200. Calcule a integral dupla∫ ∫

R1

x+y dA,R = [1, 2]X[0, 1]

Solucao .

∫ 1

0

∫ 2

1

1

x + ydxdy

=

∫ 1

0

[ln(x + y)]21dy

=

∫ 1

0

[ln(2 + y) − ln(1 − y)]dy

= [[(2 + y)ln(2 + y) − (2 + y)] − [(1 + y)ln(1 + y) − (1 + y)]]10

Integrando por partes em separado cada termo ou por uma tabela de integracao temos,

= 3(ln3) − 3 − (2ln2) + 2 − [(2ln2 − 2) − (0 − 1)]

= 3ln3 − 4ln2

= ln27

16

212

201. Determine o volume do solido que esta contido abaixo do paraboloide circular

Z = x2 + y2 e acima do retangulo R=[-2,2]X[-3,3]

Solucao .

V =

∫ ∫

R

(x2 + y2)dA

=

∫ 3

−3

∫ 2

−2

(x2 + y2)dxdy

=

∫ 3

−3

[1

3x3 + y2x]2−2dy

=

∫ 3

−3

[16

3+ 4y2]dy

= [16

3y +

4

3y3]3−3

= 2(16 + 36)

= 104

213

202. Determine o volume do solido que esta contido abaixo do paraboloide eliptico

x2

4 + y2

9 + z = 1 e acima do retangulo R=[-1,1]X[-2,2]

Solucao .

V =

∫ 2

−2

∫ 1

−1

(1 − 1

4x2 − 1

9y2)dxdy

= 4

∫ 2

0

∫ 1

0

(1 − 1

4x2 − 1

9y2)dxdy

= 4

∫ 2

0

[x − 1

12x3 − 1

9y2x]10dy

= 4

∫ 2

0

(11

12− 1

9y2)dy

= 4[11

12y − 1

27y3]20

= 483

54

=166

27

214

203. Determine o volume do solido que esta contido abaixo do paraboloide hiper-

bolico Z = y2 − x2 e acima do retangulo R=[-1,1]X[1,3]

Solucao .

V =

∫ 3

1

∫ 1

−1

(y2 − x2)dxdy

= 2

∫ 3

1

∫ 1

0

(y2 − x2)dxdy

= 2

∫ 3

1

[y2x − 1

3x3]10dy

= 2

∫ 3

1

(y2 − 1

3)dy

=2

3[y3 − y]31

= 16

215

204. Calcule a integral dupla∫ ∫

De

xy dA,D = (x, y)|1 ≤ y ≤ 2, y ≤ x ≤ y3

Solucao .∫ 2

1

∫ y3

y

exy dxdy

=

∫ 2

1

yexy ]y

3

y dy

=

∫ 2

1

(yey2 − ey)dy

= [1

2ey2 − 1

2ey2

]21

=1

2(e4 − 4e)

216

205. Calcule a integral dada colocando em coordenadas polares.∫ ∫

RydA, onde R

e a regiao limitada pelo circulo x2 + y2 = 9 e pelas retas y=x e y=.

Solucao .

∫ ∫

R

ydA

=

∫ π4

0

∫ 3

0

(rsenθ)rdrdθ

= (

∫ π4

0

senθdθ)(

∫ 3

0

r2dr)

= (

√2 − 1√

2(9)

= 91 − 1√2

217

206. Calcule a integral dada colocando em coordenadas polares.∫ ∫

R

x2 + y2dA,

onde R = (x, y)|1 ≤ x2 + y2 ≤ 9, y ≥ 0

Solucao .∫ ∫

R

x2 + y2dA

=

0

π

∫ 3

1

√r2rdrdθ

= (

0

πdθ)(

∫ 3

1

r3dr)

= [θ]π0 [1

3r3]31

= π(27 − 1

3)

=26

218

207. Calcule a integral dada colocando em coordenadas polares.∫ ∫

De−x2−y2

dA,

onde D e a regiao limitada pelo semicirculo x =√

4 − y2 e o eixo y.

Solucao .∫ ∫

D

e−x2−y2

dA

=

−π2

π2

∫ 2

0

e−r2

rdrdθ

= (

∫ π2

−π2

dθ)(

∫ 2

0

re−r2

dr)

= [θ]π2

−π2[−1

2e−r2

]20

= π(−1

2)(e−4 − e0)

2(1 − e−4)

219

208. Calcule a integral dada colocando em coordenadas polares.∫ ∫

D(x2 + y2)dA,

onde D e a regiao limitada pelas espirais r = θ e r = 2θ para 0 ≤ θ ≤ 2π.

Solucao .

∫ 2π

o

∫ 2θ

0

r2rdrdθ

=

∫ 2π

o

[1

4r4]2θ

θ dθ

=1

4

∫ 2π

o

15θ4dθ

=3

4(32π5)

= 24π5

220

209. Determine a area da superficie. A parte do plano z = 2+3x+4y que esta acima

do retangulo [0,5]X[1,4]

Solucao .

Aqui z = f(x, y) = 2 + 3x + 4y e e o retangulo [0,5]X[1,4], assim, a area da superficie e

A(s) =

∫ ∫

D

32 + 42 + 1dA

=√

26

∫ ∫

D

dA

=√

26A(D)

=√

26(5)(3)

= 15√

26

221

210. Determine a area da superficie. A parte do plano 2x + 5y + z = 10 que esta

dentro do cilindro ex2 + y2 = 9

Solucao .

z = f(x, y) = 10 − 2x − 5y e D eo disco x2 + y2 ≤ 9, assim

A(s) =

∫ ∫

D

(−2)2 + (−5)2 + 1dA

=√

30

∫ ∫

D

dA

=√

30A(D)

=√

30(π32)

= 9√

30π

222

211. Mostre que a area da parte da superficie do plano z = ax+ by + c com projecao

sobre a regiao D no plano xy com area A(D) e√

a2 + b2 + 1 A(D)

Solucao .

Aqui

z = f(x, y) = ax + bx + c, fx(x, y) = a, fy(x, y) = b

, assim

A(S) =

∫ ∫

D

a2 + b2 + 1dA

=√

a2 + b2 + 1

∫ ∫

D

dA

=√

a2 + b2 + 1A(D)

223

212. Calcule∫ 1

0

∫ z

0

∫ x+z

06xzdydxdz

Solucao .∫ 1

0

∫ z

0

[6xz]x+z0 dz

=

∫ 1

0

∫ z

0

6xz(x + z)dxdy

=

∫ 1

0

[2x3z + 3x2z2]z0dz

=

∫ 1

0

(2z4 + 3z4)dz

= 5z4]10

= 1

224

213. Determine o volume do solido, abaixo do paraboloide z = 3x2 + y2 e acima da

regiao limitada por y = x e x = y2 − y

Solucao .

v =

∫ 2

0

∫ y

y2−y

(3x2 + y2)dxdy

=

∫ 2

0

[x3 + y2x]yy2−ydy

=

∫ 2

0

[2y3 − (y6 − 3y5 + 4y4 − 2y3)]dy

= [−y7

7 +y6

2− 4y5

5+ y4]20

=144

55

225

214. Determine o volume do solido, abaixo da superficie z = xy e acima do triangulo

com vertices em (1,1), (4,1) e (1,2)

Solucao .

v =

∫ 2

1

∫ 7−3y

1

xydxdy

=

∫ 2

1

[1

2x2y]7−3y

1

=1

2(48y − 42y2 = 9y3)dy

=1

2[24y2 − 14y3 +

9

4y4]21

=31

8

226

215. Determine o volume do solido, limitado pelo parabolooide z = x2 + y2 + 4 e

pelos planos x = 0, y = 0, z = 0, x + y = 1

Solucao .

v =

∫ 1

0

∫ 1−x

0

(x2 + y2 + 4)dxdy

=

∫ 1

0

[x2y +1

3y3 + 4y]1−x

0 dx

=

[x2 − x3 +1

3(1 − X)3 + 4(1 − X)]dx

= [1

3x3 − 1

4x4 − 1

12(1 − x)4 − 2(1 − x2)]

=13

6

227

216. Determine o volume do solido, limitado pelo cilindro x2 +y2 = 1 e pelos planos

y = z, x = 0, z = 0 no primeiro octante.

Solucao .

v =

∫ 1

0

∫ √1−x2

0

ydydx

=

∫ 1

0

[y2

2]√

1−x2

0 dx

∫ 1

0

1 − x2

2dx

=1

2[X − 1

3x3]10

=1

3

228

217. No calculo de uma integral dupla sobre uma regiao D obtivemos uma soma

de integrais como a que se segue:

∫ ∫

D

f(x, y)dA =

∫ 1

0

∫ 2y

0

f(x, y)dxdy +

∫ 3

1

∫ 3−y

0

f(x, y)dxdy

Solucao .

∫ ∫

D

f(x, y)dA

=

∫ 1

0

∫ 2y

0

f(x, y)dxdy +

∫ 3

1

∫ 3−y

0

f(x, y)dxdy

=

∫ 2

0

x2

3−x

f(x, y)dydx

229

218. Seja R a regiao interior do trapezoide cujos vertices sao (2,2), (4,2), (5,4) e

(1,4). Calcule a integral∫ ∫

R

8xydxdy

Solucao . A equacao da reta pelos pontos (2,2) e (1,4) ey = 6−2x, ou 6−y2 . Igualmente,

a equacao da reta pelos pontos (4,2) e (5,4) ey = 2x − 6, ou 6+y2 . Usando o resultado

obtido acima, temos

∫ ∫

R

8xydxdy =

∫ 4

2

∫ (6+y)/2

(6−y)/2

8xydxdy =

∫ 4

2

[

8yx2

2

](6+y)/2

(6−y)/2

dy =

∫ 4

2

[(6 + y

2

)2

−(

6 − y

2

)2]

dy

=

∫ 4

2

24y2dy =

[24y3

3

]4

2

= 448.

230

219. Ache o centroide da regiao R limitada por

y = x + 2 e y = x2.

Solucao . A regiao R e do tipo I, e temos

A =

∫ 2

−1

∫ x+2

x2

dydx =

∫ 2

−1

(x + 2 − x2

)dx =

[x2

2+ 2x − x3

3

]2

−1

=9

2u.a.

Portanto,

x =1

A

∫ ∫

R

xdxdy =2

9

∫ 2

−1

∫ x+2

x2

xdydx =2

9

∫ 2

−1

[x (x + 2) − x3

]dx

=2

9

[x3

3+ x2 − x4

4

]2

−1

=2

9

(8

3− 5

12

)

=1

2

e

y =1

a

∫ ∫

R

ydxdy =2

9

∫ 2

−1

∫ x+2

x2

ydydx =2

9

∫ 2

−1

1

2

[

(x + 2)2 − x4

]

dx

2

9

[x3

3+ 2x2 + 4x − x5

5

]2

−1

=1

9

[184

15−(

−32

15

)]

=8

5.

231

220. Inverta a ordem de integracao e calcule a integral resultante.

∫ 1

0

∫ 1

x

x sin xy3dydx

Solucao . A integral iterada dada e equivalente a integral dupla

∫ ∫

R

x sin y3dxdy

Sobre a regiao R tipo I determinada pelas inequacoes 0 ≤ x ≤ 1 e x ≤ y ≤ 1. Como R e

tambem do tipo II, temos

∫ 1

0

∫ 1

x

x sin y3dydx =

∫ ∫

R

x sin y3dxdy =

∫ y=1

y=0

(∫ x=y

x=0

x sin y3dx

)

dy

=

∫ y=1

y=0

[x2 sin y3

2

]x=y

x=0

dy =1

2

∫ 1

0

y2 sin y3dy =

[

−cos y3

6

]1

0

= −1

6(cos 1 − cos 0) ≈ −1

6(0, 5403 − 1) ≈ 0, 077.

232

221. Inverta a ordem de integracao e calcule a integral resultante.

∫ 3

0

∫ 9

y2

ye−x2

dxdy

Solucao . A integracao iterada fornecida e equivalente a integral dupla∫ ∫

Rye−x2

dxdy

Sobre a regiao R do tipo II, determinada pelas inequacoes , y2 ≤ x ≤ 9 e 0 ≤ y ≤ 3

∫ 3

0

∫ 9

y2

ye−x2

dxdy =

∫ ∫

R

ye−x2

dxdy =

∫ x=9

x=0

(∫ y=

√x

y=0

ye−x2

dy

)

dx

=

∫ x=9

x=0

[

y2e−x2

2

]y=√

x

y=0

dx =

∫ 9

0

xe−x2

2dx

=

[

−e−x2

4

]9

0

= −e−81

4+

1

4=

1

4

(1 − e−81

).

Embora existam regioes que nao sao nem do tipo I nem do tipo II, e possıvel cortar

tal regiao em sub-regioes nao -superpostas, cada uma do tipo I ou do tipo II. A integral

dupla de uma funcao sobre uma regiao grande pode entao ser calculada pela integracao

da funcao sobre cada sub-regiao somando-se os valores resultantes.

233

222. Calcule∫ ∫

R(2x − y) dxdy sobre a regizao R da figura 1.a

Solucao . Embora R nao seja do tipo I e nem do tipo II, podemos decompo-las em duas

regioes distintas R1 : 2 ≤ x ≤ 4, 1 ≤ y ≤ 3 e R2 : 1 ≤ x ≤ 4, 2 ≤ y ≤ 3 (figura 1.b)

Logo,

∫ ∫

R1

(2x − y) dxdy =

∫ 4

2

∫ 2

1

(2x − y) dydx =

∫ 4

2

[

2xy − y2

2

]y=2

y=1

dx

234

∫ 4

2

(

2x − 3

2

)

dx =

[

x2 − 3

2x

]4

2

= 9

∫ ∫

R2

(2x − y) dxdy =

∫ 4

1

∫ 3

2

(2x − y) dydx =

∫ 4

1

[

2xy − y2

2

]y=3

y=2

dx

∫ 4

1

(

2x − 5

2

)

dx =

[

x2 − 5

2x

]4

1

=15

2.

Portanto,

∫ ∫

R

(2x − y) dxdy =

∫ ∫

R1

(2x − y) dxdy =

∫ ∫

R2

(2x − y) dxdy = 9 +15

2=

33

2.

235

223. Calcule a integral dupla dada pelo metodo da iteracao .

∫ ∫

R

xcos (xy)dxdy;

R : 1 ≤ x ≤ 2 e π2 ≤ y ≤ 2π

x

Solucao . A regiao e evidente do tipo I (figura); logo,

∫ ∫

R

xcos (xy)dxdy =

∫ x=2

x=1

(∫ y=2π/x

y=π/2

xcos (xy)dy

)

dx =

∫ x=2

x=1

[sin (xy)]y=2π/xy=π/2 dx

=

∫ 2

1

[

sin (xy) − sin(π

2x)]

dx =

∫ 2

1

[

− sin(πx

2

)]

dx

=

[2

πcos(πx

2

)]2

1

=2

πcos (π) − 2

πcos(π

2

)

= − 2

π.

236

224.∫ ∫

R(x + y) dxdy, onde R e a regiao no primeiro quadrante acima da curva y = x2

e abaixo da curva y =√

x.

Solucao . Neste caso, a regiao R e simultaneamente do tipo I e do tipo II (figura).

Considerando R como uma regiao do tipo II, e limitada a esquerda pela curva x = y2; a

direita pela curva y =√

x; abaixo pela linha y = 1. Portanto,

∫ ∫

R

(x + y) dxdy =

∫ y=1

y=0

[∫ x=

√y

x=y2

(x + y) dx

]

dy

=

∫ y=1

y=0

[x2

2+ yx

]x=√

y

x=y2

dy =

∫ 1

0

(y

2+ y

√y − y4

2− y3

)

dy

=

[y2

4+

2

5y5/2 − y5

10− y4

4

]1

0

=3

10.

237

225. Seja R a regiao interior ao circulo x2+y2 ≤ 4 e seja definida f (x, y) =√

4 − x2 − y2.

Calcule∫ ∫

R

f (x, y) dxdy.

Solucao . O grafico de f e um hemisferio de raio r = 2 unidades e a regiao R forma

a base deste hemisferio. O solido acima de R e abaixo do grafico de f e, portanto, um

solido hemisferio de raio r = 2 unidades (figura 1.). Logo, seu volume vale

1

2

(4

3πr3

)

=16π

3u.v

238

226. Seja R a regiao no plano xy limitada acima pela parabola y = 4 − x2 limitada

abaixo pelo eixo x. Ache o volume V sob o grafico de f (x, y) = x + 2y + 3 e

acima da regiao R.

Solucao . A regiao R e do tipo I, e limitada a esquerda pela reta vertical x = −2, a

direita pela reta vertical x = 2, acima pela parabola y = 4− x2, e abaixo pela reta y = 0.

Portanto, pelo metodo da iteracao ,

V =

∫ ∫

R

(x + 2y + 3) dxdy =

∫ 2

−2

[∫ 4−x2

0

(x + 2y + 3) dy

]

dx

=

∫ 2

−2

[xy + y2 + 3y

]4−x2

0dx =

∫ 2

−2

[

x(4 − x2

)+(4 − x2

)2+ 3

(4 − x2

)]

dx

=

∫ 2

−2

[x4 − x3 − 11x2 + 4x + 28

]dx

=

[x5

5− x4

4− 11x3

3+ 2x2 + 28x

]2

−2

=992

15u.v.

239

227. Ache a area da regiao R limitada pelas curvas

y = x2 e y = x.

Solucao . A regiao (Figura ) e do tipo I; logo, a area e dada por

A =

∫ ∫

R

dxdy =

∫ x=1

x=0

[∫ y=x

y=x2

dy

]

dx

=

∫ x=1

x=0

[y]y=xy=x2 dx =

∫ 1

0

(x − x2

)dx

=

[x2

2− x3

3

]1

0

=1

6u.a.

240

228. Encontrar a primitiva da funcao f (x, y) = 12x2y3 em relacao a x.

Solucao . Vamos admitir y como constante e integrar em relacao a x. Portanto,

f (x, y) = 12x2y3dx = 4x3y3 + C

Porem, nesse caso, a constante C e uma funcao de y. Pode ser por exemplo, C (y) =

ay3 + by2 + cy + 3 e uma das primitivas de f (x, y) = 12x2y3 sera

F (x, y) = 4x3y3 + ay3 + by2 + cy + 3

Note que,∂F (x, y)

∂x= 12x2y3

241

229. Encontrar a primitiva da funcao f (x, y) = 12x2y3 em relacao a y.

Solucao . Vamos admitir x como constante e integrar em relacao a y. Portanto,

f (x, y) = 12x2y3dy = 3x2y4

Porem, nesse caso, a constante K e uma funcao de x. Pode ser por exemplo, K (y) =

ax3 + bx2 + cx + 3 e uma das primitivas de f (x, y) = 12x2y3 sera

F (x, y) = 3x2y4 + ax3 + bx2 + cx + 3.

Note que∂F (x, y)

∂y= 12x2y3

242

230. Encontrar o valor da integral

∫ 4

0

∫ 3x

x

3√

16 − x2dydx.

Solucao . Aplicando o teorema fundamental do calculo, primeiro integrando em relacao

a y e depois em ralcao a x.

∫ 4

0

∫ 3x

x

3√

16 − x2dydx =

∫ 4

0

[

3√

16 − x2y]3x

xdx

=

∫ 4

0

(

3√

16 − x2)

(3x − x) dx =

∫ 4

0

6x√

16 − x2dx

=

[

−2

(16 − x2)3

]4

0

=

(

−2

(16 − 42)3

)

−(

−2

(16 − 02)3

)

= 128

Portanto o valor da integral

∫ 4

0

∫ 3x

x

3√

16 − x2dydx = 128.

243

231. Escreva a integral que representa a area da regiao delimitada pelas curva

x = y2, y−x = 1, y = 1 e y = −1. Tomando y como variavel independente.

Solucao . A area delimitada pelas curvas pode ser vista na Figura 3.5

Inicialmente, vamos encontrar os pontos de intersecao

x = y2

y = 1P (1, 1)

x = y2

y = −1Q (−1, 1)

y = 1 + x

y = −1R (−2,−1)

tomando y como variavel independente

244

A =

∫ 1

−1

∫ y2

y−1

dxdy =8

3

245

232. Escreva a integral, em coordenadas polares, que calcula a area sombreada 3.6

Solucao .

Cırculo 1: x2 + y2 = 4 (em cartesianas) ρ = 2 (em polar)

Cırculo 2: (x − 2)2

+ y2 = 4 (em cartesianas) ρ = 4 cos θ (em polar)

a interseccao dos dois:

cos θ = 12 −→ θ = π

3

A area e

A =

∫ π3

0

∫ 4 cos θ

2

ρdρdθ

Em coordenadas polares.

246

233. Determine o volume do solido delimitado pelos planos

z = 0, y = 0, x = 0 e y + x2 + y

2 = 2

Solucao . Vamos fazer o esbco do solido como na figura.1

Agora, vamos escolher o plano xy ver figura.2 para fazer a projecao

247

Limites R1

a esquerda x = 0

a direita x = 4

Curva inferior y = 0

Cuurva superior y = 2 − x2

superior inferior z = 0

superior superior z = 4(2 − x

2 − y)

V =

∫ 4

0

∫ 2− x2

0

∫ 4(2− x2 −y)

0

dzdydx =

∫ 4

0

∫ 2− x2

0

[z]4(2− x

2 −y)0

=

∫ 4

0

∫ 2− x2

0

(8 − 2x − 4y) =

∫ 4

0

[8y − 2xy − 2y2

]2− x2

0

=

∫ 4

0

[

2x

(1

2x − 2

)

− 4x − 2

(1

2x − 2

)2

+ 16

]

dx

=

∫ 4

0

[1

2x2 − 4x + 8

]

dx =32

3u.v.

248

234. Calcular o volume do solido delimitado pela interseccao dos cilindros

z2 + x2 = 9 e y2 + x2 = 9 no primeiro octante.

Solucao . Vamos fazer o desenho do solido e escolher um dos planos coordenados para

a projecao .

Como o solido faz parte do primeiro octante, temos os planos z = 0, y = 0, e x = 0

delimitando o solido.

Limites R1

a esquerda x = 0

a direita x = 3

Curva inferior y = 0

Cuurva superior y =√

9 − x2

superior inferior z = 0

superior superior z =√

9 − x2

V =

∫ 3

0

∫ √9−x2

0

∫ √9−x2

0

dzdydx

=

∫ 3

0

∫ √9−x2

0

9 − x2dydx

=

∫ 3

0

[

y√

9 − x2]√

9−x2

0dx

249

=

∫ 3

0

(9 − x2

)dx

=

[

9x − x3

3

]3

0

= 27 − 9 = 18 u.v.

250

235. Encontrar o volume do solido delimitado pelas superfıcies z = 9 − x2, z =

5 − y, y = 0, e y = 5.

Solucao . O primeiro passo e determinar as curvas que limitam a regiao de integracao

sobre o plano xy. Para isso resolvemos o sistema de equacoes

z = 9 − x2

z = 5 − y

Igualando as duas equacoes obtemos a parabola y = x2 − 4. Desse modo, no plano xy,

a regiao de integracao e delimitada pelas curvas y = x2 − 4, y = 0, e y = 5. Para

diminuir o trabalho no processo de integracao e conveniente tomar y como variavel inde-

pendente. Desse modo a tabela de limites e dada por:

251

O volume e dado por

V =

∫ 5

0

∫ √y+4

−√y+4

∫ 9−x2

5−y

dzdxdy

=

∫ 5

0

∫ √y+4

−√y+4

[z]9−x2

5−y dxdy

=

∫ 5

0

∫ √y+4

−√y+4

[9 − x2 − (5 − y)

]dxdy

=

∫ 5

0

∫ √y+4

−√y+4

[4 − x2 + y

]dxdy

Como a superfıcie e simetrica em relacao ao eixo y podemos escrever

= 2

∫ 5

0

∫ √y+4

0

[4 − x2 + y

]dxdy = 2

∫ 5

0

[

4x − x3

3+ yx

]√

y+4

0

dy

= 2

∫ 5

0

[

4√

y + 4 −(√

y + 4)3

3+ y√

y + 4

]

dy = 2

∫ 5

0

[8

3

y + 4 +2

3y√

y + 4

]

dy

= 2

[16

9

(√

y + 4)3

+4

15

(√

y + 4)5

− 16

9

(√

y + 4)3]5

0

= 2

[4

15

(√

y + 4)5]5

0

= 2

[4

15

(√5 + 4

)5 − 4

15

(√4)5]

= 2

[

−8

9

(√9)3

+4

15

(√9)5

−(

−8

9

(√4)3

+4

15

(√4)5)]

= 2

[

−8

9(27) +

4

15(243) −

(

−8

9(8) +

4

15(32)

)]

=1688

15= 112, 53u.v.

252

236. Seja A o retangulo 1 ≤ x ≤ 2, 0 ≤ y ≤ 1. Calcule a integral sendo f(x, y) igual a

x + 2y.∫ ∫

A

f(x, y)dxdy,

Solucao . Pelo teorema de Fubini

∫ 1

0

∫ 2

1

(x + 2y)dxdy =

∫ 1

0

α(y)dy

Para cada y fixo em [0,1], temos:

α(y) =

∫ 2

1

(x + 2y)dx

=

[x2

2+ 2xy

]2

1

=22

2+ 2 · 2y −

(12

2+ 2 · 1 · y

)

= 2 + 4y − 1

2− 2y

Ou seja,

α(y) =3

2+ 2y

⇒∫ 1

0

α(y)dy =

∫ 1

0

(3

2+ 2y

)

dy

=

[3y

2+ y2

]1

0

=3 · 1

2+ 12

=5

2

253

237. Suponha que a area de uma regiao no plano de coordenadas polares seja

A =∫ 3

4 π14 π

∫ 2sen(θ)

cosec(θ)r dr dθ. Esboce a regiao e encontre sua area.

Solucao . Em coordenadas cartesianas, a igualdade r = 2 sen(θ) corresponde a equacao

de um cırculo. De fato, multiplicando por r, obtem-se que r2 = 2 r sen(θ), onde:

r2 = x2 + y2 e 2 r sen(θ) = 2y

Assim, a igualdade corresponde a equacao , x2 + y2 = 2y, ou ainda x2 + (y − 1)2 = 1,

que e a equacao de um cırculo de raio 1 e centro no ponto (0, 1). O cırculo esta esbocado

logo a seguir. Por sua vez, a igualdade r = cosec(θ) corresponde a equacao de uma reta.

Para ver isso, basta notar que cosec(θ) = 1sen(θ) , e portanto a igualdade e equivalente a

r sen(θ) = 1, isto e y = 1, que e a equacao de uma reta paralela ao eixo Ox.

Segue-se que a regiao de integracao corresponde aquela entre o cırculo e a reta, com o

angulo θ entra π4 e 3π

4 . Essa regiao esta ilustrada na figura abaixo, na cor azul claro.

A integral pode ser calculada diretamente usando integrais iteradas. Mas pode ser calcu-

lada de uma forma indireta, uma vez que a integral corresponde a area da regiao esbocada

acima, e ja se sabe que esta area e igual a π2 (metade a area do disco de raio 1).Assim:

∫ 34 π

14 π

∫ 2 sen(θ)

cosec(θ)

r dr dθ =1

254

238. Esboce a regiao de integracao , inverta a ordem de interacao e calcule a

integral∫ 2

0

∫ 2

x2y2 sen(x y)dy dx.

Solucao . Segundo os extremos de integracao , para cada x fixo no intervalo [0, 2], y

varia no intervalo [x, 2]. Essa variacao esta ilustrada na cor vermelha na figura abaixo.

A regiao de interacao esta indicada em azul claro.

Para inverter a ordem de integracao , observe que a mesma regiao pode ser descrita da

seguinte forma: para cada y no intervalo [0, 2], x varia de 0 ate y. Essa variacao esta

indicada em azul na figura acima. Com essa descricao obtem-se que:

∫ 2

0

∫ 2

x

2y2 sen(x y)dy dx = −2 cos(y2) y + 2y dy

A vantagem dessa inversao e que, agora, as integrais podem ser calculadas de forma

simples. De fato, a primeira integral a ser feita e:

∫ y

0

2y2 sen(x y) dx = −2 cos(y2) y + 2y

Asegunda integral e igualmente facil, sendo igual a:

∫ 2

0

∫ 2

x

2y2 sen(x y)dy dx =

∫ 2

0

−2 cos(y2) y + 2y dy =

= −sin (4) + 4

255

239. Encontre o volume do solido que e limitado superiormente pelo cilındro z = x2

e inferiormente pela regiao delimitada pela parabola y = 2 − x2 e pela reta y=

x no plano Oxy.

Solucao . A figura abaixo ilustra os graficos das funcoes g(x) = 2 − x2 (em vermelho)

e h(x)= x (em azul). Do grafico e claro que, se D e a regiao limitada por essas curvas, entao

D e uma regiao do tipo Rx que pode ser descrita como D = (x, y);−2 ≤ x ≤ 1ex ≤ y ≤ 2 − x2.

Esbocando o domınio acima juntamente como o cilındro z = x2, obtem-se o grafico abaixo,

que e uma ilustracao do solido.

Da ilustracao acima e claro que o volume procurado corresponde a integral da funcao f(x,

y)= x2 sobre o domınio D. Calculando, obtem-se que o volume e dado por:

∫ ∫

x2 dx dy =

∫ 1

−2

∫ 2−x2

x

x2 dy dx =

256

=

∫ 1

−2

x2 (2 − x2 − x) dx =

=63

20

257

240. Enconte o volume da regiao no primeiro octante limitada pelos planos coor-

denados pelos plano x+ y= 4 e pelo cilindro y2 + 4z2 = 16

Solucao . A regiao esta ilustrada abaixo, em que o cilindro y2 + 4z2 = 16 esta em verde

e o plano x+ y= 4 em rosa.

Indique pro R a regiao da qual se quer calcular o volume. Da figura percebe-se que a base

B de R e aregiao triangular descrita da seguinte maneira: para cada x fixo no intervalo

[0, 4], y varia no intervalo [0, 4- x].

Isolando o valor de z da expressao do cilındro, obten-se que z =

√16− y2

2 .

Daı segue-se que R pode ser descrita como: para cada (x, y) fixo na base B, z varia no

intervalo [0,

√16−y2

2 ]. Usando essa descricao , segue-se que o volume V de R e dado pela

integral tripla:

V =

∫ 4

0

∫ 4−x

0

∫ 12

√16−y2

0

1 dz dy dx =

=

∫ 4

0

∫ 4−x

0

1

2

16 − y2 dy dx =

∫ 4

0

8x − x2 − 1

4

8x − x2 x − 4arcsen (−1 +1

4x) =

V = −32

3+ 8π

258

241. Enconte o volume da regiao que esta dentro da esfera x2 + y2 + z2 = 2 e fora

do cilindro x2 + y2 = 1.

Solucao . A regiao esta ilustrada abaixo, juntamente com parte de um corte vertical

que facilita a sua descricao .

Indique por Q a regiao , em coordenadas cartesianas, fora da esfera e dentro do cilindro.

O volume dessa regiao e dado por:

V =

∫ ∫ ∫

Q

1 dx dy dz

Essa integral pode ser calculada de va rias formas, incluindo coordenadas cilındricas. Para

isso, indique por Q1 a mesma regiao Q em coordenadas cilindrıcas. Nessas coordenadas,

a equacao da esfera e r2 + z2 = 2 e a equacao do cilındro e r2 = 1. Dessa observacao e

das ilustracoes acima, segue-se que Q1 e o conjunto dos pontos (r, θ, z) em que θ esta

no intervalo [0, 2π], r esta no intervalo [1,√

2] e z esta no intervalo [−√

2 − r2,√

2 − r2].

Assim, o volume da regiao e dado por:

V =

∫ 2π

0

∫ √2

1

∫ √2−r2

−√

2−r2

r dz dr dθ

=

∫ 2π

0

∫ √2

1

2r√

2 − r2 dr dθ =

∫ 2π

0

2

3dθ =

V =4π

3

259

242. Esboce a regiao de integracao , inverta a ordem de interacao e calcule a

integral∫ π

0

∫ π

xsen(y)

y dy dx.

Solucao . Segundo os extremos de integracao , para cada x fixo no intervalo [0,π], y

varia de x ate π. Essa variacao esta ilustrada na cor vermelha na figura abaixo, e resulta

na regiao de interacao ilustrada na cor preto.

Para inverter a ordem de integracao , observe que a regiao pode ser descrita da seguinte

forma: para cada y fixo no intervalo[0, π], x varia no intervalo [0, y]. Essa nova maneira

de descrever a regiao esta ilustrada em azul na figura acima. Daı segue-se que a integral

pode ser escrita como:∫ π

0

∫ π

xsen(y)

y dy dx =∫ π

0

∫ y

0sen(y)

y dx dy

Observe agora que a integral interna e facil de ser calculada, e e igual a:

∫ y

0

sen(y)

ydx = sen(y)

Daı segue-se que:∫ π

0

∫ y

0sen(y)

y dx dy =∫ π

0sen(y) dy

Entao :∫ π

0

∫ y

0

sen(y)

ydx dy = 2

260

243. Converta a integral para integral equivalente em coordenadas cilındricas e

calcule o resultado:∫ 1

−1

∫√

1−y2

0

∫ x

0x2 + y2 dz dx dy

Solucao . Segundo os extremos de integracao , para cada y fixo no intervalo [-1, 1], x

varia no intervalo [0,√

1 − y2]. Isso descreve um semi-disco de raio 1 no plano Oxy. Em

seguida, para cada (x, y) nesse semi-disco, z varia no intervalo [0, x]. Com essa descricao

, o domınio de integracao pode ser ilustrado como na figura abaixo.

O semi-disco corresponde ao retangulo em que θ varia no intervalo [− π2 , π

2 ] e r varia no

intervalo [0, 1]. Alem disso, para cada (r, θ) nesse retangulo, z varia de 0 ate rcos(θ).

Apos essa descricao e lembrando que x2 + y2 = r2, a integral em coordenadas cilındricas

e:

I =

∫ 1

−1

∫√

1−y2

0

∫ x

0

x2 + y2 dz dx dy =

− 12 π

∫ 1

0

∫ r cos(θ)

0

r2 |r| dz dr dθ

Onde |r| e o modulo do jacobiano da mudanca de coordenadas.

∫ r cos(θ) r2 |r| dz = r3 |r| cos(θ)

0

∫ 1

0

r3|r| cos(θ) dr =1

5cos(θ)

∫ 12 π

− 12 π

1

5cos(θ) dθ =

2

5

I =2

5

261

244. Esboce a regiao de integracao , inverta a ordem de integracao e calcula a

integral:∫ 2

0

∫ 4−x2

0x e(2y)

4−y dy dx

Solucao . Segundo os extremos de integracao , para cada x no intervalo [0, 2], y varia no

intervalo [0, 4 − x2]. Essa variacao esta indicada em vermelho na figura abaixo, em que

foi usada a notacao g(x) = 4 − x2. A regiao de integracao esta indicada em azul claro.

Para inveter a ordem de integracao observe que a mesma regiao pode ser descrita da

seguinte forma: para cada y no intervalo [0, 4], x varia de 0 ate√

4 − y. Essa variacao

esta indicada em azul na figura acima, onde foi usada a notacao h(y)=√

4 − y. Com Essa

descricao obtem-se que:

∫ 2

0

∫ 4−x2

0

x e(2y)

4 − ydy dx =

∫ 4

0

∫ √4−y

0

x e(2y)

4 − ydx dy

A vantagem dessa inversao e que, as integrais podem ser calculadas de forma simples. De

fato, a primeira integral a ser feita e:

∫ 4

0

∫ √4−y

0

x e(2y)

4 − ydx = [

e(2y)

4 − y]

∫ √4−y

0

x dx =1

2e(2y)

A segunda integral e igual a :

∫ 4

0

∫ √4−y

0

x e(2y)

4 − ydx dy =

∫ 4

0

1

2e(2y)

1

4e8 − 1

4

262

245. Monte a integral iterada para calcular∫ ∫ ∫

Df(r, θ, z) z dz dr dθ Onde D e o

cilindro reto solido cuja base e a regiao entre as circunferencias r = cos(θ) e

r = 2cos(θ) e cujo topo esta no plano z = 3 − y.

Solucao . Para descrever o solido em coordenadas cilındircas, observe que −( π2 ) < θ < (π

2 ),

pois o solido esta no primeiro e quarto quadrante. Ja o raio varia de uma circunferencia

a autura, isto e, cos(θ) < r < 2cos(θ). Finalmente, o z varia da base ate o topo, isto e,

0 < z < 3 − r sen(θ). Apos essas observacoes , a integral pode ser escrita como:

∫ ∫ ∫

D

f(r, θ, z) z dz dr dθ =

− 12 π

∫ 2cos(θ)

cos(θ)

∫ 3−r sen(θ)

0

f(r, θ, z) r dz dr dθ

V =

− 12 π

∫ 2cos(θ)

cos(θ)

r (3 − r sen(θ)) dr dθ

=

∫ 12 π

− 12 π

− 7

3sen(θ) cos(θ)3 +

9

2cos(θ)2 dθ

=9

263

246. Seja D a regiao no espaco Oxyz definida pelas desigualdades: 1 < x < 2,

0 < xy < 2, 0 < z < 1. Calcula∫ ∫ ∫

x2y + 3xyz dx dy dz aplicando a transformacao

u= x, v= xy, w= 3z e integre sobre uma regiao apropriada G do espaco Ouvw.

Solucao . A regiao D esta ilustrada na figura abaixo.

Nao e difıcil perceber que a regiao G no espaco Ouvw e dada pelas desigualdades:1 < u < 2,

0 < v < 2,0 < w < 3. No entanto, para o calculo da integral, deve-se usar a transformacao

inversa, esto e, a transformacao :

x = u, y =v

u, z =

w

3

cujo jacobiano e igual a:

det

xu yu zu

xv yv zv

xw yw zw

= det

1 − vu2 0

0 1u 0

0 0 13

=1

3

1

u

Usando essa transformacao a integral e facil de ser calculada, e e dada por:

∫ ∫ ∫

x2y + 3xyz dx dy dz =

∫ ∫ ∫1

3

uv + vw

|u| du dv dw =

=2

3+

1

3ln(2)

264

247. Considera o solido limitado abaixo pelo plano Oxy, doslados pela esfera ρ = 2

e acima pelo cone φ = π3 , conforme ilustra a figura abaixo:

a) encontre os limites de integracao em coordenadas esfericas para a integral

que calcula o volume desse solido.

b) Calcule a integral.

Solucao . As coordenadas esfericas estao ilusttradas na figura abaixo.

a) A partir das figuras acima, os limites de integracao em coordenadas esfericas sao dados

por: φ varia no intervalo [0, 2],θ varia no intervalo [0, π] e ρ varia no intervalo [ π3 , π

2 ].

b)Indique por Q a regiao em coordenadas cartesianas e por Q1 a regiao em coordenadas

esfericas. Como o Jacobiano das coordenadas esfericas e φ2, segue-se que o volume da

regiao e dado pela integral:

∫ ∫ ∫

Q

1 dx dy dz =

∫ ∫ ∫

Q

1 ρ2 sen(φ) dρ dθ dφ =

265

∫ 12 π

13 π

∫ 2π

0

∫ 2

0

ρ2 sen(φ) dρ dθ dφ

Calculando:

∫ 2

0

ρ2 sen(φ) dρ =8

3sen(φ)

∫ 2π

0

8

3sen(φ) dθ =

16

3sen(φ) π

∫ 12 π

13 π

16

3sen(φ)π dφ =

8

V =8

266

248. Encontre a area da superfıcie 2x( 32 ) + 2y( 3

2 ) − 3z = 0 acima do quadrado R: [0,

1]X[0, 1] no plano xy.

Solucao . Indique pore S a superfıcie que esta acima do quadrado e por A a sua area. A

primeiroa observacao e que S e o grafico da funcao f(x, y) = 2(x( 32)+y( 3

2))

3 com domınio

igual ao quadrado R, como ilustra a figura abaixo.

Como S e o grafico da funcao f, o elemento de area da superfıcie e dS =√

fx2 + fy2 + 1 dx dy,

onde fx = x e fy = y. Assim, a area de S e dada por:

A =

∫ ∫

R

fx2 + fy2 + 1 dx dy =

∫ ∫

R

x + y + 1 dx dy

Basta agora notar que o domınio R e descrito por o < x < 1 e 0 < y < 1. Daı:

A =

∫ 1

0

∫ 1

0

x + y + 1 dx dy =

∫ ∫

R

x + y + 1 dx dy =

=

∫ 1

0

2

3(2 + y)

32 − 2

3(y + 1)(

32 ) dy =

=12

5

√3 − 32

15

√2 +

4

15= 1.407

267

249. Encontre o volume da regiao triangular cortada do cilindro x2 + y2 = 1 pelos

planos z= -y e z= 0.

Solucao . A regiao esta ilustrada na figura abaixo.

Da figura e claro que a base B da regiao e a semi-circunferencia descrita da seguinte

maneira: para cada x fixo no intervalo [-1, 1], y varia no intervalo [−√

1 − x2, 0]. Daı

segue-se que a regiao R pode ser descrita como: para cada (x, y) fixo na base B, z varia

no intervalo [0, -y]. Usando essa descricao da cunha, claro entao que o seu volume V e

dado pela integral tripla:

V =

∫ 1

−1

∫ 0

−√

1−x2

∫ −y

0

1 dz dy dx

Calculando essa integral iteradamente, obtem-se que:

∫ −y

0

1 dz = −y

∫ 0

√1−x2

−y dy =1

2− −1

2x2

∫ 1

−1

1

2− 1

2x2 dx =

V =2

3

268

250. Encontre o volume do tetraedro no primeiro octante limitado pelos planos

coordenados e pelo plano que passa pelos pontos (1, 0, 0), (0, 2, 0)e(0, 0, 3).

Solucao . A regiao esta ilustrada na figura abaixo.

Calculando, obtem-se que a equacao do plano que passa pelos pontos dados e 6x+ 3y+

2z= 6. Em particular, esse plano corta o plano Oxy ao longo da reta 2x+ y= 2.

Da figura percebe-se que a base B do tetraedro no plano Oxy e a regiao triangular descrita

da seguinte maneira: para cada x fixo no intervalo [0, 1], y varia no intervalo [0, 2- 2x].

Daı segue-se que o tetraedro pode ser descrito como: para cada (x, y) fixo na base B, z

varia no intervalo [0, 1(6−6x−3y)2 ]. Usando essa descricao , segue-se que o volume V do

tetraedro e dado pela integral tripla:

V =

∫ 1

0

∫ 2−2x

0

∫ 3−3x− 32 y

0

1 dz dy dx

Calculando esa integral iteradamente, obten-se que o volume da regiao rada por:

V =

∫ 1

0

∫ 2−2x

0

∫ 3−3x− 32 y

0

1 dz dy dx

=

∫ 1

0

∫ 2−2x

0

3 − 3x − 3

2y dy dx =

=

∫ 1

0

6 − 6x − 3x(2 − 2x) − 3

2y dx = 1

269

251. Encontre o volume da regiao no primeiro octante limitada pelos planos coor-

denados, pelo plano y = 1 − xe pela superficie z = cos( π x2 ), com x no intervalo

[0,1].

Solucao . A regiao esta ilustrada na figura abaixo, em que a superficie z= cos πx2 esta

em verde e o plano y= 1- x em ocre.

Indique por R a regiao da qual se quer calcular o volume. Da figura percebe-se que a base

B de R e a regiao triangular descrita da seguinte maneira: para cada x fixo no intervalo

[0, 1], y varia no intervalo [0, 1- x]. Daı segue-se que R pode ser descrita como: para cada

(x, y) fixo na base B, z varia no intervalo [0, cos(π x2 )]. Usando essa descricao , segue-se

que o volume V de R e dado pela intagral tripla:

V =

∫ 1

0

∫ 1−x

0

∫ cos( 12 π x)

0

1 dz dy dx =

=

∫ 1

0

∫ 1−x

0

cos(1

2π x) dy dx =

=

∫ 1

0

cos(1

2π x) (1 − x) dx

V = 41

π2

270

252. Para a integral I =∫ 1

0

∫√y

y1 dx dy esboce a regiao de integracao e escreva

uma integral dupla equivalente com a ordem de integracao invertida.

Solucao . Na integral dada, para cada y fixo no intervalo [0, 1], x varia de x1= y ate

x2=√

y. Essa variacao esta ilustrada na cor azul na figura abaixo, e resulta na regiao de

integracao hachurada.

Para inverter a ordem de integracao , observe que a regiao pode ser descrita da seguinte

forma: para cada x fixo no intervalo [0, 1], y varia no intervalo de y1 = x2 ate y2= x.

Essa nova maneira de descrever a reiao esta ilustrada em vermelha na figura acima. Daı

segue-se que a integral pode ser escrita como:

I =

∫ 1

0

∫ x

x2

1 dy dx

271

253. Encontre o volume da regiao no primeiro octante limitada pelos planos coor-

denados e pelos planos x + y = 1 e y + 2z = 2.

Solucao . A regiao esta ilustrada na figura abaixo:

Indique por R esta regiao . Para resolver o exercıcio, e mais facil tomar como basa B

da regiao R o triangulo no plano Oxz descrito da seguinte maneira: para cada x fixo no

intervalo[0, 1], z varia no intervalo [0, 1-x]. Daı segue-se que o voluma da regiao e dado

por:

V =

∫ 1

0

∫ 1−x

0

∫ 2−2z

0

1 dy dz dx

Calculando essa integral iteradamente, obten-se que o volume da regiao dado por:

V =

∫ 1

0

∫ 1−x

0

∫ 2−2x

0

1 dy dz dx

=

∫ 1

0

∫ 1−x

0

2 − 2z dz dx =

∫ 1

0

2 − 2x − (1 − x)2 dx =

=

∫ 1

0

1 − x2 dx =

=2

3

272

254. Encontre o volume da regiao no primeiro octente limitada pelos planos coor-

denados, pelo plano y+ z= 2 e pelo cilindro x= 4- y2:

Solucao . A regiao esta ilustada na figura abaixo.

Indique por R esta regiao . Da figura segue-se que a base B de R e a area no plano Oxy

descrita da seguinte maneira: para cada y fixo no intervalo [0, 2], x varia no intervalo [0,

4 − y2]. Daı segue-se que R pode ser descrita como: para cada (x, y) fixo na base B, z

varia no intervalo [0, 2-y]. Usando essa descricao da regiao , segue-se que o seu volume V

e dado pela integral tripla:

V =

∫ 2

0

∫ 4−y2

0

∫ 2−y

0

1 dz dx dy

Calculando essa integral iteradamente, obtem-se que o volume da regiao e dado por:

V =

∫ 2

0

∫ 4−y2

0

∫ 2−y

0

1 dz dx dy =

=

∫ 2

0

∫ 4−y2

0

2 − y dx dy =

=

∫ 2

0

8 − 2y2 − y(4 − y2) dy =20

3

273

255. Esboce a regiao de integracao e calcule a integral∫ 2

1

∫ y2

y1dxdy

Solucao . De acordo com os extremos de integracao , para cada y no intervalo [1,2],

x varia no intervalo [y, y2]. Essa variacao de x esta ilustrada na figura abaixo na cor

vermelha, juntamente com os graficos das funcoes x=y e x = y2 na cor azul. A regiao de

integracao esta ilustrada em azul.

∫ y2

y

1dx = y2 − y

Em seguida, calcula-se a outra integral, obtendo:

∫ 2

1

∫ y2

y

1dxdy =

∫ 2

1

y2 − ydy =

=5

6

274

256. Seja R a regiao no primeiro quadrante do plano Oxy limitada pelas hiperboles

xy = 1 e xy = 9 e pelas retas y = x e y = 4x. Use a tranzformacao x = uv ey = uv,

com 0 ≤ v e 0 ≤ u, para rescrever∫ ∫

R

√yx +

√xydxdy como uma integral sobre

uma regiao apropriada G do plano Ouv. Depois calcule a integral uv sobre G.

Solucao . A regiao R esta ilustrada na figura a baixo.

Usando a transformacao , tem-se que xy = u2 e yx = v2, com u e v positivos. Logo, as

hiperboles (no plano Oxy) correspondem as retas u=1 e u=3 (no plano Ouv), enquanto

que as retas (no plano Oxy) correspondem as retas v=1 e v=2 (no plano Ouv). Assim, a

regiao G no plano Ouv corresponde ao retangulo ilustrado abaixo.

Para o calculo da integral, e necessario ainda calcular o jacobiano da transformacao , que

e o determinante da matriz.

∂∂ux(u, v) ∂

∂uy(u, v)

∂∂v x(u, v) ∂

∂v y(u, v)

, =

1v v

− uv2 u

,

275

Calculando, obtem-se que o jacobiano ? dado por 2uv . Logo, usando a formula de mudanca

de variaveis, segue-se que:

∫ ∫

r

√y

x+√

xydxdy =

∫ 2

1

∫ 3

1

2(v + u)u

vdudv

∫ 2

1

52

3

1

v+ 8dv

=52

3ln(2) + 8

276

257. Integre f(x, y) = ycos(xy) sobre o retangulo [0, π] × [0, 1]

Solucao . A figura abaixo ilustra o grafico da funcao juntamente com a regiao de inte-

gracao (em vermelho) no plano Oxy.

Indique por D= [0, π] X [0, 1] a regiao de interacao , de modo que a integral a ser calculada

e:

∫ ∫

D

ycos(xy) dx dy

Neste caso, observe que a integral∫

ycos(xy) dy deve ser feita por partes, e e relaiva-

mente trabalhosa. ja a integral∫

ycos(xy)dx pode ser calculada rapidamente usando

uma substituicao simples. Assim, mais facil come car com a integral

∫ π

0

ycos(yx)dx = sin(πy)

Agora, e claro que o valor procurado e dado por

∫ ∫

D

ycos(yx)dxdy =

∫ 1

0

∫ π

0

ycos(yx)dxdy =

∫ 1

0

sin(πy)dz

= 21

π

277

258. - Mude a integral cartesiana∫ 1

1

∫√1−x2

−√

1−x2 2 1(1+x2+y2)2 dy dx para uma integral

polar equivalente. Entao calcule a integral polar.

Solucao . Para x no intervalo [-1, 1], as curvas y = −√

1 − x2 e y =√

1 − x2 descrevem

os semi-cırculos inferior e superior do cırculo de raio 1. Logo, a regiao de integracao e

o disco de raio 1. A figura abaixo ilustra esse disco juntamente com o grafico da funcao

f(x,y)= 2(1+x2+y2)2 .

Em coordenadas polares x = rcos(θ) e y = rsen(θ), o disco pode ser descrito como o

conjunto dos pontos (r, θ) para os quais θ esta no intervalo (0, 2π) e r esta no intervalo

(0,1). Essa variacao esta ilustrada na figura abaixo.

Apos essas consideracoes , e lembrando que, em coordenadas polares, dxdy= rdrdθ, segue-

se que

−11

∫ √1−x2

√1−x2

21

(1 + x2 + y2)2dy dx =

∫ 2π

0

∫ 1

0

2r

(1 + r2)2dr dθ

278

Isto transforma a integral cartesiana em uma integral polar equivalente. Para o calculo

da integral polar, basta usar a substituio u = 1 + r2 . Entao du= 2r dr, e portanto:

∫ 2π

0

∫ 1

0

2r

(1 + r2)2dr dθ =

∫ 2π

0

∫ 2

1

1

u2du dθ

=

∫ 2π

0

1

2dθ = π

279

259. Encontre o volume do solido no primeiro octante limitado pelos planos coor-

denados, pelo cilındro x2 + y2 = 4 e pelo plano z+y=3

Solucao . O primeiro passo e ilustrar a regiao de integracao . Para isso, observe que

a equacao x2 = y2 = 4 corresponde a um cilindro de raio 2 ao longo do eixo Oz. Ja a

equacao z+y=3 representa um plano pelos pontos (0,3,0) e (0,0,3) e que n/ ao cruza o

eixo Ox. A partir dessas observacoes , o solido pode ser iluatrado como na figura abaixo.

Da ilustracao acima e claro que o volume procurado corresponde ? integral da funcao

f(x,y)= 3 -y sobre o dominio D limitado pelo disco x2 +y2 = 4 no plano Oxy, com 0 ≤ x e

0 ≤ y. O dominio e tanto da forma Rx como da forma Ry, e o calculo usando uma forma

nao e significativamente mais facil do que a outra. Escolhendo, por exemplo, a forma Rx,

o volume pode ser calculado por meio das integrais:

∫ ∫

D

(3 − y) dx dy =

∫ 2

0

∫ √4− x2

0

(3 − y) dy dx =

=

∫ 2

0

3√

4 − x2 − 2 +1

2x2dx =

= −8

3+ 3 π

Acima, a integral∫ 2

0

√4 − x2 dx pode ser calculada por meio de uma substituicao trigono-

metrica. Mas e claro que seu valor corresponde e 14 da area do disco de raio 2, isto e

corresponde a pi22

4 = π

280

260. Esboce a regiao de integracao e calcule a integral:

∫ 2Π

π

∫ π

0

sin(x) + cos(y)dxdy

Solucao . Comecamos fazendo:

A regiao de integracao e o retangulo [0, π] × [π, 2π], ilustrado na figura abaixo (na cor

verde) juntamente com o grafico da funcao f(x, y) = sin(x) + cos(y).

A integral dupla deve ser calculada iteradamente. Para isso, primeiro calcula-se a integral:

∫ π

0

sin(x) + cos(y)dx = 2 + cos(y)π

Daı segue-se que:

∫ 2π

π

∫ π

0

sen(x) + cos(y)dxdy =

∫ 2π

π

2 + cos(y)πdy

Finalmente, calculando-se a integral do lado direito, abtem-se que:

∫ 2π

π

sen(x) + cos(y)dxdy = 2π

281

261. Use a transformacao x = v+u3 e y = v−2u

3 para calcular a integral,∫ ∫

2x2 − xy − y2dxdy

para a regiao R no primeiro quadrante limitada pelas retas y = −2x + 4, y =

−2x + 7, y = x − 2ey = x + 1.

Solucao . Comecamos fazendo

A regiao R esta ilustrada na figura abaixo.

Observe que, com a transformacao dada, tem-se 2x+y=v e x-y=u. Logo, as retas y=

-2x + 4 e y= -2x + 7 no plano Oxy correspondem as retas horizontais v=4 e v=7 no

plano Ouv. Analogamente, as retas y= x-2 e y= x+ 1 no plano Oxy correspondem as

retas verticais u=-1 e u= 2 no plano Ouv. A figura abaixo ilustra a regiao no plano Ouv

correspondente a regiao R.

Para o calculo da integral, primeiro e necessario calcular o jacobiano da transformacao ,

que e o determinante da matriz

282

det

13

−23

23

13

, =1

3

Segundo, substituindo 2x + y = v e x - y = u no integrando obtem-se

2x2 − xy − y2 = (2x + y)(x + y) = uv

logo, usando a formula de mudanca de variaveis, segue-se que

∫ ∫

R

2x2 − xy − y2dxdy =

∫ 7

4

∫ 2

−1

1

3uvdudv =

∫ 7

4

1

2vdv

=33

4

283

262. Determine a area da superfıcie da parte da superfıcie z = x2 + 2y que esta

acima da regiao triangular T no plano X com vertices (0,0) (1,0) e (1,1) .

Solucao . Como a area da superfıcie e calculado por

A(s) =

∫ ∫√

[fx(x, y)]2 + [fy(x, y)]2 + 1dA

Entao com f(x, y) = x2 + 2y , obtemos :

A =

∫ ∫

T

(2x)2 + (2)2 + 1dA =

∫ 1

0

∫ x

0

4x2 + 5dydx

=

∫ 1

0

x√

4x2 + 5dx =1

8· 2

3(4x2 + 5)3/2]10 =

1

12(27 − 5

√5)

284

263. Determine a area do paraboloide z = x2 + y2 que esta abaixo do plano z = 9

. O plano intercpta o paraboloide no cırculo x2 + y2 = 9, z = 9 . Portanto a

superfıcie dada esta acima do disco D com centro na origem e raio 3 .

Solucao . Sabemos que

A(s) =

∫ ∫

D

1 + (dz

dx)2 + (

dz

dy)2dA

portanto ,

A =

∫ ∫

D

1 + (∂z

∂x)2 + (

∂z

∂y)dA =

∫ ∫

D

1 + (2x)2 + (2y)2dA

Se convertermos para coordenadas polares iremos facilitar os calculos e achar o resultado

A =

∫ 2π

0

∫ 3

0

1 + 4r2rdrdθ =

∫ 2π

0

∫ 3

0

1

8

1 + 4r2(8r)dr

= 2π(1

8)2

3(1 + 4r2)3/2]30 =

π

6(37

√37 − 1)

285

264. Calcule∫ 2

−2

∫√4−x2

−√

4−x2

∫ 2√x2+2

(x2 + y2)dzdydx.

Solucao . Essa integral esta sobre a regiao

E : {(x, y, z)/2 ≤ x ≤ 2,−√

4 − x2,√

x2 + 2 ≤ z ≤ 2}

sabemos que :

∫ ∫ ∫

f(x, y, z)dV =

∫ ∫

[

∫ u2(x,y)

u1(x,y)

f(x, y, z)dZ]dA

Portanto teremos

∫ 2

−2

∫ √4−x2

−√

4−x2

∫ 2

√x2+y2

(x2 + y2)dzdydx =

∫ ∫

E

(x2 + y2)dV

=

∫ 2π

0

∫ 2

0

r2r2rdzdrdθ

=

∫ 2π

0

∫ 2

0

r3(2 − r)dr = 2π[1

2r4 − 1

5r5]20 =

16

286

265. Calcule∫ ∫

e(x2+y2+z2)3/2

, onde B e a bola unitaria B = (x, y, z)/x2 + y2 + z2 ≤ 1

como a fronteira de B e uma esfera, utilizaremos coordenadas esfericas sendo

B = (p, φ, θ)− ≤ ρ ≤ 1.0 ≤ 2π, 0 ≤ φ ≤ π . Alem disso , as coordenadas esfericas

sao convenientes , pois x2 + y2 + z2 = p2

Solucao .

∫ ∫

B

e(x2+y2+z2)3/2

dV =

∫ π

0

∫ 2π

0

∫ 1

0

e(p2)3/2

p2senφsenφdpdθdφ

=

∫ π

0

senφdφ

∫ 2π

0

∫ 1

0

p2ep3

dp

= [−cosφ]π0 (2π)[1

3ep3

]10 =4

3π(e − 1)

287

266. Calcular usando cordenadas polares

I =

e(−x2)dx

Solucao .Comecamos calculando o seu quadrado:

I2 = (

e(−x2)dx)(

e(−y2)dy) =>

∫ ∫

e(−x2−y2)

Em cordenadas polares, o primeiro quadrante corresponde ao domınio em que r esta no

intervalo (0 ,∞ ) e θ esta no intervalo (0, π2 ). Daı segue-se que, em cordenadas polares,

I2 =

∫ 12 π

0

∫ ∞

0

e(−r2)rdrd

onde a integral na variavel θ e imediata e a integral na variavel r e facil de ser calculada

! De fato, com a substituicao

r2 = u

tem-se que:

∫ ∞

0

e(−r2)rdr =

∫ ∞

0

1

2e(−u)du =

1

2

Daı segue-se que:

I2 =

∫1

2dθ =

1

e portanto I =√

π2

288

267. Esboce a regiao, expresse a area como integral dupla iterada e calcule. A

parabola x = −y2 e a reta y = x + 2.

Solucao .

∫ 1

−2

∫ −y2

y−2

dxdy =

∫ 1

−2

−y2 − y + 2 dy

= −y3

3− y2

2+ 2y

∣∣∣∣∣

1

−2

= −1

3− 1

2+ 2 −

(8

3− 2 − 4

)

=9

2

289

268. Esboce a regiao, expresse a area como integral dupla iterada e calcule. A

parabola x = y − y2 e a reta y = −x.

Solucao .

∫ 2

0

∫ y−y2

−y

dxdy =

∫ 2

0

2y − y2 dy

= y2 − y3

3

∣∣∣∣∣

2

0

= 4 − 8

3

=4

3

290

269. Esboce a regiao, expresse a area como integral dupla iterada e calcule. A

curva y = ex e as retas y = 0.

Solucao .

∫ ln2

0

∫ ex

0

dydx =

∫ ln2

0

ex dx

= ex

∣∣∣∣∣

ln2

0

= 2 − 1

= 1

291

270. Esboce a regiao, expresse a area como integral dupla iterada e calcule. As

curvas y = lnx e y = 2lnx e a reta x = e, no 1o quadrante.

Solucao .

∫ e

1

∫ 2lnx

lnx

dydx =

∫ e

1

lnx dx

= xlnx − x

∣∣∣∣∣

e

1

= (e − e) − (0 − 1)

= 1

292

271. Esboce a regiao, expresse a area como integral dupla iterada e calcule. As

parabolas x = y2 e x = 2y − y2.

Solucao .

∫ 1

0

∫ 2y−y2

y2

dxdy =

∫ 1

0

2y − 2y2 dy

= y2 − 2

3y3

∣∣∣∣∣

1

0

=1

3

293

272. Determine a area da superfıcie, a parte da superfıcie z = xy que esta dentro

do cilindro x2 + y2 = 1.

Solucao .

z = f(x, y) = xy = {(x, y)/0 ≤ x2 + y2 ≤ 1}

A(s) =

∫ ∫

D

y2 + x2 + 1dA

=

∫ 2π

0

∫ 1

0

(√

(r)2 + 1)rdrdθ

=

∫ 2π

0

[1

33√

((r)2 + 1)2]10dθ

=

∫ 2π

0

1

3(2√

2 − 1)dθ

=2π

3(2√

2 − 1)

294

273. Calcule∫ ∫ ∫

Ex2dV , onde E e limitado pelo plano XZ e os hemisferios y =

√9 − x2 − z2 e y =

√16 − x2 − z2.

Solucao .

∫ ∫ ∫

E

x2dV =

∫ π

0

∫ π

0

∫ 4

3

(ρ sin ϕ cos θ)2ρ2 sin ϕdρdϕdθ

=

∫ π

0

cos θdθ

∫ π

0

sin3 ϕdϕ

∫ 4

3

ρ4dρ

= [1

2θ +

1

4sin 2θ]π0 [−1

3(2 + sin2 ϕ) cos ϕ]π0 [

1

5ρ5]43

= (π

2)(

2

3)1

5(45 − 35)

=1562π

15

295

274. Calcule a integral, transformando para coordenadas esfericas.

∫ 3

−3

∫ √9−x2

−√

9−x2

∫√

9−x2−y2

0

z√

x2 + y2 + z2dzdydx

Solucao .

∫ 2π

0

∫ π2

0

∫ 3

0

(ρ2 cosϕ)(ρ2 sinϕ)dρdϕdθ =

∫ 2π

0

∫ π2

0

cos ϕ sin ϕdϕ

∫ 3

0

ρ4dρ

= [θ]2π0 [

1

2sin2 ϕ]

π20 [

1

5ρ5]30

= (2π)(1

2)(

243

5)

=243

296

275. Calcule a integral, fazendo uma mudanca de variavel apropriada∫ ∫

R(x +

y)ex2−y2

dA, onde R e o retangulo delimitado pelas retas x − y = 0, x − y = 2,

x + y = 0 e x + y = 3.

Solucao .

u = x + y

v = x − y

x = 12 (u + v)

y = 12 (u − v)

∂(x, y)

∂(u, v)

∣∣∣∣∣∣

12

12

12 − 1

2

∣∣∣∣∣∣

u = 0;u = 3; v = 0; v = 2

∫ ∫

R

(x + y)ex2−y2

dA =

∫ 3

0

∫ 2

0

ueuv(−1

2)dvdu

=1

2

∫ 3

0

[ueuv]20du

=1

2

∫ 3

0

(e2u − 1)du

=1

2[1

2e2u − u]30

=1

2(1

2e6 − 3 − 1

2)

=1

4(e6 − 7)

297

276. Calcule a integral, fazendo uma mudanca de variavel apropriada∫ ∫

Rex+ydA,

onde R e dado pela inequacao |x| + |y| ≤ 1.

Solucao .

u = x + y

v = x − y

x = 12 (u + v)

y = 12 (u − v)

∂(x, y)

∂(u, v)

∣∣∣∣∣∣

12

12

12 − 1

2

∣∣∣∣∣∣

|u| = |x + y| ≤ |x| + |y| ≤ 1 ⇒ −1 ≤ u ≤ 1

|v| = | − x + y| ≤ |x| + |y| ≤ 1 ⇒ −1 ≤ u ≤ 1

(1, 1), (1,−1), (−1,−1)e(−1, 1)

∫ ∫

R

ex+ydA =1

2

∫ 1

−1

∫ 1

−1

eududv

=1

2[eu]1−1[v]1−1

= e − 1

e

298

277. Calcule a area da regiao limitada pela elipse x2

a2 + y2

b2 = 1, onde (a > 0 e b > 0)

Solucao .

Temos que:

x = ar cos θ 0 ≤ r ≤ 1

y = br sin θ 0 ≤ θ ≤ 2π

Calculando o jacobiano temos:

j =

∣∣∣∣∣∣

a cos θ −ar sin θ

b sin θ br cos θ

∣∣∣∣∣

= abr cos2 θ + abr sin2 θ = abr

∫ 2π

0

∫ 1

0

abrdrdθ =ab

2

∫ 2π

0

(r2

2)

∣∣∣∣∣

1

0

=ab

2

∫ 2π

0

=ab

2(2π)

= abπ

299

278. Calcule∫ ∫

Rf(x, y)dxdy, onde R e a regiao limitada pelo quadrado |x| + |y| = 1

Solucao .

x = ±(1 ± y)

y = 1 + x

y = 1 − x

y = −1 + x

y = −1 − x

∫ 0

−1

∫ x+1

−x−1

dydx +

∫ 1

0

∫ −x+1

x−1

dydx =

∫ 0

−1

y|x+1−x−1dx +

∫ 0

−1

y|−x+1x−1 dx

=

∫ 0

−1

(x + 1) − (−x − 1)dx +

∫ 0

−1

(−x + 1) − (x − 1)dx

=

∫ 0

−1

(2x + 2)dx +

∫ 1

0

(−2x + 2)dx

= (x2 + 2x)|0−1 + (−x2 + 2x)|10= 2

300

279. Encontre o jacobiano da transformacao x = u, y = uv e esboce a regiao σ =

{(u, v) ∈ R2; 1 ≤ u ≤ 2, 1 ≤ uv ≤ 2} no plano uv.

Solucao .

∂(x, y)

∂(u, v)=

∣∣∣∣∣∣

∂x∂u

∂x∂v

∂y∂u

∂y∂v

∣∣∣∣∣∣

=

∣∣∣∣∣∣

1 0

v u

∣∣∣∣∣∣

= u

σ = {(u, v) ∈ R2; 1 ≤ u ≤ 2, 1 ≤ uv ≤ 2}1

u≤ v ≤ 2

u

301

280. Calcule usando coordenadas polares∫ ∫

Bxydxdy onde B e o circulo

x2 + y2 − 2y ≤ 0 , x ≥ 0.

Solucao .

x2 + y2 − 2y = 0

x2 + (y − 1)2 − 1 = 0

x2 + (y − 1)2 = 1

V x = r cos θ

V y = r sin θ

∫ π2

0

∫ 2 sin θ

0

r2 cos θ sin θrdrdθ =

∫ π2

0

[r4 cos θ sin θ

4|2 sin θ0 ]dθ

= 4

∫ π2

0

(cos θ sin5 θ)dθ

u = sin θ

du = cos θdθ

4

∫ π2

0

u5du = 4(u6

6|

π20 )

= 4(sin4 θ

6|

π20 )

=2

3

302

281. Calcule∫ ∫

R

3√

y−x1+y+xdxdy onde R e o triangulo de vertices (0, 0),(1, 0) e (0, 1)

Solucao .

u = y − x → u + v = 1 + 2y

v = 1 + y + x → y = u+v−12

∂(x, y)

∂(u, v)=

−1

2

(0, 0) → u = 0v = 1 → (0, 1)

(0, 1) → u = 1v = 2 → (1, 2)

(1, 0) → u = −1v = 2 → (−1, 2)

∫ 1

−1

∫ 2

1

3√

u

v

1

2dvdu =

1

2

∫ 1

−1

[(ln v) 3√

u]|21du

=

∫ 1

−1

[(ln 2) 3√

u]du

= [3

8(ln 2)

3√

u4]|1−1

=3

8ln 2

303

282. Calcule o volume do conjunto x2 + y2 ≤ 4 e x2 + y2 + z2 ≤ 9

Solucao .

∫ ∫ ∫

k

dxdydz =

∫ ∫

K′

[

∫√

9−x2−y2

−√

9−x2−y2

dz]dxdy =

∫ ∫

K′

2√

9 − x2 − y2dxdy

=

∫ 2

−2

∫ √4−x2

−√

4−x2

2√

9 − x2 − y2dxdy

=

∫ 2π

0

∫ 2

0

2r√

9 − r2drdθ

= −∫ 2π

0

∫ 2

0

√ududθ

= −∫ 2π

0

[u

32

38

|20]dθ

= −∫ 2π

0

10√

3 − 54

3dθ

= −(10√

3 − 54

3)θ|2π

0

=108π − 20π

√3

3

304

283. Mostre que o volume de uma esfera de raio ”a”e 4πa3

3 unidades de volume,

usando integral tripla.

Solucao .

0 ≤ ϕ ≤ π

0 ≤ θ ≤ 2π

0 ≤ ρ ≤ a

∫ 2π

0

∫ π

0

∫ a

0

ρ2 sin ϕdρdϕdθ =

∫ 2π

0

∫ π

0

(ρ3

3|a0 sin ϕ)dϕdθ

=

∫ 2π

0

∫ π

0

a3

3sin ϕdϕdθ

=a3

3

∫ 2π

0

(− cos ϕ|π0 )dθ

=2a3

3

∫ 2π

0

=4a3π

3

305

284. Calcule o volume do conjunto dado. x + y + z ≤ 1, x ≥ 0 ,y ≥ 0 e z ≥ 0

Solucao .

Fazendo z = 1 − x − y temos:

∫ 1

0

∫ 1−y

0

(1 − x − y)dxdy =

∫ 1

0

(x − x2

2− xy)

∣∣∣∣∣

1−y

0

=

∫ 1

0

(1 − y) − (1 − y)2

2− (1 − y)ydy

=

∫ 1

0

(1 − y − 1

2+ y − y2

2− y + y2dy

=

∫ 1

0

1

2− y +

y2

2dy

= (y

2− y2

2+

y3

6)

∣∣∣∣∣

1

0

=1

6

306

285. Calcule o volume do conjunto dado. x2 + y2 ≤ z ≤ 2x

Solucao .

Fazendo z = 2x − x2 − y2 ⇒ (x − 1)2 + y2 = 1

Utilizando coordenadas polares temos que:

x = 1 + r cos θ 0 ≤ r ≤ 1

y = r sin θ 0 ≤ θ ≤ 2π

∫ ∫

2x − x2 − y2dxdy =

∫ 2π

0

∫ 1

0

2(r cos θ + 1) − (r cos θ + 1)2 − (r sin θ)2rdrdθ

=

∫ 2π

0

∫ 1

0

2r cos θ + 2 − (r2 cos2 θ + 2r cos θ + 1) − (r2 sin2 θ)rdrdθ

=

∫ 2π

0

∫ 1

0

(2r cos θ + 2 − r2 cos2 θ − 2r cos θ − 1 − r2 sin2 θ)rdrdθ

=

∫ 2π

0

∫ 1

0

1 − r2(sin2 θ + cos2 θ)rdrdθ

=

∫ 2π

0

(r2

2− r4

4)

∣∣∣∣∣

1

0

=

∫ 2π

0

(1

2− 1

4)dθ

=

∫ 2π

0

1

4dθ

2

307

286. Utilize da integral dupla para calcular a area do conjunto dado. B e o conjunto

de todos (x,y) tais que ln x ≤ y ≤ 1 + ln x, y ≥ 0 e x ≤ e

Solucao .

∫ e

0

∫ 1+ln x

ln x

dydx =

∫ e

0

(1 + ln x − ln x)dx

=

∫ e

0

1dx

= x

∣∣∣∣∣

e

0= e

308

287. Utilize da integral dupla para calcular a area do conjunto dado. B = {(x, y) ∈R2|x3 ≤ y ≤ √

x}

Solucao .

x3 = x12

x3 − x12 = 0

x(x2 − x− 1

2) = 0

x = 0

x − x− 1

2= 0

x2 − 1√x

x2√

x − 1 = 0

x2x12 − 1 = 0

x32 = 1

x = 1

∫ 1

0

∫ √x

x3

dydx =

∫ 1

0

(√

x − x3)dx

=

∫ 1

0

(x12 − x3)dx

=x

32

32

− x4

4

∣∣∣∣∣

1

0

=2

3(1)

32 − 1

4

=8 − 3

12

=5

12

309

288. Calcule∫

A

∫sen2x

1+4y2 dxdy, onde A e o retangulo 0 ≤ x ≤ π2 , 0 ≤ y ≤ 1

2

Solucao .

∫ 12

0

∫ π2

0

sin2 x

1 + 4y2dxdy =

∫ 12

0

∫ π2

0

(1

2 + 8y2)1 − cos 2xdxdy

=

∫ 12

0

1

2 + 8y2(x − sin 2x

2)

∣∣∣∣∣

π2

0

dy

=

∫ 12

0

1

2 + 8y2(π

2)dy

=

∫ 12

0

π

4 + 16y2dy

=

∫ 12

0

π16

416 + y2

dy

16

∫ 12

0

1

( 12 )2 + y2

dy

16(112

arctany12

)

∣∣∣∣∣

12

0

16(2 arctan 2y)

∣∣∣∣∣

12

0

16(2

π

4)

=π2

32

310

289. Encontre o volume do solido dentro do cone z =√

x2 + y2 entre os planos z = 1

e z = 2.

Solucao .

V =

∫ Π2

0

∫ Π4

0

∫ 2secφ

secφ

ρ2senφ dρdφdΘ

=4

3

∫ Π2

0

∫ Π4

0

(8sec3φ − sec3φ)senφ dφdΘ

=28

3

∫ Π2

0

∫ Π4

0

sec3φ − senφ dφdΘ

=28

3

∫ Π2

0

∫ Π4

0

tgφsec2φdφdΘ

=28

3

∫ Π2

0

[1

2tg2φ

]Π4

0

=14

3

∫ Π2

0

=7π

3

311

290. Encontre o volume da regiao limitada abaixo pelo plano z = 0, lateralmente

pelo cilindro x2 + y2 = 1 e acima pelo paraboloide z = x2 + y2.

Solucao .

V = 4

∫ π2

0

∫ 1

0

∫ r2

0

dzrdrdΘ

= 4

∫ π2

0

∫ 1

0

r3 drdΘ

=

∫ π2

0

2

312

291. Encontre o volume da regiao limitada abaixo pelo paraboloide z = x2 + y2,

lateralmente pelo cilindro x2 + y2 = 1 e acima pelo paraboloide z = x2 + y2 + 1.

Solucao .

V = 4

∫ ı2

0

∫ 1

0

∫ r2+1

r2

dzrdrdΘ

= 4

∫ π2

0

∫ 1

0

r drdΘ

= 2

∫ π2

0

= π

313

292. Encontre o volume do solido cortado do cilindro espesso 1 ≤ x2 +y2 ≤ 2 e pelos

cones z = ±√

x2 + y2.

Solucao .

V = 8

∫ π2

0

∫ √2

1

∫ r

0

dzrdrdΘ

= 8

∫ π2

0

∫ √2

1

r2 drdΘ

= 8

(

2√

2 − 1

3

)∫ π

2

0

=4π(2

√2 − 1)

3

314

293. Encontre o volume da regiao que esta dentro da esfera x2 + y2 + z2 = 2 e fora

do cilindro x2 + y2 = 1.

Solucao .

V = 8

∫ π2

0

∫ √2

1

∫ √2−r2

0

dzrdrdΘ

= 8

∫ π2

0

∫ √2

1

r√

2 − r2 drdΘ

= 8

∫ π2

0

[

−1

3(2 − r2)3/2

]√

2

1

=8

3

∫ π2

0

=4π

3

315

294. Encontre o volume da regiao limitada pelo cilindro x2 + y2 = 4 e pelos planos

z = 0 e y + z = 4.

Solucao .

V =

∫ 2π

0

∫ 2

0

∫ 4−rsenΘ

0

dzrdrdΘ

=

∫ 2π

0

∫ 2

0

(4r − r2senΘ) drdΘ

= 8

∫ 2π

0

(

1 − senΘ

3

)

= 16π

316

295. Encontre o volume da regiao limitada acima pelo paraboloide z = 5− x2 − y2 e

abaixo pelo paraboloide z = 4x2 + 4y2.

Solucao .

V = 4

∫ π2

0

∫ 1

0

∫ 5−r2

4r2

dzrdrdΘ

= 4

∫ π2

0

∫ 1

0

(5r − 5r3) drdΘ

= 20

∫ π2

0

r2

2− r4

4|10 dΘ

= 5

∫ π2

0

=5π

2

317

296. Encontre o volume da regiao limitada acima pelo paraboloide z = 9− x2 − y2 e

abaixo pelo plano xy e que esta fora do cilindro x2 + y2 = 1.

Solucao .

V = 4

∫ π2

0

∫ 3

1

∫ 9−r2

0

dzrdrdΘ

= 4

∫ π2

0

∫ 3

1

(9r − r3) drdΘ

= 4

∫ π2

0

9r2

2− r4

4|31 dΘ

= 4

∫ π2

0

(81

4− 17

4

)

= 32π

318

297. Encontre o volume da regiao cortada do cilindro solido x2 +y2 ≤ 1 e pela esfera

x2 + y2 + z2 = 4.

Solucao .

V = 8

∫ π2

0

∫ 1

0

∫ √4−r2

0

dzrdrdΘ

= 8

∫ π2

0

∫ 1

0

r(4 − r2)1/2 drdΘ

= 8

∫ π2

0

[

−1

3(4 − r2)3/2

]1

0

= −8

3

∫ Π2

0

(

33/2 − 8)

=4π(8 − 3

√3)

3

319

298. Monte a integral interada para calcular∫∫∫

f(r,Θ, z) dzrdrdΘ sobre a regiao D

dada. D e o cilindro reto solido cuja base e a regiao entre as circunferencias

r = cosΘ e r = 2cosΘ e cujo topo esta no plano z = 3 − x.

Solucao .

∫ π2

−π2

∫ 2cosΘ

cosΘ

∫ 3−rsenΘ

0

f(r,Θ, z) dzrdrdΘ

320

299. Monte a integral iterada para calcular∫∫∫

f(r,Θ, z) dzrdrdΘ sobre a regiao D

dada. D e o prisma cuja base e o triangulo no plano xy limitado pelo eixo x e

pelas retas y = x e x = 1 e cujo topo esta no plano z = 2 − y.

Solucao .

∫ π4

0

∫ secΘ

0

∫ 2−rsenΘ

0

f(r,Θ, z) dzrdrdΘ

321

300. Monte a integral iterada para calcular∫∫∫

f(r,Θ, z) dzrdrdΘ sobre a regiao D

dada. D e o prisma cuja base e o triangulo no plano xy limitado pelo eixo y e

pelas retas y = x e y = 1 e cujo topo esta no plano z = 2 − x.

Solucao .

∫ π2

π4

∫ cosecΘ

0

∫ 2−rsenΘ

0

f(r,Θ, z) dzrdrdΘ

322

301. Calcule a integral em coordenadas esfericas.

∫ π

0

∫ π

0

∫ 2senφ

0

ρ2senφ dρdφdΘ

Solucao .

∫ π

0

∫ π

0

∫ 2senφ

0

ρ2senφ dρdφdΘ =8

3

∫ π

0

∫ π

0

sen4φdφdΘ

=8

3

∫ π

0

([

−sen3φ cosφ

4

0

+3

4

∫ π

0

sen2φdφ

)

= 2

∫ π

0

∫ π

0

sen2φdφ dΘ

=

∫ π

0

[

Θ − sen2Θ

Θ

0

=

∫ π

0

π dΘ

= π2

323

302. Calcule a integral em coordenadas esfericas.

∫ 2π

0

∫ π4

0

∫ 2

0

(ρcosφ)ρ2senφ dρdφdΘ

Solucao .

∫ 2π

0

∫ π4

0

∫ 2

0

(ρcosφ)ρ2senφ dρdφdΘ =

∫ 2π

0

∫ π4

0

4cosφsenφ dφdΘ

=

∫ 2π

0

[2sen2φ

]Π4

0dΘ

=

∫ 2π

0

= 2π

324

303. Calcule a integral em coordenadas esfericas.

∫ 2π

0

∫ π

0

∫ 1−cosφ2

0

ρ2senφ dρdφdΘ

Solucao .

∫ 2Π

0

∫ π

0

∫ 1−cosφ2

0

ρ2senφ dρdφdΘ =1

24

∫ 2π

0

∫ π

0

(1 − cosφ)3senφ dφdΘ

=1

96

∫ 2π

0

(1 − cosφ)4 |π0 dΘ

=1

96

∫ 2π

0

(24 − 0) dΘ

=16

96

∫ 2π

0

3

325

304. Calcule a integral em coordenadas esfericas.

∫ 3π2

0

∫ π

0

∫ 1

0

5ρ3sen3φdρdφdΘ

Solucao .

∫ 3π2

0

∫ π

0

∫ 1

0

5ρ3sen3φdρdφdΘ =5

4

∫ 3π2

0

∫ Π

0

sen3φdφdΘ

=5

4

∫ 3π2

0

([

−sen2φcosφ

3

0

+2

3

∫ π

0

senφ dφ

)

=5

6

∫ 3π2

0

[−cosφ]π0 dΘ

=5

3

∫ 3π2

0

=5π

2

326

305. Calcule a integral em coordenadas esfericas.

∫ 2π

0

∫ π3

0

∫ 2

secφ

3ρ2senφ dρdφdΘ

Solucao .

∫ 2π

0

∫ π3

0

∫ 2

secφ

3ρ2senφ dρdφdΘ =

∫ 2π

0

∫ π3

0

(8sec3φ)senφ dφdΘ

=

∫ 2π

0

[

−8cosφ − 1

2sec2φ

]π3

0

=

∫ 2π

0

[

(−4 − 2) −(

−8 − 1

2

)]

=5

2

∫ 2π

0

= 5π

327

306. Calcule a integral em coordenadas esfericas.

∫ 2π

0

∫ π4

0

∫ secφ

0

(ρcosφ)ρ2senφ dρdφdΘ

Solucao .

∫ 2π

0

∫ π4

0

∫ secφ

0

(ρcosφ)ρ2senφ dρdφdΘ =1

4

∫ 2π

0

∫ π4

0

tgφsec2φdφdΘ

=1

4

∫ 2π

0

[1

2tg2φ

]π4

0

=1

8

∫ 2π

0

4

328

307. Encontre o volume da porcao da esfera solida ρ ≤ a que esta entre os cones

φ = π3 e φ = 2π

3 .

Solucao .

V =

∫ 2π

0

∫ 2π3

π3

∫ a

0

ρ2senφ dρdφdΘ

=

∫ 2π

0

∫ 2π3

π3

a3

3senφ dφdΘ

=a3

3

∫ 2π

0

[−cosφ]2π3

π3

=a3

3

∫ 2π

0

(1

2+

1

2

)

=2πa3

3

329

308. Encontre o volume da regiao cortada da esfera solida ρ ≤ a pelos semi-planos

Θ = 0 e Θ = Π6 no 1o octante.

Solucao .

V =

∫ π6

0

∫ π2

0

∫ a

0

ρ2senφ dρdφdΘ

=a3

3

∫ π6

0

∫ π2

0

senφ dφdΘ

=a3

3

∫ π6

0

=a3π

18

330

309. Encontre o volume da regiao menor cortada da esfera solida ρ ≤ 2 pelo plano

z = 1.

Solucao .

V =

∫ 2π

0

∫ π3

0

∫ 2

secφ

ρ2senφ dρdφdΘ

=1

3

∫ 2π

0

∫ π3

0

8senφ − tgφsec2φdφdΘ

=1

3

∫ 2π

0

[

−8cosφ − 1

2tg2φ

]π3

0

=1

3

∫ 2π

0

[

−4 − 1

2(3) + 8

]

=1

3

∫ 2π

0

5

2dΘ

=5π

3

331

310. A integral impropria∫∞0

e−x2

dx importante na teoria da probabilidade e em

outras partes da Matematica. Calcularemos seu valor com um artifıcio in-

teligente que usa uma integral dupla impropria em coodenadas polares.

Solucao . Escrevemos

I =

∫ ∞

0

e−y2

dy.

Como nao importa a letra que utilizemos para variavel de integracao , temos

I2 =

(∫ ∞

0

e−x2

dx

)(∫ ∞

0

e−y2

dy

)

.

Colocando o primeiro fator para dentro do segundo sinal de integracao , podemos escrever

a exprecao acima na forma

I2 =

∫ ∞

0

(∫ ∞

0

e−x2

dx

)

e−y2

dy =

∫ ∞

0

(∫ ∞

0

e−x2

e−y2

dx

)

dy

=

∫ ∞

0

∫ ∞

0

e−(x2+y2)dxdy.

Essa integral dupla se da em regiao que e todo o primeiro quadrante do plano xy. Em

coordenadas polares, temos

I2 =

∫ π/2

0

∫ ∞

0

e−r2

rdrdθ =

∫ π/2

0

[

−1

2e−r2

]∞

0

dθ =

∫ π/2

0

1

2dθ =

π

4,

logo

I =1

2

√π

ou∫ ∞

0

e−x2

dx =1

2

√π.

Essa formula e praticamente notavel, pois sabe-se que a integral indefinida∫

e−x2

dx

e impossıvel de ser expressa como funcao elementar*.

* Existe uma historia famosa envolvendo Lord Kelvin, fısico escoces do seculo XIX. Certa

ocasiao , Kelvin perguntou a classe: ”Voces sabem o que e um matematico?”Ele foi ao

quadro-negro e escreveu∫ ∞

−∞e−x2

dx =√

π,

que e equivalente a. ”Um matematico”, continuou, ”e uma pessoa para quem isto e tao

obivio quanto o fato de dois mais dois ser quatro para voces”. Na verdade, essa formula

nao e obivia nem para o autor nem para qualquer dos matematicos que ele conhece. Poder-

se-ia concluir que Kelvin estava se exibindo e tentando desmoralizar sua classe.

332

311. Calcule o momento de inercia Ix de uma lamina ocupando a regiao R : 0 ≤x ≤ 1, 0 ≤ y ≤

√1 − x2 se a densidade de massa no ponto (x, y) e dada por

σ (x, y) = 3y3 gramas por centımetro quadrado.

Solucao .

Ix =

∫ ∫

R

σ (x, y) y2dxdy =

∫ 1

0

∫ √1−x2

0

(3y3)y2dydx =

∫ 1

0

[y6

2

]√

1−x2

0

dx

=1

2

∫ 1

0

(1 − x2

)3dx =

1

2

∫ 1

0

(1 − 3x2 + 3x4 − x6

)dx =

1

2

[

x − x3 +3x5

5− x7

7

]1

0

=8

35g.cm2.

333

312. Seja R o interior do triangulo no plano cujos vertices sao A = (0, 0) , B = (1, 2)

e C = (5, 14) .Calcule∫ ∫

Rdxdy.

Solucao . A area do triangulo ABC e o valor absoluto de

1

2

0 0 1

1 2 1

5 14 1

=1

2

1 2

5 14

=1

2(14 − 10) = 2

Se uma funcao f tem valores nao -negativos sobre a regiao R, entao

∫ ∫

R

f (x, y) dxdy

pode ser interpretada como o volume V do solido abaixo do grafico de f e acima da regiao

R. Frequentemente V pode ser encontrado pelos metodos apresentados no Cap.7 (fatias,

discos circulares, figuras cilındricas e assim por diante), e desta forma

∫ ∫

R

f (x, y) dxdy

pode ser calcuado.

334

313. Seja R o retangulo 0 ≤ x ≤ 1 e 0 ≤ y ≤ 1 esto e, sobre a diagonal y = x, seja

R2 a parte abaixo, sob a diagonal y = x. Suponha que

∫ ∫

R1

f (x, y) dxdy = 3,

∫ ∫

R1

g (x, y) dxdy = −2

∫ ∫

R2

f (x, y) dxdy = 5,

∫ ∫

R2

g (x, y) dxdy = 1.

Ache

(a)

∫ ∫

R

f (x, y) dxdy, (b)

∫ ∫

R

g (x, y) dxdy, (c)

∫ ∫

R

[4f (x, y) − 3g (x, y)] dxdy.

Solucao . (a) (Aditividade em relacao a regiao de integracao ). Seja R uma regiao

admissıvel e suponha que R possa ser dedecomposta em duas regioes admissıveis nao -

superpostas R1 e R2.(Nota: As regioes podem dividir pontos comuns de limites.) Se

f e Riemann-integravel sobre as regioes R1 e R2, entao f e Riemann-integravel sobre

R. Logo,

∫ ∫

R

f (x, y) dxdy =

∫ ∫

R1

f (x, y) dxdy +

∫ ∫

R2

f (x, y) dxdy = 3 + 5 = 8.

(b) Assim como em (a)

∫ ∫

R

g (x, y) dxdy =

∫ ∫

R1

g (x, y) dxdy +

∫ ∫

R2

g (x, y) dxdy = −2 + 1 = −1.

(c) Usando (a) e (b) e o fato de (Propriedade linear).Se f e g sao funcoes Riemann-

integraveis sobre a regiao admissıvel R e se A e B sao constantes, entao Af ± Bg e

tambem Riemann-integravel Sobre R, assim temos

∫ ∫

R

[4f (x, y) − 3g (x, y)] dxdy = 4

∫ ∫

R

f (x, y) dxdy−3

∫ ∫

R

g (x, y) dxdy = (4) (8)−(3) (−1) = 35.

335

314. Calcule a integral iterada∫ 2

−1

∫ 2

0x2y3dydx

Solucao . Pelo teorema de Fubini

∫ 2

−1

∫ 2

0

x2y3dydx =

∫ x=2

x=−1

[∫ y=2

y=0

x2y3dy

]

dx

=

[x2y4

4

]y=2

y=0

dx =

∫ x=2

x=−1

[16x2

4− 0

]

dx

=

[4

3x3

]2

−1

=

(32

3

)

−(

−4

3

)

= 12.

336

315. Calcule a integral iterada∫ 4

0

∫ 3x/2

0

√16 − x2dydx

Solucao . Pelo Teorema de Fubibi

∫ 4

0

∫ 3x/2

0

16 − x2dydx =

∫ x=4

x=0

[∫ y=3x/2

y=0

16 − x2dy

]

dx

=

∫ x=4

x=0

[√

16 − x2

∫ y=3x/2

y=0

dy

]

dx =

∫ x=4

x=0

16 − x2 [y]y=3x/2y−0 dx

=

∫ 4

0

16 − x2

(3x

2

)

dx.

A integral posterior pode ser calculada usando a substituicao u = 16 − x2, seguindo-se

que du = −2x, xdx = − 12du e

(3x2

)dx = − 3

4du. Visto que u = 16 quando x = 0 e u = 0

quando x = 4, temos

∫ 4

0

16 − x2

(3x

2

)

dx =

∫ 0

16

√u

(

−3

4

)

du =3

4

∫ 16

0

√udu =

3

4

[2

3u3/2

]16

0

= 32.

337

316. Calcule a integral iterada∫ π

0

∫ y2

0sin(

xy

)

dxdy

Solucao . Pelo Teorema de Fubini

∫ π

0

∫ y2

0

sin

(x

y

)

dxdy =

∫ π

0

[∫ y2

0

sin

(x

y

)

dx

]

dy =

∫ π

0

[

−y cosx

y

]y2

0

dy =

∫ π

0

(

−y cosy2

y+ y cos

0

y

)

dy

=

∫ π

0

(y − y cos y) dy =

∫ π

0

ydy −∫ π

0

y cos ydy

[Integrando por partes para u = y, dv = cos ydy, du = dy e v = sin y temos ]

=

[y2

2

0

− [y sin y + cos y]π0 =

π2

2− (π sin π + cosπ) + (0 sin 0 cos 0) =

π2

2+ 2.

338

317. A carga eletrica e distribuıda sobre a regiao R triangular 0 ≤ y ≤ 1 e 0 ≤ x ≤ 1.

Visto que a densidade de carga em qualquer ponto (x, y) em R e dada por

σ (x, y) =(x − x2

) (y − y2

)Coulomb/cm2. Ache a soma total de carga eletrica na

regiao R.

Solucao . A carga total na superfıcie R e dada por

∫ ∫

R

σ (x, y) dxdy =

∫ 1

0

∫ 1

x

(x − x2

) (y − y2

)dydx

=

∫ 1

0

(x − x2

)[(

y2

2− y3

3

)]x

0

dx

=

∫ 1

0

(x3

2− 5x4

6+

x5

3

)

dx =

[x4

8− x5

6+

x6

18

]1

0

=1

72coulomb.

339

318. Calcule os momentos de inercias Ix, Iy, e I0 de uma lamina quadrada cujos

lados medem 2 centımetros de comprimento , sao paralelos aos eixos x e y e

cujo centro esta na origem. Suponha que a lamina e homogenia (isto, e sua

massa e distribuıda uniformemente) e que sua massa total e de 8 gramas.

Solucao . A area de lamina vale 4 centımetros quadrados. Visto que ela e homogenia,

sua densidade de massa e uma constante, 82 gramas por centımetro quadrado. Logo,

σ (x, y) = 2 para todos os pontos (x, y) dentro da lamina. Logo,

Ix =

∫ ∫

R

σ (x, y) y2dxdy = 2

∫ ∫

R

y2dxdy = 2

∫ 1

−1

∫ 1

−1

y2dxdy

= 2

∫ 1

−1

2y2dy =

[4y3

3

]1

−1

=8

3g.cm2

Iy =

∫ ∫

R

σ (x, y) x2dxdy = 2

∫ ∫

R

x2dxdy = 2

∫ 1

−1

∫ 1

−1

x2dxdy

=2

3

∫ 1

−1

[x3]1

−1dy =

2

3

∫ 1

−1

2dy =8

3g.cm2

I0 =

∫ ∫

R

σ (x, y)(x2 + y2

)dxdy = Ix + Iy =

16

3g.cm2.

340

319. Determine o volume do solido que esta sob o paraboloide z = x2 + y2, acima

do plano xy, e dentro do cilindro x2 + y2 = 2x .

Solucao . A fronteira do disco D possui a seguinte equacao x2 + y2 = 2x ou apos

completar os quadrados (x − 1) + y2 = 1 Para achar as coodenadas polares, faremos

x2 + y2 = r2 e x = rcosθ , entao a fronteira circular r2 = 2cosθ, ou r = 2cosθ . Portanto

o disco D vai possuir uma regiao igual a :

D = {(r, θ)/ − π/2 ≤ θ ≤ π/2, 0 ≤ r ≤ 2cosθ}

e se F continua em uma regiao polar da forma

D = {(r, θ/ α ≤ θ} ≤ β, h1(θ) ≤ r ≤ h2(θ)

Entao∫ ∫

(x, y)dA =

∫ β

α

∫ h2

h1

f(r(cosθ, rsenθ)rdrdθ

. Portanto ,

V =

∫ ∫

D

(x2 + y2)dA =

∫ π/2

−π/2

∫ 2cosθ

0

r2rdrdθ

=

∫ π/2

−π/2

[r4

4]2cosθ0 dθ = 4

∫ π/2

−π/2

cos4θdθ

= 8

∫ π/2

0

cos4θdθ = 8

∫ π/2

0

(1 + cos2θ

2

)2

= 2

∫ π/2

0

[1 + 2cos2θ +1

2(1 + cos4θ)]dθ

= 2[3

2θ + sen2θ +

1

8sen4θ]

π/20 = 2(

3

2)(

π

2) =

2

341

320. Determine a massa e o centro de massa de uma lamina triangular com vertices

(0,0), (1,0) e (0,2) se a funcao densidade e p(x,y) = 1 + 3x + y.

Solucao .O triangulo tem equacao da fronteira superior igual a y = 2 - 2x . Entao a

massa da lamina sera.

M =

∫ ∫

D

p(x, y)dA =

∫ 1

0

∫ 2−2x

0

(1 + 3x + y)dydx

M =

∫ 1

0

[

y + 3xy +y2

2

]y=2−2x

y=0

dx

M = 4

∫ 1

0

(1 − x2)dx = 4

[

x − x3

3

]1

0

=8

3

Achada a massa acharemos o centro de massa da seguinte forma :

X =1

m

∫ ∫

D

xp(x, y)dA =3

8

∫ 1

0

∫ 2−2x

0

(x + 3x2 + xy)dydx

=3

8

∫ 1

0

[

xy + 3x2y + xy2

2

]y=2−2x

y=0

dx

=3

2

∫ 1

0

(x − x3)dx =

[x2

2− x4

4

]1

0

=3

8

y =1

m

∫ ∫

D

yp(x, y)dA =3

8

∫ 2−2x

0

(y + 3xy + y2)dydx

=3

8

∫ 1

0

[y2

2+ 3x

y2

2+

y3

3

]y=2−2x

y=0

dx =1

4

∫ 1

0

(7 − 9x − 3x2 + 5x3)dx

=1

4

[

7x − 9x2

2− x3 + 5

x4

4

]1

0

=11

16

Portanto o centro de massa sera ( 38 , 11

16 )

342

321. A densidade em qualquer ponto de uma lamina semicircular e proporcinonal

a distancia do centro do cırculo.Determine o centro de massa da lamina .

Solucao .Se considerarmos a lamina como sendo a metade superior do cırculo x2+y2 = a2

teremos a distancia do ponto (x,y) , ao centro do cırculo que e a origem , e√

x2 + y2.

A funcao densidade sera : p(x, y) = k√

x2 + y2 , sendo k uma constante . Esta equacao

da funcao densidade e o formato da lamina nos leva a uma conversao para coordenadas

polares. Entao√

x2 + y2 = r e a regiao D e dada por 0 ≤ r ≤ a, 0 ≤ θ ≤ π. A massa da

lamina sera portanto :

M =

∫ ∫

D

p(x, y)dA =

∫ ∫

D

k√

x2 + y2dA

=

∫ π

0

∫ a

0

(kr)rdrdθ = k

∫ π

0

∫ a

0

r2dr

= kπr3

3]a0 =

kπa3

3

A funcao densidade e a lamina sao simetricas com relacao ao eixo y, por isso que o centro

de massa tem que estar sobre o eixo y ou seja x = 0 , entao tentaremos encontrar a

coordenada y do seguinte modo :

y =1

m

∫ ∫

D

yp(x, y)dA =3

kπa3

∫ π

0

∫ a

0

rsenθ(kr)rdrdθ

=3

πa3

∫ π

0

senθdθdθ

∫ a

0

r3dr =3

πa3[−cosθ]π0 [

r4

4]a0

=3

πa3

2a4

4=

3a

O centro de massa, portanto esta localizado no seguinte ponto (0, 3a/(2π)).

343

322. Considere o conjunto S = f(x, y, z)2 r3 : 0 ≤ x ≤ 1;x2−1 ≤ y ≤ x2, x3 ≤ z ≤ x3+2g

e a funcao g: r3 ! r3 definida por g(x, z, z) = (x; y − x2, z − x3) :

i) mostre que a funcao ge uma mudanca de coordenadas.

ii) Use a mudanca de coordenadas g para calcular o integral∫

z−x3

1+x2

Solucao .A funcao g sendo polinomial e claramente de classe c1 . A funcao g e injectativa.

De fato, se g(x1, y1, z1) = g(x2, y2, z2) entao (x1; y1− x21; z1− x3) = (x2, y2− x22, z2−x32) e portanto , x1 = x2 ; y1 = y2 ; z1 = z2 A derivada de ge representada pela matriz

Dg(x; y; z) =

∣∣∣∣∣∣∣∣∣

1 0 0

−2x 1 0

−3x2 0 1

∣∣∣∣∣∣∣∣∣

e, portanto , detDg ( x; y; z ) = 1 = 0 . Assim , ge uma mudanca de coordenadas emR3

da descricao do conjunto S temos 0 < x < 1;−1 < y − x2 < 0; 0 < z − x3 < 2 e fazendo

( u , v, w ) - g (x, y , z ) , obtemos 0 ¡ u ¡ 1 -1 ¡ v ¡ 0 0 ¡ w ¡ 2 . Note- se que atraves

da funcao g ao conjunto S corresponde o intervalo t = g(s) = (u, v, w) ∈ R3 : 0 < u <

1;−1 < v < 0; 0 < w < 2 Note -se que atraves da funcao g , ao conjunto S corresponde o

intervalo T = g(s) = {(u, vw) ∈ R3 : 0 < u < 1;−1 < v < 0; 0 < w < 2} Portanto,

∫z − x2

1 + x2dxdz =

∫ 1

0

(

∫ 0

−1

(

∫ 2

0

w

1 + u2dw))dv)du

= 2

∫ 1

0

(

∫ 0

−1

1

1 + u2dv)du

= 2

∫ 1

0

1

1 + u2=

π

2

344

323. Determine a area da superfıcie da parte da superfıcie z = x2 + 2y que esta

acima da regiao triangular T no plano X com vertices (0,0) (1,0) e (1,1) .

Solucao .Como a area da superfıcie e calculado por

A(s) =

∫ ∫√

[fx(x, y)]2 + [fy(x, y)]2 + 1dA

Entao com f(x, y) = x2 + 2y , obtemos :

A =

∫ ∫

T

(2x)2 + (2)2 + 1dA =

∫ 1

0

∫ x

0

4x2 + 5dydx

=

∫ 1

0

x√

4x2 + 5dx =1

8· 2

3(4x2 + 5)3/2]10 =

1

12(27 − 5

√5)

345

324. Determine a area do paraboloide z = x2 + y2 que esta abaixo do plano z = 9

. O plano intercpta o paraboloide no cırculo x2 + y2 = 9, z = 9 . Portanto a

superfıcie dada esta acima do disco D com centro na origem e raio 3 .

Solucao .Sabemos que

A(s) =

∫ ∫

D

1 + (dz

dx)2 + (

dz

dy)2dA

portanto ,

A =

∫ ∫

D

1 + (∂z

∂x)2 + (

∂z

∂y)dA =

∫ ∫

D

1 + (2x)2 + (2y)2dA

Se convertermos para coordenadas polares iremos facilitar os calculos e achar o resultado

A =

∫ 2π

0

∫ 3

0

1 + 4r2rdrdθ =

∫ 2π

0

∫ 3

0

1

8

1 + 4r2(8r)dr

= 2π(1

8)2

3(1 + 4r2)3/2]30 =

π

6(37

√37 − 1)

346

325. Calcule∫ 2

−2

∫√4−x2

−√

4−x2

∫ 2√x2+2

(x2 + y2)dzdydx.

Solucao .Essa integral esta sobre a regiao

E : {(x, y, z)/2 ≤ x ≤ 2,−√

4 − x2,√

x2 + 2 ≤ z ≤ 2}

sabemos que :

∫ ∫ ∫

f(x, y, z)dV =

∫ ∫

[

∫ u2(x,y)

u1(x,y)

f(x, y, z)dZ]dA

Portanto teremos

∫ 2

−2

∫ √4−x2

−√

4−x2

∫ 2

√x2+y2

(x2 + y2)dzdydx =

∫ ∫

E

(x2 + y2)dV

=

∫ 2π

0

∫ 2

0

r2r2rdzdrdθ

=

∫ 2π

0

∫ 2

0

r3(2 − r)dr = 2π[1

2r4 − 1

5r5]20 =

16

347

326. Calcule o volume do conjunto de todos (x,y,z) tais que 0 ≤ x ≤ 1, 0 ≤ y ≤ 1 e

0 ≤ z ≤ x2 + y2.

Solucao . Seja D = [0, 1]× [0, 1]. O volume de tal conjunto e∫

D

∫(x2 +y2)dxdy. Logo,

∫ 1

0

∫ 1

0

(x2 + y2) dxdy =

∫ 1

0

x3

3+ y2

∣∣∣∣∣

1

0

dy

=

∫ 1

0

1

3+ y2dy

=1

3y +

y3

3

∣∣∣∣∣

1

0

=1

3+

1

3

=2

3

348

327. Calcule∫

B

∫xy dxdy onde B e o conjunto de todos (x,y) tais que 0 ≤ x ≤ 1,

0 ≤ y ≤ x2.

Solucao . Se integrarmos primeiro em relacao a y, obteremos:

B

xy dxdy =

∫ 1

0

∫ x2

0

xy dydx

=

∫ 1

0

xy2

2

∣∣∣∣∣

x2

0

dx

=

∫ 1

0

xx4

2dx

=x6

12

∣∣∣∣∣

1

0

=1

12

349